50 CHAPTER Care of Patients with Musculoskeletal Problems, MUSKOSLETAL, Lewis Ch. 63 Musculoskeletal Problems, Chapter 63: Musculoskeletal Problems Lewis: Medical-Surgical Nursing, 10th Edition, Musculoskeletal NCLEX style questions, Lewis Ch 64 Musc...

Lakukan tugas rumah & ujian kamu dengan baik sekarang menggunakan Quizwiz!

The daily requirement of calcium for an adult woman is

1200 mg

A patient with osteomyelitis is to receive vancomycin (Vancocin) 500 mg IV every 6 hours. The vancomycin is diluted in 100 mL of normal saline and needs to be administered over 1 hour. The nurse will set the IV pump for how many milliliters per minute? (Round to the nearest hundredth.)

ANS: 1.67 To administer 100 mL in 60 minutes, the IV pump will need to provide 1.67 mL/min. DIF: Cognitive Level: Understand (comprehension) REF: 1497 TOP: Nursing Process: Implementation MSC: NCLEX: Physiological Integrity

A nurse provides discharge teaching for a client who had a total hip replacement. Which activities to avoid identified by the client indicate an understanding of the teach?

Crossing the legs. Sitting in a low chair.

What nursing intervention is used for osteomyelitis (also used for wound debridement)

Cultures or bone biopsies

You are teaching a patient with osteopenia. What is important to include in the teaching plan? a.Lose weight. b.Stop smoking. c.Eat a high-protein diet. d.Start swimming for exercise

b.Stop smoking. Patients with osteopenia should be instructed to quit smoking in order to decrease loss of bone mass.

Lengthy Antibiotic therapy can result in overgrowth of what?

candida albicans

What diagnoses acute osteomyelitis?

cultures or bone biopsies

To confirm a diagnosis of low back pain, which of the following diagnostic procedures would be ordered to rule out the presence of a tumor? Bone scan Computed tomography Magnetic resonance imaging Electromyogram

A bone scan is the preferred diagnostic procedure to disclose tumors in a patient with low back pain.

When the nurse is reviewing laboratory data for a patient who is taking methotrexate (Rheumatrex) to treat rheumatoid arthritis, which information is most important to communicate to the health care provider? a. The blood glucose is 75 mg/dL. b. The rheumatoid factor is positive. c. The white blood cell (WBC) count is 1500/L. d. The erythrocyte sedimentation rate is elevated.

ANS: C Bone marrow suppression is a possible side effect of methotrexate, and the patient's low WBC count places the patient at high risk for infection. The elevated erythrocyte sedimentation rate and positive rheumatoid factor are expected in rheumatoid arthritis. The blood glucose is normal.

The incidence of what type of fracture is highest in older adults?

Hip

How long does acute osteomyelitis last?

less than one month

Vascular compromise and ischemia caused by an increase in the bone is caused by the growth of what?

microorganisms

Monitor what levels when treating patients with antibiotics?

peak and trough levels

A client with osteoporosis is prescribed calcitonin (Miacalcin) 100 units subcutaneously. The medication is available 200 units per ml. How many milliliters will the nurse administer to the client?

0.5 Explanation: 100 units x 1 ml/200 units = 0.5 ml. pg.2065

When preparing to administer an ordered blood transfusion, which IV solution does the nurse use when priming the blood tubing? Lactated Ringer's 5% dextrose in water 0.9% sodium chloride 0.45% sodium chloride

0.9% sodium chloride

What are the five diagnostic tools for appendicitis?

1. CT Scan (Preferred) 2. Ultrasound (Preferred for children and pregnant women) 3. Urinalysis (To rule out UTI) 4. Abdominal X-Ray 5. Pelvic Exam

7. Which statement describes osteosarcoma? a High rate of local recurrence b. Very malignant and metastasizes early c. Arises in cancellous ends of long bones d. Develops in the medullary cavity of long bones

7. b. Osteosarcoma, the most common primary bone cancer, occurs in the metaphyseal region of long bones of the arms, legs, or pelvis. It is extremely malignant and metastasizes early and is often brought to attention by injury. A high rate of local recurrence occurs with osteoclastoma that arises in cancellous ends of long bones. Ewing's sarcoma develops in the medullary cavity of long bones as above.

The nurse is instructing a 65-year-old female client diagnosed with osteoporosis. The most important instruction regarding exercise would be to: a-Exercise doing weight bearing activities. b-Exercise to reduce weight. c-Avoid exercise activities that increase the risk of fracture. d-Exercise to strengthen muscles and thereby protect bones.

A: Weight bearing exercises are beneficial in the treatment of osteoporosis. B, C, D: Although loss of bone cannot be substantially reversed, further loss can be greatly reduced if the client includes weight bearing exercises along with estrogen replacement and calcium supplements in their treatment protocol.

Which information will the nurse include when teaching a patient with newly diagnosed ankylosing spondylitis (AS) about the management of the condition? a. Exercise by taking long walks. b. Do daily deep breathing exercises. c. Sleep on the side with hips flexed. d. Take frequent naps during the day.

ANS: B Deep breathing exercises are used to decrease the risk for pulmonary complications that may occur with the reduced chest expansion that can occur with ankylosing spondylitis (AS). Patients should sleep on the back and avoid flexed positions. Prolonged standing and walking should be avoided. There is no need for frequent naps.

Which finding will the nurse expect when assessing a 60-year-old patient who has osteoarthritis (OA) of the left knee? a. Heberden's nodules b. Pain upon joint movement c. Redness and swelling of the knee joint d. Stiffness that increases with movement

ANS: B Initial symptoms of OA include pain with joint movement. Heberden's nodules occur on the fingers. Redness of the joint is more strongly associated with rheumatoid arthritis (RA), and stiffness in OA is worse right after the patient rests and decreases with joint movement.

The health care provider has prescribed the following collaborative interventions for a 49-year-old who is taking azathioprine (Imuran) for systemic lupus erythematosus. Which order will the nurse question? a. Draw anti-DNA blood titer. b. Administer varicella vaccine. c. Use naproxen (Aleve) 200 mg BID. d. Take famotidine (Pepcid) 20 mg daily.

ANS: B Live virus vaccines, such as varicella, are contraindicated in a patient taking immunosuppressive drugs. The other orders are appropriate for the patient.

To determine whether a patient with joint swelling and pain has systemic lupus erythematosus, which test will be most useful for the nurse to review? a. Rheumatoid factor (RF) b. Antinuclear antibody (ANA) c. Anti-Smith antibody (Anti-Sm) d. Lupus erythematosus (LE) cell prep

ANS: C The anti-Sm is antibody found almost exclusively in SLE. The other blood tests also are used in screening but are not as specific to SLE.

The nurse should reposition the patient who has just had a laminectomy and diskectomy by a. instructing the patient to move the legs before turning the rest of the body. b. having the patient turn by grasping the side rails and pulling the shoulders over. c. placing a pillow between the patient's legs and turning the entire body as a unit. d. turning the patient's head and shoulders first, followed by the hips, legs, and feet.

ANS: C The spine should be kept in correct alignment after laminectomy. The other positions will create misalignment of the spine

After teaching a patient with a bunion about how to prevent further problems, the nurse will determine that more teaching is needed if the patient says, a. "I will throw away my high heel shoes." b. "I will use the bunion pad to relieve the pain." c. "I will need to wear open sandals at all times." d. "I will take ibuprofen (Motrin) when I need it."

ANS: C The patient can wear shoes that have a wide forefoot. The other patient statements indicate that the teaching has been effective.

When administering alendronate (Fosamax) to a patient, the nurse will first a. be sure the patient has recently eaten. b. ask about any leg cramps or hot flashes. c. assist the patient to sit up at the bedside. d. administer the ordered calcium carbonate.

ANS: C To avoid esophageal erosions, the patient taking bisphosphonates should be upright for at least 30 minutes after taking the medication. Fosamax should be taken on an empty stomach, not after taking other medications or eating. Leg cramps and hot flashes are not side effects of bisphosphonates.

The nurse evaluating effectiveness of prescribed calcitonin and ibandronate (Boniva) for a patient with Paget's disease will consider the patient's a. oral intake. c. grip strength. b. daily weight. d. pain intensity.

ANS: D Bone pain is a common early manifestation of Paget's disease, and the nurse should assess the pain intensity to determine if treatment is effective. The other information will also be collected by the nurse but will not be used in evaluating the effectiveness of the therapy. DIF: Cognitive Level: Apply (application) REF: 1514 TOP: Nursing Process: Evaluation MSC: NCLEX: Physiological Integrity

A long-term care patient who takes multiple medications develops acute gouty arthritis. The nurse will consult with the health care provider before giving the prescribed dose of a. sertraline (Zoloft). b. famotidine (Pepcid). c. oxycodone (Roxicodone). d. hydrochlorothiazide (HydroDIURIL).

ANS: D Diuretic use increases uric acid levels and can precipitate gout attacks. The other medications are safe to administer.

Which menu choice by a patient with osteoporosis indicates the nurse's teaching about appropriate diet has been effective? a. Pancakes with syrup and bacon b. Whole wheat toast and fresh fruit c. Egg-white omelet and a half grapefruit d. Oatmeal with skim milk and fruit yogurt

ANS: D Skim milk and yogurt are high in calcium. The other choices do not contain any high-calcium foods. DIF: Cognitive Level: Apply (application) REF: 1512 TOP: Nursing Process: Evaluation MSC: NCLEX: Physiological Integrity

A patient is receiving gentamicin (Garamycin) 80 mg IV twice daily for acute osteomyelitis. Which action should the nurse take before administering the gentamicin? a. Ask the patient about any nausea. b. Obtain the patient's oral temperature. c. Change the prescribed wet-to-dry dressing. d. Review the patient's blood urea nitrogen (BUN) and creatinine levels.

ANS: D Gentamicin is nephrotoxic and can cause renal failure. Monitoring the patient's temperature before gentamicin administration is not necessary. Nausea is not a common side effect of IV gentamicin. There is no need to change the dressing before gentamicin administration.

Which of these is a risk factor for osteoporosis a-Age and family history b-Obesity and smoking c-All of the above

All of the above

You are teaching a patient with osteopenia. What is important to include in the teaching plan? a )Lose weight. b )Stop smoking. c )Eat high protein diet. d )Start swimming for exercise.

Answer b

9. A patient with a herniated intravertebral disk undergoes a laminectomy and diskectomy. Following the surgery, the nurse should position the patient on the side by a. elevating the head of the bed 30 degrees and having the patient extend the legs while turning to the side. b. turning the patient's head and shoulders and then the hips, keeping the patient centered in the bed. c. having the patient turn by grasping the side rails and pulling the shoulders over. d. placing a pillow between the patient's legs and turning the entire body as a unit.

Answer: D Rationale: Logrolling is used to maintain correct body alignment after laminectomy. The other positions will create misalignment of the spine. Cognitive Level: Application Text Reference: p. 1683 Nursing Process: Implementation NCLEX: Physiological Integrity

The nurse is caring for a patient with polycythemia vera. What is an important action for the nurse to initiate? Encourage deep breathing and coughing. Assist with or perform phlebotomy at the bedside. Teach the patient how to maintain a low-activity lifestyle. Perform thorough and regularly scheduled neurologic assessments.

Assist with or perform phlebotomy at the bedside.

The nurse prepares to administer IV ibandronate (Boniva) to a 67-year-old woman with osteoporosis. What is a priority laboratory assessment to make before the administration of ibandronate? A. Serum calcium B. Serum creatinine C. Serum phosphate D. Serum alkaline phosphatase

B. Serum creatinine. Ibandronate is a bisphosphonate that is administered IV every 3 months and is administered slowly over 15 to 30 seconds to prevent renal damage. Ibandronate should not be used by patients taking other nephrotoxic drugs or by those with severe renal impairment (defined as serum creatinine above 2.3 mg/dL or creatinine clearance less than 30 mL/min).

The nurse studying osteoporosis learns that which drugs can cause this disorder? REMEMBER SECONDARY OSTEOPOROSIS CAN BE CAUSED BY MEDICATIONS (Select all that apply.) a. Antianxiety agents b. Antibiotics c. Barbiturates d. Corticosteroids e. Loop diuretics

C, D, E Several classes of drugs can cause secondary osteoporosis, including barbiturates, corticosteroids, and loop diuretics. Antianxiety agents and antibiotics are not associated with the formation of osteoporosis.

The nurse prepares to administer IV ibandronate (Boniva) to a 67-year-old woman with osteoporosis. What is a priority laboratory assessment to make before the administration of ibandronate? A. Serum calcium B. Serum creatinine C. Serum phosphate D. Serum alkaline phosphatase

C. Serum phosphate Ibandronate is a bisphosphonate that is administered IV every 3 months and is administered slowly over 15 to 30 seconds to prevent renal damage. Ibandronate should not be used by patients taking other nephrotoxic drugs or by those with severe renal impairment (defined as serum creatinine above 2.3 mg/dL or creatinine clearance less than 30 mL/min).

Which of the following are routes of administration for Calcitonin? Select all that apply. Nasal spray Subcutaneous Intramuscular injection Intravenous Oral

Calcitonin is administered by nasal spray or by subcutaneous or intramuscular injections.

A patient has had surgery for carpal tunnel syndrome (CTS). What does the nurse teach the patient before discharge?

Check neurovascular status of the fingers every hour.

A patient with leukemia is admitted for severe hypovolemia after prolonged diarrhea has a platelet count of 43,000/µL. It is most important for the nurse to take which action? Insert two 18-gauge IV catheters. Administer prescribed enoxaparin. Monitor the patient's temperature every 2 hours. Check stools for presence of frank or occult blood.

Check stools for presence of frank or occult blood.

A 75-year-old client had surgery for a left hip fracture yesterday. When completing the plan of care, the nurse should include assessment for which complications? Select all that apply. Pneumonia Necrosis of the humerus Skin breakdown Sepsis Delirium

Complications in clients with hip fractures are often related to the client's age. During the first 24 to 48 hours following surgery for hip fracture, atelectasis or pneumonia can develop as a result of the anesthesia. Thromboemboli are possible, as is sepsis. Elderly clients are also at risk for delirium in hospital settings because of the stress of the trauma, unfamiliar surroundings, sleep deprivation, and medications. An elderly client with decreased mobility is at risk for skin breakdown. Necrosis is a potential complication of the surgery, but the complication would be with the femur, not the humerus.

The nurse is changing a patient's stump dressing. How would the nurse document this dressing technique? 1. Figure-of-eight bandage 2. Binder wrapping 3. Splinting 4. Bivalving

Correct Answer: 1 Rationale: Compression wrapping of the extremity helps to prevent edema. Figure-of-eight bandaging starts at the distal stump (after the bandage is anchored around the waist) and is wrapped back toward the waist.

A patient is seen at the urgent care center after falling on the right arm and shoulder. It will be most important for the nurse to determine a. whether there is bruising at the shoulder area. b. whether the right arm is shorter than the left. c. the amount of pain the patient is experiencing. d. how much range of motion (ROM) is present.

Correct Answer: B Rationale: A shorter limb after a fall indicates a possible dislocation, which is an orthopedic emergency. The nurse will expect bruising and pain at the area, even without an injury that requires surgery. The shoulder should be immobilized until it is evaluated by the health care provider.

50. The nurse is assessing an older Caucasian man and notes there are flexion contractures of the fourth and fifth fingers. The patient reports that he had a similar problem on the other hand and had a fasciectomy which improved the function. What is this condition known as? a. Dupuytren's contracture b. Ganglion cyst c. Bunion d. Plantar digital neuritis

Dupuytren's contracture

A client is admitted to the orthopedic unit with a fractured femur after a motorcycle accident. The client has been placed in traction until his femur can be rodded in surgery. For what early complications should the nurse monitor this client? Select all that apply. Systemic infection Complex regional pain syndrome Deep vein thrombosis Compartment syndrome Fat embolism

Early complications include shock, fat embolism, compartment syndrome, and venous thromboemboli (deep vein thrombosis [DVT], pulmonary embolism [PE]). Infection and complex regional pain syndrome are later complications of fractures.

The nurse is completing a physical assessment on a client with possible fibromyalgia. To support the diagnosis of​ fibromyalgia, the nurse must assess and document pain upon palpation on how many standard tender​ points? Eleven Nine Five Seven

Eleven To assist with the diagnosis of​ fibromyalgia, the client must have pain upon palpation at 11 or more of the 18 standard tender​ points, not​ nine, seven, or five of the standard tender points.

A patient has anemia related to inadequate intake of essential nutrients. Which intervention would be appropriate for the nurse to include in the plan of care for this patient? Plan for 30 minutes of rest before and after every meal. Encourage foods high in protein, iron, vitamin C, and folate. Instruct the patient to select soft, bland, and nonacidic foods. Give the patient a list of medications that inhibit iron absorption.

Encourage foods high in protein, iron, vitamin C, and folate.

27. A patient with Paget's disease has a kidney problem associated with increased serum calcium. What is the nursing priority for this patient? a. Encourage the patient to increase fluids, unless contraindicated. b. Encourage moderate consumption of milk and dairy products. c. Assist the patient to problem-solve incontinence issues. d. Direct the UAP to measure and record all urine output.

Encourage the patient to increase fluids, unless contraindicated.

The nurse is caring for an older adult client diagnosed with osteomalacia. The nurse anticipates that the physician will request which medication? A. Ascorbic acid (vitamin C) B. Ergocalciferol (calciferol) C. Phenytoin (Dilantin) D. Prednisone (Deltasone)

Ergocalciferol (calciferol)

1. True or False: The appendix is found on the left lower side of the abdomen and is connected to the cecum of the large intestine.

False. The appendix is found on the RIGHT (not left) lower side of the abdomen and is connected to the cecum of the large intestine.

True or False: Rheumatoid arthritis tends to affect women more than men and people who are over the age of 60.

False: Yes, RA tends to affect women more than men BUT it can affect all ages...most commonly 20-60 years old.

1.Which risk factors are associated with osteoporosis? a. Male over 50 years of age, European heritage b. Female, white, menopausal, thin, lean, immobilized c. Older adult, vitamin D deficiency, insufficient exposure to sunlight d. Weight-bearing exercise, moderate alcohol intake

Female, white, menopausal, thin, lean, immobilized

48. Which type of malignant bone tumor has manifestations of local tenderness in lower extremity long bones? a. Chondrosarcoma b. Ewing's sarcoma c. Fibrosarcoma d. Osteosarcoma

Fibrosarcoma

Which term is used to describe a round, benign cyst, often found on a wrist or foot joint or tendon?

Ganglion Cyst

What are the nursing intervention for acute osteomyelitis?

Immobilization and careful handling of affected limb, assess and treat pain, dressing care, and proper positioning to prevent complications of immobility

28. A patient is having diagnostic testing to determine the probability of Paget's disease. If the disease is present, which laboratory result does the nurse expect to see? a. Slightly decreased serum calcium level b. Increased serum alkaline phosphatase (ALP) c. Decreased pyridinium (PYD) d. Absence of osteocalcin

Increased serum alkaline phosphatase (ALP)

What are the three types of entry for osteomyelitis?

Indirect entry (hematogenous), direct entry, and foreign body presence

36. A patient is diagnosed with Ewing's sarcoma. Which characteristics are specific to this type of bone cancer? (Select all that apply.) a. Low-grade fever is present. b. Leukocytosis is present. c. In 40% of cases, the tumor is found in a distal femur. d. Death usually results from metastasis to lungs and other bones. e. Ribs and upper extremities are most affected.

Low-grade fever is present. Leukocytosis is present. Death usually results from metastasis to lungs and other bones.

A patient who has sickle cell disease has developed cellulitis above the left ankle. What is the nurse's priority for this patient? Start IV fluids. Maintain oxygenation. Maintain distal warmth. Check peripheral pulses.

Maintain oxygenation

54. The nurse is assessing a patient with a spinal deformity. Which technique does the nurse use to accomplish inspection of the spine? a. Observe the patient from the front and back while standing and during forward flexion from the hips. b. Observe the patient in a sitting and standing position and ask the patient to walk around the room. c. Ask the patient to remove the clothes from the waist up and then view the visible curvature of the spine. d. Look at the patient's back while the patient moves in different positions: touching toes, lateral bending, twisting.

Observe the patient from the front and back while standing and during forward flexion from the hips.

What should you teach patients regarding the adverse and toxic reactions to antibiotic therapy?

Ototoxicity, nephrotoxicity, neurotoxicity, (hives, diarrhea, bloody stools, throat & mouth sores and tendon rupture)

3. The nurse assesses a patient with a musculoskeletal disorder and finds hip flexion contractures, flushed and warm skin, and a soft thick and enlarged skull. Which disorder does the nurse recognize? a. Osteoporosis b. Osteomalacia c. Paget's disease d. Osteomyelitis

Paget's disease

A patient is admitted to the same day surgery unit following a meniscectomy. What does post operative care for this patient include?

Perform neurovascular checks every hour for the first few hours and then every 4 hours. Check the surgical dressing for bleeding. Teach about signs and symptoms of infection.

In addition to genetic​ factors, what additional factor is likely to contribute to the development of​ fibromyalgia? Excess physical exercise Physical or emotional trauma Nutritional deficiency Previous injury to the bone

Physical or emotional trauma Clients with fibromyalgia may recall the onset of pain after physical or emotional trauma. Excess​ exercise, previous injury to​ bones, and nutritional deficiencies may cause pain and​ discomfort, but none of these are associated with the development of fibromyalgia.

49. A patient who is a long-distance runner reports severe pain in the arch of the foot, especially when getting out of bed and with weight bearing. What does the nurse suspect in this patient? a. Morton's neuroma b. Plantar fasciitis c. Hammertoe d. Hallux valgus deformity

Plantar fasciitis

25. A patient with Paget's disease has complications related to bony enlargements of the skull. Which complication is potentially the most serious and life-threatening? a. Basilar complications with compression on the cranial nerves b. Platybasia, or basilar invagination with brainstem manifestations c. Blockage of cerebrospinal fluid (CSF), resulting in hydrocephalus d. Pressure from an enlarged temporal bone leading to deafness and vertigo

Platybasia, or basilar invagination with brainstem manifestations

17. Which osteoporosis drug should not be given to women with a history of venous thromboembolism? a. Raloxifene (Evista) b. Ibandronate (Boniva) c. Risedronate (Actonel) d. Alendronate (Fosamax)

Raloxifene (Evista)

The nurse is reviewing the medication administration record of the client. Which of the following medications would lead the nurse to suspect that the client is at risk for osteoporosis? penicillamine (Cuprimine) methotrexate (Rheumatrex) plicamycin (Mithracin) raloxifene (Evista)

Raloxifene (Evista) is used for the prevention and treatment of osteoporosis.

A physician prescribes raloxifene to a hospitalized client. The client's history includes a right hip fracture, hysterectomy, deep vein thrombosis, and hypertension. Which action by the nurse demonstrates safe nursing care? Administering the raloxifene in the evening Holding the raloxifene and notifying the physician Administering the raloxifene with food or milk Having the patient sit upright for 30-60 minutes following administration

Raloxifene is contraindicated in clients with a history of deep vein thrombosis. The nurse should hold the medication and notify the physician. Raloxifene can be given without regard to food or time of day. Raloxifene is a selective estrogen receptor modulation medication. Sitting upright for 30-60 minutes is indicated with drugs classified as bisphosphonates.

22. A patient reports pain in the lower legs and pelvis which is aggravated by activity and worse at night. The nurse observes muscle weakness which appears to be causing a waddling and unsteady gait. What additional information supports the likelihood of osteomalacia in this patient? a. Recent immigration from a country where famine is common b. Taking hormone replacement therapy for a prolonged time c. Unable to perform a prescribed exercise regimen d. History of recent vertebroplasty for osteoporosis

Recent immigration from a country where famine is common

A patient is brought to the emergency department (ED) via ambulance after a motor vehicle crash. What condition does the nurse assess for first?

Respiratory distress

55. The nurse is caring for a patient with muscular dystrophy. Although all body systems can be affected, the nurse is alert and carefully assesses for which major problem? a. Renal failure b. Cardiac failure c. Muscle weakness d. Respiratory failure

Respiratory failure

29. A patient with Paget's disease has been prescribed drug therapy. The nurse prepares patient teaching information for which medication as a first-line therapy? a. Calcitonin b. Ibuprofen (Motrin) c. Plicamycin (Mithracin) d. Risedronate (Actonel)

Risedronate (Actonel)

4. The nurse is conducting an assessment on a patient with osteoporosis. Which factors and/ or patient data may be associated with this disorder? (Select all that apply.) a. Muscle cramps b. Sedentary lifestyle c. Back pain relieved by rest d. Fracture e. Urinary or renal stones

Sedentary lifestyle Back pain relieved by rest Fracture

Name four long term and mostly rare complications

Septicemia, Septic Arthritis, Pathologic Fractures, Anyloidosis

26. A patient with Paget's disease comes to the clinic for evaluation. Which symptom reported by the patient alerts the nurse to the possibility of osteogenic sarcoma? a. Change in hearing b. Warmth and redness of the joints c. Changes in balance and gait d. Severe bone pain

Severe bone pain

24. The nurse is assessing a patient who reports moderate bone pain in the hip and has a family history of Paget's disease. In performing a musculoskeletal assessment, the nurse pays particular attention to which element? a. Size and shape of the skull b. Long-bone bowing in the legs c. Asymmetrical deformity of the extremities d. Loose teeth and difficulty chewing

Size and shape of the skull

51. A patient is diagnosed with plantar fasciitis. What instruction does the nurse give to the patient about self-care for this condition? a. Use rest, elevation, and warm packs. b. Perform gentle jogging exercises. c. Strap the foot to maintain the arch. d. Wear loose or open shoes, such as sandals.

Strap the foot to maintain the arch.

10. The nurse is assessing an older adult patient at risk for osteoporosis. Which task can be delegated to the unlicensed assistive personnel (UAP)? a. Inspect the vertebral column. b. Take height and weight measurements. c. Compare observations to previous findings. d. Ask if the patient is shorter or has gained or lost weight.

Take height and weight measurements.

The nurse is caring for a patient with microcytic, hypochromic anemia. What teaching should the nurse provide that would be beneficial to the patient? Take enteric-coated iron with each meal. Take cobalamin with green leafy vegetables. Take the iron with orange juice one hour before meals. Decrease the intake of the antiseizure medications to improve.

Take the iron with orange juice one hour before meals.

A patient with rheumatoid arthritis is experiencing sudden vision changes. Which medication found in the patient's medication list can cause retinal damage? A. Hydroxychloroquine (Plaquenil) B. Lefluomide (Arava) C. Sulfasalazine (Azulfidine) D. Methylprednisolone (Medrol)

The answer is A. This medication is a DMARD and can cause retinal damage. Therefore, the patient should be monitored for vision changes.

You're providing education to a group of nursing students about the care of a patient with appendicitis. Which statement by a nursing student requires re-education about your teaching? "After an appendectomy the patient may have a nasogastric tube to remove stomach fluids and swallowed air." "Non-pharmacological techniques for a patient with appendicitis include application of heat to the abdomen and the side-lying position." "The nurse should monitor the patient for signs and symptoms of peritonitis which includes increased heart rate, respirations, temperature, abdominal distention, and intense abdominal pain." "It is normal for some patients to have shoulder pain after a laparoscopic appendectomy."

The answer is B. This statement by the nursing student requires re-education because heat should NEVER be applied to abdomen if appendicitis is suspected or known. Heat application can increase the risk of appendix perforation. Ice application is recommended, if warranted. However, the side-lying position can help relieve the patient's pain and is recommended. All the other options are correct.

35. The nurse is teaching a patient about antibiotic therapy for osteomyelitis. What information does the nurse give to the patient? a. Single-agent therapy is the most effective treatment for acute infections. b. Chronic osteomyelitis may require 1 month of antibiotic therapy. c. Patients usually remain hospitalized to complete the full course of antibiotic therapy. d. The infected wound may be irrigated with one or more types of antibiotic solutions.

The infected wound may be irrigated with one or more types of antibiotic solutions.

The nurse is changing the dressing of a chronic wound. There is no sign of infection or heavy drainage. How long will the nurse leave the wound covered for? 6 to 12 hours 12 to 24 hours 24 to 36 hours 48 to 72 hours

The natural wound-healing process should not be disrupted. Unless the wound is infected or has a heavy discharge, it is common to leave chronic wounds covered for 48 to 72 hours and acute wounds for 24 hours.

19. Why do men develop osteoporosis after the age of 50? a. Their testosterone levels decrease. b. Older men are prescribed more medications. c. It is secondary to hyperparathyroidism. d. As men age they are less active.

Their testosterone levels decrease.

The nurse is caring for a patient with a diagnosis of disseminated intravascular coagulation (DIC). What is the first priority of care? Administer heparin. Administer whole blood. Treat the causative problem. Administer fresh frozen plasma.

Treat the causative problem.

A patient is being treated for non-Hodgkin's lymphoma (NHL). What should the nurse first teach the patient about the treatment? Skin care that will be needed Method of obtaining the treatment Gastrointestinal tract effects of treatment Treatment type and expected side effects

Treatment type and expected side effects

15. A patient has been advised by the health care provider that exercising may help prevent osteoporosis. Which exercise does the nurse recommend to the patient? a. Swimming 10 to 15 laps 3 to 5 times a week b. Running for 20 minutes 4 times a week c. Bowling for 60 minutes 3 times a week d. Walking for 30 minutes 3 to 5 times a week

Walking for 30 minutes 3 to 5 times a week

Where can antibiotic treatment be given for osteomyelitis?

at home or in skilled nursing facility

During a routine physical examination on a 75-year-old female client, a nurse notes that the client is 5 feet, 3/8 inches (1.6 m) tall. The client states, "How is that possible? I was always 5 feet and 1/2? (1.7 m) tall." Which statement is the best response by the nurse? a) "After age 40, height may show a gradual decrease as a result of spinal compression" b) "After menopause, the body's bone density declines, resulting in a gradual loss of height." c) "The posture begins to stoop after middle age." d) "There may be some slight discrepancy between the measuring tools used."

b) "After menopause, the body's bone density declines, resulting in a gradual loss of height." Explanation: The nurse should tell the client that after menopause, the loss of estrogen leads to a loss in bone density, resulting in a loss of height. This client's history doesn't indicate spinal compression. Telling the client that measuring tools used to obtain the client's height may have a discrepancy or that the posture begins to stoop after middle age doesn't address the client's question. pg.1143

A patient has been diagnosed with osteosarcoma of the humerus. He shows an understanding of his treatment options when he states a."I accept that I have to lose my arm with surgery." b."The chemotherapy before surgery will shrink the tumor." c."This tumor is related to the melanoma I had 3 years ago." d."I'm glad they can take out the cancer with such a small scar."

b."The chemotherapy before surgery will shrink the tumor." A patient with osteosarcoma usually has preoperative chemotherapy to decrease tumor size before surgery. As a result, limb-salvage procedures, including a wide surgical resection of the tumor, are being used more often. Osteosarcoma is a primary bone tumor that is extremely aggressive and rapidly metastasizes to distant sites.

What are local manifestations of osteomyelitis?

pain unrelieved by rest that worsens with activity, swelling, tenderness, warmth, restricted movement

An 80 - year - old Asian woman presents to your office with back pain. She notes that the pain began on the previous day after stepping off a curb. She has been unable to sleep due to the pain. Physical examination reveals she has a dorsal kyphosis and tenderness over a midthoracic spinous process. Results of neurologic examination are negative for signs of cord compression. You prescribe a mild narcotic pain reliever and order thoracic spine films. Which of the following medications would be of most immediate benefit to this patient? (A) Calcium supplements (B) Residronate (C) Calcitonin (D) Estrogen (E) Alendronate

(C) Calcitonin. Sudden onset of severe back pain following minimal or no trauma suggests an osteo- porotic vertebral fracture. The presence of a dorsal kyphosis also points to osteoporosis. Treatment with narcotic analgesia is appropriate for a presumed ver- tebral fracture. Plain radiographs may or may not reveal the acute fracture but are useful to rule out other bone pathology, such as metastatic disease. This patient should have calcium supplementation, but other medication is indicated for treatment of osteoporosis. Estrogen, alendronate, and risedronate are FDA- approved for the prevention and treatment of osteoporosis but do not provide immediate relief. However, the FDA recommends nonestrogen treatments be considered first for osteoporosis. In addition to reducing vertebral fractures, calcitonin has been shown to produce significant alleviation of the acute pain from osteoporotic vertebral fractures.

A 65 - year - old white woman presents for interpretation of BMD testing. Her personal medical history is unremarkable, but her mother died at age 75 years due to complications of a hip fracture. Her T score is - 2. Which of the following choices is the correct interpretation of this patient's T score and treatment? (A) Normal BMD; calcium supplementation (B) Normal BMD; alendronate for prevention (C) Osteopenia; zinc supplementation for preven- tion (D) Osteopenia; alendronate for treatment (E) Osteoporosis; residronate for treatment

(D) Osteopenia; alendronate for treatment. The T score compares the patient's BMD in standard deviations with the average BMD of a young adult of the same gender. T score between 0 and - 1 indi- cates normal bone density. A T score between - 1 and - 2.5 are consistent with osteopenia. T score 2.5 standard deviations or more below the standard (T score ≤- 2.5) indicates osteoporosis. This patientis at increased risk for osteoporosis owing to her race and family history. Her BMD testing results indicate that she has osteopenia.

All of the following are risk factors for osteoporo-sis EXCEPT: (A) Low calcium intake (B) Smoking (C) Alcohol use (D) Turner's syndrome (E) Obesity

(E) Obesity. Risk factors for osteoporosis may be inherited or acquired. 1,2 Inherited factors include female sex and white or Asian ancestry. Women with Turner's syndrome or any condition leading to low estrogen levels are at increased risk of osteoporosis. Women with a personal history of fracture as an adult or history of osteoporosis in a first-degree rela- tive also are at increased risk. Acquired risk factors include current smoking ( ≥ 1 pack per day), alco- hol use ( ≥ 2 drinks per day), low calcium intake (< 1200 mg per day), and sedentary lifestyle. Finally, women with low body weight (< 127 lb) are at in- creased risk for osteoporosis. Guidelines for osteo- porosis screening and prevention are available only for women at this time.

What is the major concern related to pelvic injury? 1 Loss of blood volume 2 Avascular necrosis (AVN) 3 Risk for developing pneumonia 4 Risk for neurologic dysfunction

1. Loss of blood volume Pelvic injury most commonly results in venous oozing or arterial bleeding. Loss of blood volume due to pelvic injury leads to hypovolemic shock. AVN may cause death and necrosis of bone tissue, resulting in pain and decreased mobility. This problem is most likely seen in patients with displaced fractures of the femur. Chest trauma might increase the risk for developing pneumonia. Neurologic dysfunction can be seen when trabecular, or cancellous, bone within the vertebra becomes weakened and causes the vertebral body to collapse, resulting in compression fractures.

What are three common activity level recommendations for postop appendectomy pts?

1. Resume normal activity in 2-4 weeks 2. No heavy lifting for 6 weeks 3. No driving for 2 weeks (medication use, seatbelt pressure on abdomen)

12. Priority Decision: Before repositioning the patient on the side after a lumbar laminectomy, what should be the nurse's first action? a. Raise the head of the bed 30 degrees. b. Have the patient flex the knees and hips. c. Place a pillow between the patient's legs. d. Have the patient grasp the side rail on the opposite side of the bed.

12. c. After spinal surgery, patients are logrolled to maintain straight alignment of the spine at all times, requiring the patient to be turned with a pillow between the legs and moving the body as a unit. The head of the bed is usually kept flat and the legs are extended

The nurse notes a physician's order written at 10:00 AM for two units of packed red blood cells to be administered to a patient who is anemic as a result of chronic blood loss. If the transfusion is picked up at 11:30 AM, the nurse should plan to hang the unit no later than what time? 11:45 AM 12:00 noon 12:30 PM 3:30 PM

12:00 noon

Before starting a transfusion of packed red blood cells for an older anemic patient, the nurse would arrange for a peer to monitor his or her other assigned patients for how many minutes when the nurse begins the transfusion? 5 15 30 60

15

16. What are characteristics of Paget's disease (select all that apply)? a. Results from vitamin D deficiency b. Loss of total bone mass and substance c. Abnormal remodeling and resorption of bone d. Most common in bones of spine, hips, and wrists e. Generalized bone decalcification with bone deformity f. Replacement of normal marrow with vascular connective tissue

16. c, f. Paget's disease is abnormal remodeling and resorption of bone with replacement of normal marrow with vascular connective tissue. Osteoporosis is loss of total bone mass and substance with abnormal remodeling and resorption of bone and is most common in bones of the spine, hips, and wrists. Osteomalacia results from vitamin D deficiency and causes generalized bone decalcification with bone deformity.

How long is the minimum course of antibiotics for a patient diagnosed with acute osteomyelitis?

4-6 weeks minimum

4. Following 2 weeks of IV antibiotic therapy, a patient with acute osteomyelitis of the tibia is prepared for discharge from the hospital. The nurse determines that additional instruction is needed when the patient makes which statement? a. "I will need to continue antibiotic therapy for 4 to 6 weeks." b. "I shouldn't bear weight on my affected leg until healing is complete." c. "I can use a heating pad on my lower leg for comfort and to promote healing." d. "I should notify the health care provider if the pain in my leg becomes worse."

4. c. Activities such as exercise or heat application, which increase circulation and serve as stimuli for the spread of infection, should be avoided by patients with acute osteomyelitis. Oral or IV antibiotic therapy is continued at home for 4 to 6 weeks, weight bearing is contraindicated to prevent pathologic fractures, and notification of the health care provider if increased pain occurs is necessary.

A Notify the health care provider immediately of this change in patient's condition, which suggest development of compartment syndrome. Pain unrelieved by drugs and out of proportion to the level of injury is one of the first indications of impending compartment syndrome. Changes in sensation (tingling) also suggest compartment syndrome. Because elevation of the extremity may lower venous pressure and slow arterial perfusion, the extremity should not be elevated above heart level. Similarly, the application of cold compresses may result in vasoconstriction and exacerbate compartment syndrome. Administration of morphine may be warranted, but it is not the first priority.

A 28-yr-old woman with a fracture of the proximal left tibia in a long leg cast and complains of severe pain and a prickling sensation in the left foot. The toes on the left foot are pale and cool. Which nursing action is a priority? Notify the health care provider immediately. Elevate the left leg above the level of the heart. Administer prescribed morphine sulfate intravenously. Apply ice packs to the left proximal tibia over the cast.

C To prevent hip flexion contractures, the patient should lie on the abdomen for 30 minutes three or four times each day and position the hip in extension while prone. The patient should avoid sitting in a chair for more than 1 hour with hips flexed or having pillows under the surgical extremity. The patient should avoid dangling the residual limb over the bedside to minimize edema.

A 42-yr-old man underwent amputation below the knee on the left leg after a recent heavy farm machinery accident. Which intervention should the nurse include in the plan of care? Sit in a chair for 1 to 2 hours three times each day. Dangle the residual limb for 20 to 30 minutes every 6 hours. Lie prone with hip extended for 30 minutes four times per day. Elevate the residual limb on a pillow for 4 to 5 days after surgery.

A nurse is providing an educational class to a group of older adults at a community senior center. In an effort to prevent osteoporosis, the nurse should encourage participants to ensure that they consume the recommended intake of what nutrients? Select all that apply. Vitamin B12 Potassium Calcitonin Calcium Vitamin D

A diet rich in calcium and vitamin D protects against skeletal demineralization. Intake of vitamin B12 and potassium does not directly influence the risk for osteoporosis. Calcitonin is not considered to be a dietary nutrient.

In which order will the nurse implement these interprofessional interventions prescribed for a patient admitted with acute osteomyelitis with a temperature of 101.2° F? (Put a comma and a space between each answer choice [A, B, C, D].) a. Obtain blood cultures from two sites. b. Administer dose of gentamicin 60 mg IV. c. Send to radiology for computed tomography (CT) scan of right leg. d. Administer acetaminophen (Tylenol) now and every 4 hours PRN for fever.

ANS: A, B, D, C The highest treatment priority for possible osteomyelitis is initiation of antibiotic therapy, but cultures should be obtained before administration of antibiotics. Addressing the discomfort of the fever is the next highest priority. Because the purpose of the CT scan is to determine the extent of the infection, it can be done last. DIF: Cognitive Level: Analyze (analysis) REF: 1497 OBJ: Special Questions: Prioritization TOP: Nursing Process: Planning MSC: NCLEX: Physiological Integrity

A patient with osteomyelitis is to receive vancomycin (Vancocin) 500 mg IV every 6 hours. The vancomycin is diluted in 100 mL of normal saline and needs to be administered over 1 hour. The nurse will set the IV pump for how many mL/minute? (Round to the nearest hundredth.)

ANS: 1.67 To administer 100 mL in 60 minutes, the IV pump will need to provide 1.67 mL/minute.

Anakinra (Kineret) is prescribed for a patient who has rheumatoid arthritis (RA). When teaching the patient about this drug, the nurse will include information about a. self-administration of subcutaneous injections. b. taking the medication with at least 8 oz of fluid. c. avoiding concurrently taking aspirin or nonsteroidal antiinflammatory drugs (NSAIDs). d. symptoms of gastrointestinal (GI) irritation or bleeding.

ANS: A Anakinra is administered by subcutaneous injection. GI bleeding is not a side effect of this medication. Because the medication is injected, instructions to take it with 8 oz of fluid would not be appropriate. The patient is likely to be concurrently taking aspirin or NSAIDs, and these should not be discontinued.

After teaching a patient diagnosed with progressive systemic sclerosis about health maintenance activities, the nurse determines that additional instruction is needed when the patient says, a. "I should lie down for an hour after meals." b. "Paraffin baths can be used to help my hands." c. "Lotions will help if I rub them in for a long time." d. "I should perform range-of-motion exercises daily."

ANS: A Because of the esophageal scarring, patients should sit up for 2 hours after eating. The other patient statements are correct and indicate that the teaching has been effective.

The nurse evaluating effectiveness of prescribed calcitonin (Cibacalcin) and ibandronate (Boniva) for a patient with Paget's disease will consider the patient's a. pain level. b. oral intake. c. daily weight. d. grip strength.

ANS: A Bone pain is one of the common early manifestations of Paget's disease, and the nurse should assess the pain level to determine whether the treatment is effective. The other information will also be collected by the nurse, but will not be used in evaluating the effectiveness of the therapy

A patient with an acute attack of gout is treated with colchicine. The nurse determines that the drug is effective upon finding a. relief of joint pain. b. increased urine output. c. elevated serum uric acid. d. decreased white blood cells (WBC).

ANS: A Colchicine produces pain relief in 24 to 48 hours by decreasing inflammation. The recommended increase in fluid intake of 2 to 3 L/day would increase urine output but would not indicate the effectiveness of colchicine. Elevated uric acid levels would result in increased symptoms. The WBC count might decrease with decreased inflammation, but this would not be as useful in determining the effectiveness of colchicine as a decrease in pain.

A patient with acute osteomyelitis of the left femur is hospitalized for regional antibiotic irrigation. Which intervention will be included in the initial plan of care? a. Immobilization of the left leg b. Positioning the left leg in flexion c. Assisted weight-bearing ambulation d. Quadriceps-setting exercise repetitions

ANS: A Immobilization of the affected leg helps decrease pain and reduce the risk for pathologic fractures. Weight-bearing exercise increases the risk for pathologic fractures. Flexion of the affected limb is avoided to prevent contractures

Which action will the nurse take when caring for a patient with osteomalacia? a. Teach about the use of vitamin D supplements. b. Educate about the need for weight-bearing exercise. c. Discuss the use of medications such as bisphosphonates. d. Emphasize the importance of sunscreen use when outside.

ANS: A Osteomalacia is caused by inadequate intake or absorption of vitamin D. Weight-bearing exercise and bisphosphonate administration may be used for osteoporosis but will not be beneficial for osteomalacia. Because ultraviolet light is needed for the body to synthesize vitamin D, the patient might be taught that 20 minutes/day of sun exposure is beneficial

Which statement by a 24-year-old woman with systemic lupus erythematosus (SLE) indicates that the patient has understood the nurse's teaching about management of the condition? a. "I will use a sunscreen whenever I am outside." b. "I will try to keep exercising even if I am tired." c. "I should take birth control pills to keep from getting pregnant." d. "I should not take aspirin or nonsteroidal anti-inflammatory drugs."

ANS: A Severe skin reactions can occur in patients with SLE who are exposed to the sum. Patients should avoid fatigue by balancing exercise with rest periods as needed. Oral contraceptives can exacerbate lupus. Aspirin and nonsteroidal anti-inflammatory drugs are used to treat the musculoskeletal manifestations of SLE.

When helping a patient with rheumatoid arthritis (RA) plan a daily routine, the nurse informs the patient that it is most helpful to start the day with a. a warm bath followed by a short rest. b. a short routine of isometric exercises. c. active range-of-motion (ROM) exercises. d. stretching exercises to relieve joint stiffness.

ANS: A Taking a warm shower or bath is recommended to relieve joint stiffness, which is worse in the morning. Isometric exercises would place stress on joints and would not be recommended. Stretching and ROM should be done later in the day, when joint stiffness is decreased.

An assessment finding that alerts the nurse to the presence of osteoporosis in a middle-aged patient is a. measurable loss of height. b. the presence of bowed legs. c. an aversion to dairy products. d. statements about frequent falls.

ANS: A Osteoporosis occurring in the vertebrae produces a gradual loss of height. Bowed legs are associated with osteomalacia. Low intake of dairy products is a risk factor for osteoporosis, but it does not indicate that osteoporosis is present. Frequent falls increase the risk for fractures but are not an indicator of osteoporosis.

A 20-year-old patient with a history of muscular dystrophy is hospitalized with a respiratory tract infection. Which nursing action will be included in the plan of care? a. Assist the patient with ambulation. b. Logroll the patient every 1 to 2 hours. c. Discuss the need for genetic testing with the patient. d. Teach the patient about the muscle biopsy procedure.

ANS: A Since the goal for the patient with muscular dystrophy is to keep the patient active for as long as possible, assisting the patient to ambulate will be part of the care plan. The patient will not require logrolling. Muscle biopsies are necessary to confirm the diagnosis but are not necessary for a patient who already has a diagnosis. There is no need for genetic testing since the patient already knows the diagnosis

Which actions will the nurse include in the plan of care when caring for a patient with metastatic bone cancer of the left femur (select all that apply)? a. Monitor serum calcium level. b. Teach about the need for strict bed rest. c. Avoid use of sustained-release opioids for pain. d. Support the left leg when repositioning the patient. e. Support family as they discuss the prognosis of patient

ANS: A, D, E The nurse will monitor for hypercalcemia caused by bone decalcification. Support of the leg helps reduce the risk for pathologic fractures. Although the patient may be reluctant to exercise, activity is important to maintain function and avoid the complications associated with immobility. Adequate pain medication, including sustained-release and rapidly acting opioids, is needed for the severe pain that is frequently associated with bone cancer. The prognosis for metastatic bone cancer is poor so the patient and family need to be supported as they deal with the reality of the situation.

A nurse who works on the orthopedic unit has just received change-of-shift report. Which patient should the nurse assess first? a. Patient who reports foot pain after hammertoe surgery b. Patient who has not voided 10 hours after a laminectomy c. Patient with low back pain and a positive straight-leg-raise test d. Patient with osteomyelitis who has a temperature of 100.5° F (38.1° C)

ANS: B Difficulty in voiding may indicate damage to the spinal nerves and should be assessed and reported to the surgeon immediately. The information about the other patients is consistent with their diagnoses. The nurse will need to assess them as quickly as possible, but the information about them does not indicate a need for immediate intervention. DIF: Cognitive Level: Analyze (analysis) REF: 1507 OBJ: Special Questions: Prioritization | Special Questions: Multiple Patients TOP: Nursing Process: Planning MSC: NCLEX: Safe and Effective Care Environment

A patient whose employment requires frequent lifting has a history of chronic back pain. After the nurse has taught the patient about correct body mechanics, which patient statement indicates the teaching has been effective? a. "I will keep my back straight when I lift above than my waist." b. "I will begin doing exercises to strengthen and support my back." c. "I will tell my boss I need a job where I can stay seated at a desk." d. "I can sleep with my hips and knees extended to prevent back strain."

ANS: B Exercises can help strengthen the muscles that support the back. Flexion of the hips and knees places less strain on the back than keeping these joints extended. Sitting for prolonged periods can aggravate back pain. Modifications in the way the patient lifts boxes are needed, but the patient should not lift above the level of the elbows. DIF: Cognitive Level: Apply (application) REF: 1504 TOP: Nursing Process: Evaluation MSC: NCLEX: Health Promotion and Maintenance

Which action should the nurse take before administering gentamicin (Garamycin) to a patient who has acute osteomyelitis? a. Ask the patient about any nausea. b. Review the patient's creatinine level. c. Obtain the patient's oral temperature. d. Change the prescribed wet-to-dry dressing.

ANS: B Gentamicin is nephrotoxic and can cause renal failure. Monitoring the patient's temperature before gentamicin administration is not necessary. Nausea is not a common side effect of IV gentamicin. There is no need to change the dressing before gentamicin administration

After the nurse has finished teaching a patient with osteoarthritis (OA) of the left hip and knee about how to manage the OA, which patient statement indicates a need for more education? a. "I can take glucosamine to help decrease my knee pain." b. "I will take 1 g of acetaminophen (Tylenol) every 4 hours." c. "I will take a shower in the morning to help relieve stiffness." d. "I can use a cane to decrease the pressure and pain in my hip."

ANS: B No more than 4 g of acetaminophen should be taken daily to avoid liver damage. The other patient statements are correct and indicate good understanding of OA management.

The nurse instructs a patient who has osteosarcoma of the tibia about a scheduled above-the-knee amputation. Which statement by a patient indicates that additional patient teaching is needed? a. "I will need to participate in physical therapy after surgery." b. "I did not have this bone cancer until my leg broke a week ago." c. "I wish that I did not have to have chemotherapy after this surgery." d. "I can use the patient-controlled analgesia (PCA) to control postoperative pain."

ANS: B Osteogenic sarcoma may be diagnosed following a fracture, but it is not caused by the injury. The other patient statements indicate that patient teaching has been effective

When teaching a patient who has rheumatoid arthritis (RA) about how to manage activities of daily living, the nurse instructs the patient to a. stand rather than sit when performing household chores. b. avoid activities that require continuous use of the same muscles. c. strengthen small hand muscles by wringing sponges or washcloths. d. protect the knee joints by sleeping with a small pillow under the knees.

ANS: B Patients are advised to avoid repetitious movements. Sitting during household chores is recommended to decrease stress on joints. Wringing water out of sponges would increase the joint stress. Patients are encouraged to position joints in the extended position, and sleeping with a pillow behind the knees would decrease the ability of the knee to extend and also decrease knee range of motion (ROM).

The nurse obtains this information when assessing a patient who is taking hydroxychloroquine (Plaquenil) to treat rheumatoid arthritis. Which symptom is most important to report to the health care provider? a. Abdominal cramping b. Complaint of blurry vision c. Phalangeal joint tenderness d. Blood pressure 170/84 mm Hg

ANS: B Plaquenil can cause retinopathy; the medication should be stopped. The other findings are not related to the medication, although they also will be reported.

A 35-year-old patient with three school-age children who has recently been diagnosed with rheumatoid arthritis (RA) tells the nurse that the inability to be involved in many family activities is causing stress at home. Which response by the nurse is most appropriate? a. "You may need to see a family therapist for some help." b. "Tell me more about the situations that are causing stress." c. "Perhaps it would be helpful for you and your family to get involved in a support group." d. "Your family may need some help to understand the impact of your rheumatoid arthritis."

ANS: B The initial action by the nurse should be further assessment. The other three responses might be appropriate based on the information the nurse obtains with further assessment.

Which action will the nurse take first when a patient is seen in the outpatient clinic with neck pain? a. Provide information about therapeutic neck exercises. b. Ask about numbness or tingling of the hands and arms. c. Suggest the patient alternate the use of heat and cold to the neck. d. Teach about the use of nonsteroidal antiinflammatory drugs (NSAIDs).

ANS: B The nurse's initial action should be further assessment of related symptoms because cervical nerve root compression will require different treatment than musculoskeletal neck pain. The other actions may also be appropriate, depending on the assessment findings. DIF: Cognitive Level: Analyze (analysis) REF: 1507 OBJ: Special Questions: Prioritization TOP: Nursing Process: Assessment MSC: NCLEX: Physiological Integrity

A patient is being discharged after 1 week of IV antibiotic therapy for acute osteomyelitis in the right leg. Which information will be included in the discharge teaching? a. How to apply warm packs to the leg to reduce pain b. How to monitor and care for a long-term IV catheter c. The need for daily aerobic exercise to help maintain muscle strength d. The reason for taking oral antibiotics for 7 to 10 days after discharge

ANS: B The patient will be taking IV antibiotics for several months. The patient will need to recognize signs of infection at the IV site and know how to care for the catheter during daily activities such as bathing. IV antibiotics rather than oral antibiotics are used for acute osteomyelitis. Patients are instructed to avoid exercise and heat application because these will increase swelling and the risk for spreading infection. DIF: Cognitive Level: Apply (application) REF: 1499 TOP: Nursing Process: Implementation MSC: NCLEX: Physiological Integrity

When caring for a patient who has osteoarthritis, the nurse will anticipate the need to teach the patient about which of these medications? a. Adalimumab (Humira) b. Prednisone (Deltasone) c. Capsaicin cream (Zostrix) d. Sulfasalazine (Azulfidine)

ANS: C Capsaicin cream blocks the transmission of pain impulses and is helpful for some patients in treating OA. The other medications would be used for patients with RA.

Which action should the nurse take before administering gentamicin (Garamycin) to a patient with acute osteomyelitis? a. Ask the patient about any nausea. b. Obtain the patient's oral temperature. c. Review the patient's serum creatinine. d. Change the prescribed wet-to-dry dressing.

ANS: C Gentamicin is nephrotoxic and can cause renal failure as reflected in the patient's serum creatinine. Monitoring the patient's temperature before gentamicin administration is not necessary. Nausea is not a common side effect of IV gentamicin. There is no need to change the dressing before gentamicin administration. DIF: Cognitive Level: Apply (application) REF: 1498 TOP: Nursing Process: Assessment MSC: NCLEX: Physiological Integrity

Which nursing action included in the care of a patient after laminectomy can the nurse delegate to experienced unlicensed assistive personnel (UAP)? a. Check ability to plantar and dorsiflex the foot. b. Determine the patient's readiness to ambulate. c. Log roll the patient from side to side every 2 hours. d. Ask about pain control with the patient-controlled analgesia (PCA).

ANS: C Repositioning a patient is included in the education and scope of practice of UAP, and experienced UAP will be familiar with how to maintain alignment in the postoperative patient. Evaluation of the effectiveness of pain medications, assessment of neurologic function, and evaluation of a patient's readiness to ambulate after surgery require higher level nursing education and scope of practice

An appropriate nursing intervention for a patient who has acute low back pain and muscle spasms is to teach the patient to a. keep both feet flat on the floor when prolonged standing is required. b. twist gently from side to side to maintain range of motion in the spine. c. keep the head elevated slightly and flex the knees when resting in bed. d. avoid the use of cold packs because they will exacerbate the muscle spasms.

ANS: C Resting with the head elevated and knees flexed will reduce the strain on the back and decrease muscle spasms. Twisting from side to side will increase tension on the lumbar area. Prolonged standing will cause strain on the lumbar spine, even with both feet flat on the floor. Alternate application of cold and heat should be used to decrease pain. DIF: Cognitive Level: Apply (application) REF: 1503 TOP: Nursing Process: Planning MSC: NCLEX: Physiological Integrity

An appropriate nursing intervention for a patient who has acute low back pain and muscle spasms is to teach the patient to a. keep both feet flat on the floor when prolonged standing is required. b. twist gently from side to side to maintain range of motion in the spine. c. keep the head elevated slightly and flex the knees when resting in bed. d. avoid the use of cold packs because they will exacerbate the muscle spasms.

ANS: C Resting with the head elevated and knees flexed will reduce the strain on the back and decrease muscle spasms. Twisting from side to side will increase tension on the lumbar area. A pillow placed under the upper back will cause strain on the lumbar spine. Alternate application of cold and heat should be used to decrease pain

A patient who has rheumatoid arthritis is seen in the outpatient clinic and the nurse notes that rheumatoid nodules are present on the patient's elbows. Which action will the nurse take? a. Draw blood for rheumatoid factor analysis. b. Teach the patient about injection of the nodule. c. Assess the nodules for skin breakdown or infection. d. Discuss the need for surgical removal of the nodule.

ANS: C Rheumatoid nodules can break down or become infected. They are not associated with changes in rheumatoid factor and injection is not needed. Rheumatoid nodules are usually not removed surgically because of a high probability of recurrence

While working at a summer camp, the nurse notices a circular lesion with a red border and clear center on the arm of a patient who is in the camp clinic complaining of chills and muscle aches. Which action should the nurse take next? a. Palpate the abdomen. b. Auscultate the heart sounds. c. Ask the patient about recent outdoor activities. d. Question the patient about immunization history.

ANS: C The patient's clinical manifestations suggest possible Lyme disease. A history of recent outdoor activities such as hikes will help confirm the diagnosis. The patient's symptoms do not suggest cardiac or abdominal problems or lack of immunization.

A patient has muscle spasms and acute low back pain. An appropriate nursing intervention for this problem is to teach the patient to a. avoid the use of cold because it will exacerbate the muscle spasms. b. keep both feet flat on the floor when prolonged standing is required. c. keep the head elevated slightly and flex the knees when resting in bed. d. twist gently from side to side to maintain range of motion in the spine.

ANS: C Resting with the head elevated and knees flexed will reduce the strain on the back and decrease muscle spasms. Twisting from side to side will increase tension on the lumbar area. A pillow placed under the upper back will cause strain on the lumbar spine. Alternate application of cold and heat should be used to decrease pain.

A patient with a herniated intravertebral disk undergoes a laminectomy and discectomy. Following the surgery, the nurse should position the patient on the side by a. instructing the patient to move the legs before turning the rest of the body. b. having the patient turn by grasping the side rails and pulling the shoulders over. c. placing a pillow between the patient's legs and turning the entire body as a unit. d. turning the patient's head and shoulders first, followed by the hips, legs, and feet.

ANS: C The spine should be kept in correct alignment after laminectomy. The other positions will create misalignment of the spine.

A patient with an acute attack of gout in the left great toe has a new prescription for probenecid (Benemid). Which information about the patient's home routine indicates a need for teaching regarding gout management? a. The patient sleeps about 8 to 10 hours every night. b. The patient usually eats beef once or twice a week. c. The patient generally drinks about 3 quarts of juice and water daily. d. The patient takes one aspirin a day prophylactically to prevent angina.

ANS: D Aspirin interferes with the effectiveness of probenecid and should not be taken when the patient is taking probenecid. The patient's sleep pattern will not affect gout management. Drinking 3 quarts of water and eating beef only once or twice a week are appropriate for the patient with gout.

The nurse sees several clients with osteoporosis. For which client would Fosamax not be a good option? a. Client with diabetes who has a serum creatinine of 0.8 mg/dL b. Client who recently fell and has vertebral compression fractures c. Hypertensive client who takes calcium channel blockers d. Client with a spinal cord injury who cannot tolerate sitting up

ANS: D Clients on bisphosphonates must be able to sit upright for 30 to 60 minutes after taking them. The client who cannot tolerate sitting up is not a good candidate for this class of drug. Poor renal function also makes clients bad candidates for this drug, but the client with a creatinine of 0.8 mg/dL is within normal range. Diabetes and hypertension are not related unless the client also has renal disease. The client who recently fell and sustained fractures is a good candidate for this drug if the fractures are related to osteoporosis.

A 54-year-old woman who recently reached menopause and has a family history of osteoporosis is diagnosed with osteopenia following densitometry testing. In teaching the woman about her osteoporosis, the nurse explains that a. estrogen replacement therapy must be started to prevent rapid progression to osteoporosis. b. continuous, low-dose corticosteroid treatment is effective in stopping the course of osteoporosis. c. with a family history of osteoporosis, there is no way to prevent or slow gradual bone resorption. d. calcium loss from bones can be slowed by increasing calcium intake and weight-bearing exercise.

ANS: D Progression of osteoporosis can be slowed by increasing calcium intake and weight-bearing exercise. Estrogen replacement therapy does help prevent osteoporosis, but it is not the only treatment and is not appropriate for some patients. Corticosteroid therapy increases the risk for osteoporosis

Which menu choice by a patient with osteoporosis indicates that the nurse's teaching about appropriate diet has been effective? a. Pancakes with syrup and bacon b. Whole wheat toast and fresh fruit c. Egg-white omelet and a half grapefruit d. Oatmeal with skim milk and fruit yogurt

ANS: D Skim milk and yogurt are high in calcium. The other choices do not contain any high-calcium foods

The nurse will determine more teaching is needed if a patient with discomfort from a bunion says, "I will a. give away my high-heeled shoes." b. take ibuprofen (Motrin) if I need it." c. use the bunion pad to cushion the area." d. only wear sandals, no closed-toe shoes."

ANS: D The patient can wear shoes that have a wide forefoot (toe box). The other patient statements indicate the teaching has been effective. DIF: Cognitive Level: Apply (application) REF: 1509 TOP: Nursing Process: Evaluation MSC: NCLEX: Physiological Integrity

A patient with systemic lupus erythematosus (SLE) who has a facial rash and alopecia tells the nurse, "I hate the way I look! I never go anywhere except here to the health clinic." An appropriate nursing diagnosis for the patient is a. activity intolerance related to fatigue and inactivity. b. impaired social interaction related to lack of social skills. c. impaired skin integrity related to itching and skin sloughing. d. social isolation related to embarrassment about the effects of SLE.

ANS: D The patient's statement about not going anywhere because of hating the way he or she looks supports the diagnosis of social isolation because of embarrassment about the effects of the SLE. Activity intolerance is a possible problem for patients with SLE, but the information about this patient does not support this as a diagnosis. The rash with SLE is nonpruritic. There is no evidence of lack of social skills for this patient.

Following a laminectomy with a spinal fusion, a patient reports numbness and tingling of the right lower leg. The first action indicated by the nurse is to a. report the patient's complaint to the surgeon. b. check the vital signs for indications of hemorrhage. c. turn the patient to the side to relieve pressure on the right leg. d. check the chart for preoperative neuromuscular assessment data.

ANS: D The postoperative movement and sensation of the extremities should be unchanged (or improved) from the preoperative assessment. If the numbness and tingling are new, this information should be immediately reported to the surgeon. Numbness and tingling are not symptoms associated with hemorrhage at the site. Turning the patient will not relieve the numbness.

18. When planning care for a patient who will be treated with 2 days of bed rest for low back pain, which intervention will the nurse include? a. Telling the patient about the importance of a high fiber and fluid intake b. Instructing the patient to avoid positioning the knee in the flexed position c. Educating the patient that continuous heat application will reduce pain d. Teaching the patient that the prone position will help relieve back pain

Answer: A Rationale: Prevention of constipation caused by immobility is a goal for the patient with low back pain. The knee should be flexed to prevent pressure on the muscles and support structures of the spine. Heat and cold should be alternated. The prone position places more strain on the back and should be avoided. Cognitive Level: Application Text Reference: p. 1676 Nursing Process: Planning NCLEX: Physiological Integrity

14. The nurse determines that teaching regarding diet for a patient with osteoporosis has been successful when the patient selects which of these meals as having the highest amount of calcium? a. Sardine sandwich on whole wheat bread, one cup of fruit yogurt, and one cup of skim milk b. Two-egg omelet with American cheese, one slice of whole-wheat toast, and a half grapefruit c. Ham and Swiss cheese sandwich on whole-wheat bread, steamed broccoli, and an apple d. Chicken stir-fry with bok choy, onions, and snap peas and one cup of steamed brown rice

Answer: A Rationale: Sardines, yogurt, and milk are all high in calcium. The other choices have some foods that are high in calcium but also include foods that are low in calcium, such as eggs, apples, and grapefruit. Cognitive Level: Application Text Reference: p. 1689 Nursing Process: Evaluation NCLEX: Physiological Integrity

8. A patient whose work involves loading and unloading boxes has a history of chronic back pain. Which statement after the nurse has taught the patient about correct body mechanics indicates that the teaching has been effective? a. "I plan to start doing sit-ups and leg lifts to strengthen the muscles of my back." b. "I will try to sleep with my hips and knees extended to prevent back strain." c. "I can tell my boss that I need to change to a job where I can work at a desk." d. "I will keep my back straight when I need to lift anything higher than my waist."

Answer: A Rationale: Sit-ups and leg lifts will help to strengthen the muscles that support the back. Flexion of the hips and knees places less strain on the back. Modifications in the way the patient lifts boxes are needed, but sitting for prolonged periods can aggravate back pain. The patient should not lift above the level of the elbows. Cognitive Level: Application Text Reference: p. 1677 Nursing Process: Evaluation NCLEX: Health Promotion and Maintenance

11. After teaching a patient with a bunion about how to prevent further problems, the nurse will determine that more teaching is needed if the patient says, a. "I will wear soft slippers whenever possible." b. "I will throw away my high heel shoes." c. "I will use the bunion pad to relieve the pain." d. "I will take ibuprofen (Motrin) when I need it."

Answer: A Rationale: The shank of the shoe should be rigid enough to support the foot. The other patient statements indicate that the teaching has been effective. Cognitive Level: Application Text Reference: pp. 1684-1685 Nursing Process: Evaluation NCLEX: Physiological Integrity

16. When evaluating the effectiveness of treatment for a patient who is being treated for Paget's disease with calcitonin (Cibacalcin) and ibandronate (Boniva), the nurse will ask the patient about a. weight loss. b. skeletal pain. c. decreased appetite. d. frequent cough.

Answer: B Rationale: Bone pain is one of the common early manifestations of Paget's disease, and the nurse should ask about improvement in pain levels to determine whether the treatment is effective. Weight loss, anorexia, and frequent cough are not symptoms of Paget's disease. Cognitive Level: Application Text Reference: p. 1690 Nursing Process: Evaluation NCLEX: Physiological Integrity

17. A patient is receiving gentamicin (Garamycin) 80 mg IV twice daily for acute osteomyelitis. Which action should the nurse take before administering the gentamicin? a. Obtain the patient's oral temperature. b. Review the patient's BUN and creatinine levels. c. Ask the patient about any nausea. d. Change the wet-to-dry dressing.

Answer: B Rationale: Gentamicin is nephrotoxic and can cause renal failure. Monitoring the patient's temperature before gentamicin administration is not necessary. Nausea is not a common side effect of IV gentamicin. There is no need to change the dressing before gentamicin administration. Cognitive Level: Application Text Reference: p. 1670 Nursing Process: Assessment NCLEX: Physiological Integrity

2. A patient is hospitalized for initiation of regional antibiotic perfusion for acute osteomyelitis of the right femur. Which intervention will be included in the plan of care? a. Frequent weight-bearing exercise b. Immobilization of the right leg c. Avoid administration of NSAIDs d. Support right leg in a flexed position

Answer: B Rationale: Immobilization of the affected leg helps to decrease pain and reduce the risk for pathologic fractures. Weight-bearing exercise increases the risk for pathologic fractures. NSAIDs are frequently prescribed to treat pain. Flexion of the affected limb is avoided to prevent contractures. Cognitive Level: Application Text Reference: pp. 1670-1671 Nursing Process: Planning NCLEX: Physiological Integrity

5. Which statement by a patient who is scheduled for an above-the-knee amputation for treatment of an osteogenic sarcoma of the right tibia indicates that patient teaching is needed? a. "I wish that I did not have to have chemotherapy after this surgery." b. "I do not mind the surgery because it will finally cure the cancer." c. "I know that I will need lots of physical therapy after surgery." d. "I will use the patient-controlled analgesia to help control my pain level after surgery."

Answer: B Rationale: Osteogenic sarcoma is an aggressive cancer with early metastasis and is not considered cured by surgery alone. Postoperative chemotherapy will also be required. The other patient statements indicate that patient teaching has been effective. Cognitive Level: Application Text Reference: p. 1674 Nursing Process: Evaluation NCLEX: Physiological Integrity

12. An assessment finding that alerts the nurse to the presence of osteoporosis in a middle-aged patient is a. the presence of bowed legs. b. measurable loss of height. c. an aversion to dairy products. d. statements about frequent falls.

Answer: B Rationale: Osteoporosis occurring in the vertebrae produces a gradual loss of height. Bowed legs are associated with osteomalacia. Low intake of dairy products is a risk factor for osteoporosis, but it does not indicate that osteoporosis is present. Frequent falls increase the risk for fractures but are not an indicator of osteoporosis. Cognitive Level: Comprehension Text Reference: p. 1687 Nursing Process: Assessment NCLEX: Physiological Integrity

13. A 58-year-old woman who has been menopausal for 5 years is diagnosed with osteoporosis following densitometry testing. The woman has been concerned about her risk for osteoporosis because her mother has the condition. In teaching the woman about her osteoporosis, the nurse explains that a. with a family history of osteoporosis, there is no way to prevent or slow gradual bone resorption. b. estrogen replacement therapy must be started to prevent rapid progression of her osteoporosis. c. even with a family history of osteoporosis, the calcium loss from bones can be slowed by increased calcium intake and exercise. d. continuous, low-dose corticosteroid treatment is effective in stopping the course of osteoporosis.

Answer: C Rationale: Progression of osteoporosis can be slowed by increasing calcium intake and weight-bearing exercise. Estrogen replacement therapy does help to prevent osteoporosis, but it is not the only treatment and is not appropriate for some patients. Corticosteroid therapy increases the risk for osteoporosis. Cognitive Level: Application Text Reference: p. 1689 Nursing Process: Implementation NCLEX: Physiological Integrity

19. Which of these nursing actions included in the care of a patient after laminectomy can the nurse delegate to an experienced nursing assistant? a. Evaluation of the effectiveness of the PCA b. Monitoring plantar and dorsiflexion of the feet c. Logrolling the patient from side to side every 2 hours d. Determining the patient's readiness to ambulate

Answer: C Rationale: Repositioning a patient is included in the education and scope of practice of nursing assistants, and an experienced nursing assistant would be familiar with logrolling. Evaluation of the effectiveness of pain medications, assessment of neurologic function, and evaluation of a patient's readiness to ambulate after surgery require higher-level nursing education and scope of practice. Cognitive Level: Application Text Reference: pp. 1683-1684 Nursing Process: Planning NCLEX: Safe and Effective Care Environment

7. The nurse identifies a nursing diagnosis of pain related to muscle spasms for a patient with acute low back pain associated with acute lumbosacral strain. An appropriate nursing intervention for this problem is to teach the patient to a. twist gently from side to side to maintain range-of-motion in the spine. b. place a small pillow under the upper back to flex the lumbar spine gently. c. keep the head elevated slightly and flex the knees when resting in bed. d. avoid the use of cold because it will exacerbate the muscle spasms.

Answer: C Rationale: Resting with the head elevated and knees flexed will reduce the strain on the back and decrease muscle spasms. Twisting from side to side will increase tension on the lumbar area. A pillow placed under the upper back will cause strain on the lumbar spine. Alternate application of cold and heat should be used to decrease pain. Cognitive Level: Application Text Reference: pp. 1676-1677 Nursing Process: Planning NCLEX: Physiological Integrity

3. A patient is being discharged after 2 weeks of IV antibiotic therapy for acute osteomyelitis in the left leg. Which information will be included in the discharge teaching? a. The reason for taking oral antibiotics for 7 to 10 days after discharge b. The need for daily aerobic exercise to help maintain muscle strength c. How to monitor and care for the long-term IV catheter site d. How to apply warm packs safely to the leg to reduce pain

Answer: C Rationale: The patient will be on IV antibiotics for several months, and the patient will need to recognize signs of infection at the IV site and how to care for the catheter during daily activities such as bathing. IV antibiotics rather than oral antibiotics are used for acute osteomyelitis. Patients are instructed to avoid exercise and heat application because these will increase swelling and the risk for spreading infection. Cognitive Level: Application Text Reference: p. 1670 Nursing Process: Implementation NCLEX: Physiological Integrity

15. When administering alendronate (Fosamax) to a patient, the nurse will first a. administer the ordered calcium carbonate. b. be sure the patient has recently eaten. c. assist the patient to sit up at the bedside. d. ask about any leg cramps or hot flashes.

Answer: C Rationale: To avoid esophageal erosions, the patient taking bisphosphonates should be upright for at least 30 minutes after taking the medication. Fosamax should be taken on an empty stomach, not after taking other medications or eating. Leg cramps and hot flashes are not side effects of bisphosphonates. Cognitive Level: Application Text Reference: p. 1689 Nursing Process: Implementation NCLEX: Physiological Integrity

Which of the following observations by the nurse warrants further investigation to determine if the client has rheumatoid arthritis (RA)? Complaints of pain with movement Negative family history Complaints of prolonged morning stiffness lasting for 1 hour Occasional use of NSAIDS for aches and pains

Answer: Complaints of prolonged morning stiffness lasting for 1 hour Rationale: Prolonged morning stiffness is associated with RA. Occasional use of NSAIDS is not by itself a direct link to RA. Pain with movement is more likely associated with degenerative joint disease. Negative family history indicates the client does not have an elevated risk for RA.

Prednisone is prescribed for a patient with an acute exacerbation of rheumatoid arthritis. When the patient has a follow-up visit 1 month later, the nurse recognizes that the patient's response to the treatment may be best evaluated by a. blood glucose testing. b. liver function tests. c. serum electrolyte levels. d. C-reactive protein level.

Answer: D Rationale: ****C-reactive protein is a marker for inflammation****, and a decrease would indicate that the corticosteroid therapy was effective. Blood glucose and serum electrolyte levels will also be monitored to check for side effects of prednisone. Liver function is not routinely monitored for patients receiving steroids.

6. A 20-year-old patient with a 6-year history of muscular dystrophy is hospitalized with a respiratory tract infection. Which nursing action will be included in the plan of care? a. Logroll the patient every 1 to 2 hours. b. Teach the patient about the muscle biopsy procedure. c. Provide the patient with a pureed diet. d. Assist the patient with active range-of-motion (ROM) exercises.

Answer: D Rationale: The goal for the patient with muscular dystrophy is to keep the patient active for as long as possible. The patient would not be confined to bed rest and would not require logrolling. Muscle biopsies are necessary to confirm the diagnosis but would not be ordered for a patient who already had a diagnosis. There is no indication that the patient requires a pureed diet. Cognitive Level: Application Text Reference: p. 1675 Nursing Process: Planning NCLEX: Physiological Integrity

10. Following a laminectomy with a spinal fusion, a patient reports numbness and tingling of the right lower leg. The first action indicated by the nurse is to a. report the patient's complaint to the surgeon. b. check the vital signs for indications of hemorrhage. c. turn the patient to the side to relieve pressure on the operative area. d. check the chart for preoperative neuromuscular assessment data.

Answer: D Rationale: The postoperative movement and sensation of the extremities should be unchanged (or improved) from the preoperative assessment. If the numbness and tingling are new, this information should be immediately reported to the surgeon. Numbness and tingling are not symptoms associated with hemorrhage at the site. Turning the patient will not relieve the numbness. Cognitive Level: Application Text Reference: p. 1683 Nursing Process: Implementation NCLEX: Physiological Integrity

6. Which factors may be associated with Paget's disease? (Select all that apply.) a. Loss of height b. Apathy or lethargy c. Urinary or renal stones d. Cigarette smoker e. Muscle cramps

Apathy or lethargy Urinary or renal stones

Tamoxifen (Soltamox) Indications Estrogen-receptor positive metastatic breast cancer. Adjuvant treatment of early-stage estrogen-receptor positive breast cancer. To reduce the occurrence of contralateral breast cancer when used as adjuvant therapy for breast cancer. Action Competes with estrogen for binding sites in breast and other tissues. Reduces DNA synthesis and estrogen response. Contraindication/Precautions Contraindicated in: Hypersensitivity; Concurrent warfarin therapy with history of deep vein thrombosis (patients at high risk for breast cancer only); Adverse Reactions/Side Effects CNS: STROKE, confusion, depression, headache, weakness. CV: THROMBOEMBOLISM, edema EENT: blurred vision F and E: hypercalcemia GI: nausea, vomiting GU: UTERINE MALIGNANCIES, vaginal bleeding Route/Dosage Metastatic Breast Cancer PO (Adults): 20 mg once daily or 20 mg twice daily. Adjuvant Treatment of Breast Cancer PO (Adults): 20 mg once daily for 5-10 yrs. Prevention of Breast Cancer in High-Risk Women or Ductal Carcinoma in Situ PO (Adults): 20 mg once daily for 5 yrs.

Assessment Assess for an increase in bone or tumor pain. Confer with health care professional regarding analgesics. This transient pain usually resolves despite continued therapy. Implementation PO Administer with food or fluids if GI irritation becomes a problem. Consult health care professional if patient vomits shortly after administration of medication to determine need for repeat dose.Do not crush, break, chew, or administer an antacid within 1-2 hr of enteric-coated tablet. Patient/Family Teaching Instruct patient to notify health care professional promptly if pain or swelling of legs, shortness of breath, weakness, sleepiness, confusion, nausea, vomiting, weight gain, dizziness, headache, loss of appetite, or blurred vision occurs. Patient should also report menstrual irregularities, vaginal bleeding, pelvic pain or pressure. Evaluation/Desired Outcomes Decrease in the size or spread of breast cancer. Observable effects of therapy may not be seen for 4-10 wk after initiation.

ferrous sulfate (Feosol) Indications PO Treatment & prevention iron deficiency anemia. Action An essential mineral found in hemoglobin, myoglobin, and many enzymes. Enters the bloodstream and is transported to the organs of the reticuloendothelial system (liver, spleen, bone marrow) where it becomes part of iron stores. Contraindication/Precautions Contraindicated in: Anemia not due to iron deficiency Hemochromatosis Hemosiderosis Hypersensitivity to iron products. Adverse Reactions/Side Effects CNS: dizziness, headache, syncope GI: nausea, constipation, dark stools, epigastric pain, GI bleeding, vomiting Misc: temporary staining of teeth (liquid preparations) Route/Dosage Oral iron dosages are expressed as mg of elemental iron. Multiple salt forms exist-see approximate equivalent doses below or consider % elemental iron of each salt for dose conversions. PO (Adults): Deficiency -2-3 mg/kg/day in 2-4 divided doses or 60-100 mg elemental iron twice daily. Prophylaxis- 60-100 mg elemental iron daily.

Assessment Assess nutritional status and dietary history to determine possible cause of anemia and need for patient teaching. Assess bowel function for constipation or diarrhea. Notify physician or other health care professional and use appropriate nursing measures should these occur. Implementation Discontinue oral iron preparations prior to parenteral administration. Patient/Family Teaching Explain purpose of iron therapy to patient. Encourage patient to comply with medication regimen. Take missed doses as soon as remembered within 12 hr; otherwise, return to regular dosing schedule. Do not double doses. Evaluation/Desired Outcomes Increase in hemoglobin, which may reach normal parameters after 1-2 mo of therapy. May require 3-6 mo for normalization of body iron stores. Improvement in or prevention of iron deficiency anemia.

Abacavir (Ziagen) Indications Treatment of HIV-1 infection. Action Converted inside cells to carbovir triphosphate, its active metabolite. Carbovir triphosphate inhibits the activity of HIV-1 reverse transcriptase, which in turn terminates viral DNA growth. Contraindication/Precautions Contraindicated in: Hypersensitivity (rechallenge may be fatal); Presence of HLA-B*5701 allele; Moderate to severe hepatic impairment; Concurrent use of antiretroviral combination products containing abacavir; Lactation: Breast feeding not recommended for HIV-infected patients. Adverse Reactions/Side Effects CNS: headache, insomnia CV: MYOCARDIAL INFARCTION Derm: rash F and E: LACTIC ACIDOSIS GI: HEPATOMEGALY (WITH STEATOSIS), diarrhea, nausea, vomiting, anorexia Misc: HYPERSENSITIVITY REACTIONS, immune reconstitution syndrome Route/Dosage PO (Adults): 300 mg twice daily. PO (Children ≥3 mo): Oral solution- 8 mg/kg twice daily or 16 mg/kg once daily (not to exceed 600 mg/day); Tablets- 14-19 kg: 150 mg twice daily or 300 mg once daily; 20-24 kg: 150 mg in AM and 300 mg in PM or 450 mg once daily; ≥25 kg: 300 mg twice daily or 600 mg once daily.

Assessment Assess patient for change in severity of HIV symptoms and for symptoms of opportunistic infections throughout therapy.May cause lactic acidosis and severe hepatomegaly with steatosis. Monitor patient for signs (↑ serum lactate levels, ↑ liver enzymes, liver enlargement on palpation). Therapy should be suspended if clinical or laboratory signs occur. Implementation PO May be administered with or without food. Oral solution is clear to opalescent and yellow, may turn brown over time. Store at room temperature or refrigerated; do not freeze. Tablet may be used with children if able to swallow and dose is correctly calculated. Patient/Family Teaching Advise patient of potential for hypersensitivity reactions that may result in death. Instruct patient to discontinue abacavir and notify health care professional immediately if symptoms of hypersensitivity or signs of Immune Reconstitution Syndrome (signs and symptoms of an infection) occur. A warning card summarizing symptoms of abacavir hypersensitivity is provided with each prescription; instruct patient to carry card at all times. Evaluation/Desired Outcomes Delayed progression of AIDS, and decreased opportunistic infections in patients with HIV. Decrease in viral load and increase in CD4 cell counts.

Alendronic acid (Fosomax) Indications Treatment and prevention of postmenopausal osteoporosis. Treatment of osteoporosis in men. Treatment of Paget's disease of the bone. Action Inhibits resorption of bone by inhibiting osteoclast activity. Contraindication/Precautions Contraindicated in: Abnormalities of the esophagus which delay esophageal emptying (i.e. strictures, achalasia); Inability to stand/sit upright for at least 30 min; Renal impairment (CCr <35 mL/min); Adverse Reactions/Side Effects CNS: headache CV: atrial fibrillation Derm: erythema, photosensitivity, rash EENT: blurred vision, conjunctivitis, eye pain/inflammation Route/Dosage PO (Adults): Treatment of osteoporosis- 10 mg once daily or 70 mg once weekly. Prevention of osteoporosis- 5 mg once daily or 35 mg once weekly. Paget's disease- 40 mg once daily for 6 mo. Retreatment may be considered for patients who relapse. Treatment of corticosteroid-induced osteoporosis in men and premenopausal women- 5 mg once daily. Treatment of corticosteroid-induced osteoporosis in postmenopausal women not receiving estrogen- 10 mg once daily.

Assessment Assess patients for low bone mass before and periodically during therapy. Paget's Disease: Assess for symptoms of Paget's disease (bone pain, headache, decreased visual and auditory acuity, increased skull size). Implementation PO Administer first thing in the morning with 6-8 oz plain water 30 min before other medications, beverages, or food. Oral solution should be followed by at least 2 oz of water. Swallow tablets whole; do not crush, break, or chew. Patient/Family Teaching Instruct patient on the importance of taking exactly as directed, first thing in the morning, 30 min before other medications, beverages, or food. Waiting longer than 30 min will improve absorption. Alendronate should be taken with 6-8 oz plain water (mineral water, orange juice, coffee, and other beverages decrease absorption). If a dose is missed, skip dose and resume the next morning; do not double doses or take later in the day. If a weekly dose is missed, take the morning after remembered and resume the following wk on the chosen day. Do not take 2 tablets on the same day. Do not discontinue without consulting health care professional. Evaluation/Desired Outcomes Prevention of or decrease in the progression of osteoporosis in postmenopausal women. Reassess need for medication periodically. Consider discontinuation of alendronate after 3-5 yr in patients with low-risk of fractures. If discontinued, reassess fracture risk periodically. Treatment of osteoporosis in men. Decrease in the progression of Paget's disease. Treatment of corticosteroid-induced osteoporosis.

Epoetin alfta (Epogen) Indications Anemia associated with chronic kidney disease (CKD). Anemia secondary to zidovudine (AZT) therapy in HIV-infected patients. Anemia from chemotherapy in patients with nonmyeloid malignancies when there is ≥2 additional mo of planned chemotherapy. Action Stimulates erythropoiesis (production of red blood cells). Contraindication/Precautions Contraindicated in: Hypersensitivity to albumin or mammalian cell-derived products; Uncontrolled hypertension; Patients with erythropoietin levels >200 mUnits/mL; Patients with cancer receiving hormonal agents, biologic products, or radiotherapy, unless also receiving concomitant myelosuppressive chemotherapy; Adverse Reactions/Side Effects CNS: SEIZURES, headache CV: HF, MI, STROKE, THROMBOEMBOLIC EVENTS (ESPECIALLY WITH HEMOGLOBIN >11 G/DL), hypertension Derm: ERYTHEMA MULTIFORME, STEVENS-JOHNSON SYNDROME, TOXIC EPIDERMAL NECROLYSIS, transient rashes Endo: restored fertility, resumption of menses Misc: ↑ MORTALITY AND ↑ TUMOR GROWTH (WITH HEMOGLOBIN >12 G/DL) Route/Dosage Anemia of CKD (Do not initiate if hemoglobin ≥10 g/dL) SC IV (Adults): 50-100 units/kg 3 times weekly initially; use lowest dose sufficient to ↓ the need for red blood cell transfusions (do not exceed hemoglobin of 11 g/dL [patients on dialysis] or 10 g/dL [patients not on dialysis]); if Hgb ↑ by >1.0 g/dL in 2 wk, ↓ dose by 25%; if Hgb ↑ by <1.0 g/dL after 4 wk of therapy (with adequate iron stores), ↑ dose by 25%; do not ↑ dose more frequently than every 4 wk.

Assessment Monitor BP before and during therapy. Inform health care professional if severe hypertension is present or if BP begins to increase. Additional antihypertensive therapy may be required during initiation of therapy. Monitor for symptoms of anemia (fatigue, dyspnea, pallor). Monitor dialysis shunts (thrill and bruit) and status of artificial kidney during hemodialysis. Heparin dose may need to be increased to prevent clotting. Monitor patients with underlying vascular disease for impaired circulation. Implementation IV Administration Transfusions are still required for severe symptomatic anemia. Supplemental iron should be initiated with epoetin and continued throughout therapy. Patient/Family Teaching Advise patient to read the Medication Guide prior to initiating therapy and with each Rx refill in case of changes. Patient must sign the patient-health care provider acknowledgment form before each course of therapy. Evaluation/Desired Outcomes Increase in hematocrit to 30-36% with improvement in symptoms of anemia in patients with chronic renal failure. Increase in hematocrit in anemia secondary to zidovudine therapy. Increase in hematocrit in patients with anemia resulting from chemotherapy. Reduction of need for transfusions after surgery.

Denosumab (Prolia) Indications Treatment of osteoporosis in postmenopausal women who are at high risk for fracture or those who have failed/are intolerant of conventional osteoporosis therapy. Action A monoclonal antibody that binds specifically to the human receptor activator of nuclear factor kappa-B-ligand (RANKL), which is required for formation, function, and survival of osteoclasts. Binding inhibits osteoclast formation, function, and survival. Contraindication/Precautions Contraindicated in: Hypersensitivity; Hypocalcemia (correct before administering); adequate supplemental calcium and vitamin D required; Adverse Reactions/Side Effects CNS: headache Derm: dermatitis, eczema, rashes F and E: hypocalcemia, hypophosphatemia, hypercalcemia GI: PANCREATITIS, diarrhea, nausea GU: cystitis Metabolic: hypercholesterolemia MS: back pain, extremity pain, musculoskeletal pain, atypical femoral fracture, osteonecrosis of the jaw, suppression of bone turnover Route/Dosage Prolia SC (Adults): 60 mg every 6 mo. Xgeva SC (Adults ): Multiple myeloma or bone metastasis from solid tumors- 120 mg every 4 wk; Giant cell tumor of bone- 120 mg every 4 wk, with additional doses of 120 mg given on Days 8 and 15 of first mo of therapy; Hypercalcemia of malignancy- 120 mg every 4 wk, with additional doses of 120 mg given on Days 8 and 15 of first mo of therapy.

Assessment Monitor for signs and symptoms of hypersensitivity reactions (hypotension, dyspnea, upper airway edema, lip swelling, rash, pruritus, urticaria). Treat symptomatically and discontinue medication if symptoms occur. Implementation Grey needle cap on single-use prefilled Prolia syringe should not be handled by people sensitive to latex. Patient/Family Teaching Explain the purpose of denosumab to patient. If a dose is missed, administer injection as soon as possible. Advise patient once treatment is stopped there may be an increased risk of having broken bones in the spine, especially in patients who have had a fracture or who have had osteoporosis. Advise patients not to interrupt therapy without their health care professional advice. Evaluation/Desired Outcomes Reversal of the progression of osteoporosis with ↓ fractures and other sequelae. ↑ bone mass. Decreased growth of giant cell tumors. Reduction in hypercalcemia of malignancy that is refractory to bisphosphonate therapy.

Allopurinol (Zyloprim) Indications PO Prevention of attack of gouty arthritis and nephropathy. PO IV Treatment of secondary hyperuricemia, which may occur during treatment of tumors or leukemias. Action Inhibits the production of uric acid by inhibiting the action of xanthine oxidase. Contraindication/Precautions Contraindicated in: Hypersensitivity. Use Cautiously in: Acute attacks of gout; Adverse Reactions/Side Effects CNS: drowsiness CV: hypotension, flushing, hypertension, bradycardia, and heart failure (reported with IV administration) Derm: rash (discontinue drug at first sign of rash), urticaria GI: diarrhea, hepatitis, nausea, vomiting GU: renal failure, hematuria Hemat: bone marrow depression Route/Dosage Management of Gout PO (Adults and Children >10 yr): Initially- 100 mg/day; ↑ at weekly intervals based on serum uric acid (not to exceed 800 mg/day). Doses >300 mg/day should be given in divided doses; Maintenance dose- 100-200 mg 2-3 times daily. Doses of ≤300 mg may be given as a single daily dose.

Assessment Monitor intake and output ratios. Decreased kidney function can cause drug accumulation and toxic effects. Ensure that patient maintains adequate fluid intake (minimum 2500-3000 mL/day) to minimize risk of kidney stone formation. Implementation Do not confuse Zyloprim with zolpidem. PO May be administered after milk or meals to minimize gastric irritation; give with plenty of fluid. May be crushed and given with fluid or mixed with food for patients who have difficulty swallowing. Patient/Family Teaching Alkaline diet may be ordered. Urinary acidification with large doses of vitamin C or other acids may increase kidney stone formation Evaluation/Desired Outcomes Decreased serum and urinary uric acid levels. May take 2-6 wk to observe clinical improvement in patients treated for gout.

Etanercept (Enbrel) Indications Treatment of the following conditions:Moderately to severely active rheumatoid arthritis (may be used alone or with methotrexate),Moderate to severely active polyarticular juvenile idiopathic arthritis,Active ankylosing spondylitis. Action Binds to tumor necrosis factor (TNF), making it inactive. TNF is a mediator of inflammatory response. Contraindication/Precautions Contraindicated in: Hypersensitivity; Active infection (including localized); Untreated infections; Granulomatosis with polyangiitis (receiving immunosuppressive agents); Concurrent cyclophosphamide or anakinra; Lactation. Adverse Reactions/Side Effects CNS: headache, dizziness, weakness. Derm: psoriasis, rash EENT: rhinitis, pharyngitis GI: abdominal pain, dyspepsia Route/Dosage Rheumatoid Arthritis, Psoriatic Arthritis, and Ankylosing Spondylitis SC (Adults): 50 mg once weekly. Plaque Psoriasis SC (Adults): 50 mg twice weekly for 3 mo, then 50 mg once weekly, may also be given as 25-50 mg once weekly as an initial dose.

Assessment Monitor patients who develop a new infection while taking etanercept closely. Discontinue therapy in patients who develop a serious infection or sepsis. Do not initiate therapy in patients with active infections. Implementation Do not confuse Enbrel (etanercept) with Levbid (hyoscyamine). Administer a tuberculin skin test prior to administration of etanercept. Patients with active latent TB should be treated for TB prior to therapy. Patient/Family Teaching Instruct patient to notify health care professional if upper respiratory or other infections occur. Therapy may need to be discontinued if serious infection occurs. Evaluation/Desired Outcomes Reduction in symptoms of rheumatoid arthritis. Symptoms may return within 1 mo of discontinuation of therapy. Reduced severity of plaques in chronic plaque psoriasis.

Methotrexate (Otrexup) Indications Alone or with other treatment modalities in the treatment of:Trophoblastic neoplasms (choriocarcinoma, chorioadenoma destruens, hydatidiform mole),Leukemias,Breast carcinoma,Head carcinoma,Neck carcinoma,Lung carcinoma. Action Interferes with folic acid metabolism. Result is inhibition of DNA synthesis and cell reproduction (cell-cycle S-phase-specific). Also has immunosuppressive activity. Contraindication/Precautions Contraindicated in: Hypersensitivity; Alcoholism or hepatic impairment Immunosuppression ↓ bone marrow reserve; Adverse Reactions/Side Effects CNS: arachnoiditis (IT use only), dizziness, drowsiness, headache, malaise, seizures Derm: ERYTHEMA MULTIFORME, STEVENS-JOHNSON SYNDROME, TOXIC EPIDERMAL NECROLYSIS, alopecia, painful plaque erosions (during psoriasis treatment), photosensitivity, pruritus, rash, skin ulceration, soft tissue necrosis, urticaria Route/Dosage Trophoblastic Neoplasms PO IM (Adults): 15-30 mg/day for 5 days; repeat after 1 or more wk for 3-5 courses. Breast Cancer IV (Adults): 40 mg/m2 on days 1 and 8 (with other agents; many regimens are used). Leukemia PO (Adults): Induction- 3.3 mg/m2 /day, usually with prednisone. PO IM (Adults): Maintenance- 20-30 mg/m2 twice weekly.

Assessment Monitor vital signs periodically during administration. Report significant changes.Monitor for abdominal pain, diarrhea, or stomatitis; therapy may need to be discontinued.Monitor for bone marrow depression. Assess for bleeding (bleeding gums, bruising, petechiae, guaiac stools, urine, and emesis) and avoid IM injections and taking rectal temperatures if platelet count is low. Apply pressure to venipuncture sites for 10 min. Assess for signs of infection during neutropenia. Anemia may occur. Monitor for increased fatigue, dyspnea, and orthostatic hypotension. Implementation High Alert: Fatalities have occurred with chemotherapeutic agents. Before administering, clarify all ambiguous orders; double-check single, daily, and course-of-therapy dose limits; have second practitioner independently double-check original order, calculations and infusion pump settings. Methotrexate for non-oncologic use is given at a much lower dose and frequency-often just once a wk. Do not confuse non-oncologic dosing regimens with dosing regimens for cancer patients. Do not confuse methotrexate with metolazone. Patient/Family Teaching Instruct patient to notify health care professional promptly if rash, fever; chills; cough; hoarseness; sore throat; signs of infection; lower back or side pain; painful or difficult urination; bleeding gums; bruising; petechiae; blood in stools, urine, or emesis; increased fatigue; dyspnea; or orthostatic hypotension occurs. Caution patient to avoid crowds and persons with known infections. Instruct patient to use soft toothbrush and electric razor and to avoid falls. Caution patient not to drink alcoholic beverages or take medication containing aspirin or other NSAIDs; may precipitate gastric bleeding. Evaluation/Desired Outcomes Improvement of hematopoietic values in leukemia.

Vancomycin (Vancocin) Indications IV Treatment of potentially life-threatening infections when less toxic anti-infectives are contraindicated. Particularly useful in staphylococcal infections Action Binds to bacterial cell wall, resulting in cell death. Contraindication/Precautions Contraindicated in: Hypersensitivity. Use Cautiously in: Renal impairment (dosage reduction required if CCr ≤80 mL/min); Hearing impairment; Intestinal obstruction or inflammation (↑ systemic absorption when given orally); Adverse Reactions/Side Effects CV: hypotension Derm: rash EENT: ototoxicity GI: nausea, vomiting GU: nephrotoxicity Hemat: eosinophilia, leukopenia Route/Dosage Serious Systemic Infections IV (Adults): 500 mg every 6 hr or 1 g every 12 hr (up to 4 g/day).

Assessment Observe patient for signs and symptoms of anaphylaxis (rash, pruritus, laryngeal edema, wheezing). Discontinue drug and notify health care professional immediately if these problems occur. Keep epinephrine, an antihistamine, and resuscitation equipment close by in case of an anaphylactic reaction. Implementation Vancomycin must be given orally for treatment of staphylococcal enterocolitis and C. difficile-associated diarrhea. Orally administered vancomycin is not effective for other types of infections. PO Use calibrated measuring device for liquid preparations. Oral solution is stable for 14 days if refrigerated. Patient/Family Teaching Advise patients on oral vancomycin to take as directed. Take missed doses as soon as remembered unless almost time for next dose; do not double dose. Instruct patient to report signs of hypersensitivity, tinnitus, vertigo, or hearing loss. Evaluation/Desired Outcomes Resolution of signs and symptoms of infection. Length of time for complete resolution depends on organism and site of infection. Endocarditis prophylaxis.

Penicillin Indications Treatment of a wide variety of infections including:Pneumococcal pneumonia,Streptococcal pharyngitis,Syphilis,Gonorrhea strains. Action\ Bind to bacterial cell wall, resulting in cell death. Contraindication/Precautions Contraindicated in: Previous hypersensitivity to penicillins (cross-sensitivity exists with cephalosporins and other beta-lactam antibiotics) Some products may contain tartrazine and should be avoided in patients with known hypersensitivity. Adverse Reactions/Side Effects CNS: SEIZURES Derm: rash, urticaria GI: diarrhea, epigastric distress, nausea, vomiting, Clostridioides difficile-associated diarrhea (CDAD) GU: interstitial nephritis Hemat: eosinophilia, leukopenia Route/Dosage 250 mg = 400,000 units PO (Adults and Children ≥12 yr): Most infections- 125-500 mg every 6-8 hr. Rheumatic fever prevention- 125-250 mg every 12 hr.

Assessment Observe patient for signs and symptoms of anaphylaxis (rash, pruritus, laryngeal edema, wheezing). Discontinue drug and notify physician or other health care professional immediately if these symptoms occur. Keep epinephrine, an antihistamine, and resuscitation equipment close by in case of an anaphylactic reaction. Implementation Do not confuse penicillin with penicillamine. PO Administer around the clock. Penicillin V may be administered without regard for meals. Patient/Family Teaching Instruct patient to notify health care professional if fever and diarrhea develop, especially if stool contains blood, pus, or mucus. Advise patient not to treat diarrhea without consulting health care professional. Evaluation/Desired Outcomes Resolution of signs and symptoms of infection. Length of time for complete resolution depends on the organism and site of infection.

A client recently diagnosed with fibromyalgia is receiving education from the nurse. Which reasons should the nurse provide to the client regarding pain perception occurring at lower levels of stimulation with this​ disorder? ​(Select all that​ apply.) Autoantibodies attacking host tissues Abnormal hypothalamic response Decreased neurotransmitter levels Fat and connective tissue replace muscle fibers Autonomic nervous system differences

Autonomic nervous system differences Abnormal hypothalamic response A client with fibromyalgia perceives pain at a lower level of stimulation because of increased neurotransmitter levels and an abnormal hypothalamic response. A client with fibromyalgia perceives pain at a lower level of stimulation because of​ increased, not​ decreased, neurotransmitter levels. A client with fibromyalgia does not perceive pain at a lower level of stimulation because of fat and connective tissue replacing muscle fibers or because of antibodies becoming autoantibodies and attacking host tissues.

During a health screening event which assessment finding would alert the nurse to the possible presence of osteoporosis in a white 61-year-old female? A. The presence of bowed legs B. A measurable loss of height C. Poor appetite and aversion to dairy products D. Development of unstable, wide-gait ambulation

B. A measurable loss of height A gradual but measurable loss of height and the development of kyphosis or "dowager's hump" are indicative of the presence of osteoporosis in which the rate of bone resorption is greater than bone deposition. Bowed legs may be caused by abnormal bone development or rickets but is not indicative of osteoporosis. Lack of calcium and Vitamin D intake may cause osteoporosis but are not indicative it is present. A wide gait is used to support balance and does not indicate osteoporosis.

The nurse is planning health promotion teaching for a 45-year-old patient with asthma, low back pain from herniated lumbar disc, and schizophrenia. What does the nurse determine would be the best exercise to include in an individualized exercise plan for the patient? A. Yoga B. Walking C. Calisthenics D. Weight lifting

B. Walking The patient would benefit from an aerobic exercise that takes into account the patient's health status and fits the patient's lifestyle. The best exercise is walking, which builds strength in the back and leg muscles without putting undue pressure or strain on the spine. Yoga, calisthenics, and weight lifting would all put pressure on or strain the spine.

A client with osteoporosis is asking the nurse regarding the use of Salmon calcitonin (Miacalcin) nasal spray. The nurse tells the client to do the following, except? a-Delivery system contains enough medication for at least 30 doses. Discard any unused solution after 30 doses. b-If you do not feel the spray while using it, repeat the dose on the other nostrils. c-Miacalcin is usually given as one spray per day into only one of your nostrils. d-Take extra vitamin D while you are using Miacalcin.

B: Miacalcin spray delivers a fine mist into the nose. Even if the client does not feel the spray while using it, the medication is still being absorbed by the nasal passages. A: Discard any unused solution after 30 doses because spray may not deliver correct dose. C: Use the other nostril the next day and continue alternating back and forth for each daily dose. D: Vitamin D helps in treating osteoporosis by helping in maintaining healthy bones.

What is the best way to diagnose osteoporosis

Bone density scan

A nurse is providing education to a community women's group about lifestyle changes helpful in preventing osteoporosis. What topics does the nurse cover? (Select all that apply.) a. Cut down on tobacco product use. b. Limit alcohol to two drinks a day. c. Strengthening exercises are important. d. Take recommended calcium and vitamin D. e. Walk 30 minutes at least 3 times a week.

C, D, E Lifestyle changes can be made to decrease the occurrence of osteoporosis and include strengthening and weight-bearing exercises and getting the recommended amounts of both calcium and vitamin D. Tobacco should be totally avoided. Women should not have more than one drink per day.

The nurse receives report from the licensed practical nurse about care provided to patients on the orthopedic surgical unit. It is most important for the nurse to follow up on which statement? A. "The patient who had a spinal fusion 12 hours ago has hypoactive bowel sounds and is not passing flatus." B. "The patient who had cervical spine surgery 2 days ago wants to wear her soft cervical collar when out of bed." C. "The patient who had spinal surgery 3 hours ago is complaining of a headache and has clear drainage on the dressing." D. "The patient who had a laminectomy 24 hours ago is using patient-controlled analgesia with morphine for pain management."

C. "The patient who had spinal surgery 3 hours ago is complaining of a headache and has clear drainage on the dressing." After spinal surgery there is potential for cerebrospinal fluid (CSF) leakage. Severe headache or leakage of CSF (clear or slightly yellow) on the dressing should be reported immediately. The drainage is CSF if a dipstick test is positive for glucose. Patients after spinal surgery may experience paralytic ileus and interference with bowel function for several days. Postoperatively most patients require opioids such as morphine IV for 24 to 48 hours. Patient-controlled analgesia is the preferred method for pain management during this time. After cervical spine surgery patients often wear a soft or hard cervical collar to immobilize the neck.

The nurse determines that dietary teaching for a 75-year-old patient with osteoporosis has been successful when the patient selects which highest-calcium meal? A. Chicken stir-fry with 1 cup each onions and green peas, and 1 cup of steamed rice B. Ham and Swiss cheese sandwich on whole wheat bread, steamed broccoli, and an apple C. A sardine (3 oz) sandwich on whole wheat bread, 1 cup of fruit yogurt, and 1 cup of skim milk D. A two-egg omelet with 2 oz of American cheese, one slice of whole wheat toast, and a half grapefruit

C. A sardine (3 oz) sandwich on whole wheat bread, 1 cup of fruit yogurt, and 1 cup of skim milk The highest calcium content is present in the lunch containing milk and milk products (yogurt) and small fish with bones (sardines). Chicken, onions, green peas, rice, ham, whole wheat bread, broccoli, apple, eggs, and grapefruit each have less than 75 mg of calcium per 100 g of food. Swiss cheese and American cheese have more calcium, but not as much as the sardines, yogurt, and milk.

A 50-year-old female patient is concerned that she will develop osteoporosis because both her maternal aunts have been diagnosed with the disorder. The nurse would suggest which intervention? 1. Prophylactic nonsteroidal anti-inflammatory medication 2. A DEXA test 3. Prescription for risedronate (Actonel) 4. Increase daily intake of calcium

Correct Answer: 2 Rationale Assessment of bone mass is the primary measurement for osteoporosis. The bone mineral density test, or DEXA test, uses a technique that measures any skeletal site and then compares bone density values with other values in a reference population of the same age, race, and gender.

The nurse is discussing the symptomology of osteoarthritis (OA) with a patient. The nurse would describe which common initial symptom of the disease? 1. A fine red rash on the elbow that is constant 2. Painful stiffness in the joints of the fingers 3. Popping sensation in the wrist joint when typing 4. Knee pain when the leg is at rest

Correct Answer: 2 Rationale The onset of osteoarthritis (OA) is gradual and progressive. The symptoms that are noticed first are pain and stiffness in the affected joint or joints.

A patient is receiving a series of diagnostic tests to confirm the diagnosis of osteoarthritis (OA). The nurse would interpret which results as supporting the diagnosis of OA? Select all that apply. 1. Presence of antinuclear antibodies in blood 2. Asymmetrical joint cartilage loss seen on X-ray 3. Increased erythrocyte sedimentation rate (ESR) in blood 4. Bone spurs visible on computed tomography (CT) 5. Increased bone density in Dexa scan

Correct Answer: 2,3,4,5 Rationale 1: The presence of antinuclear antibodies in blood is reflective of RA, not OA. Rationale 2: Asymmetrical joint cartilage loss is a positive diagnostic result for OA. Rationale 3: Increased ESR is a positive diagnostic result for OA. Rationale 4: Bone spurs are a positive diagnostic result for OA. Rationale 5: Increased bone density is a positive diagnostic result for OA.

The nurse is preparing a flyer on rheumatoid arthritis (RA) for distribution during a community health fair. Which information should the nurse include? Select all that apply. 1. Rheumatoid arthritis typically affects weight-bearing joints. 2. Onset generally occurs between 30 and 40 years of age. 3. Rheumatoid arthritis is the most common form of arthritis. 4. Women are more likely to be affected than men. 5. Rheumatoid arthritis appears to have a genetic component.

Correct Answer: 2,4,5 Rationale 1: RA most often affects the joints of the hands and feet. Osteoarthritis affects weight-bearing joints. Rationale 2: RA can occur at any age, with the peak incidence between ages 30 and 40. Rationale 3: Osteoarthritis (OA) is the most common form of arthritis. Rationale 4: RA affects three times more women than men worldwide. Rationale 5: RA is thought to be an autoimmune disorder that not only involves tissue hypersensitivity but also has a genetic component.

A 20-year-old baseball pitcher has an arthroscopic repair of a rotator cuff injury performed in same-day surgery. When the nurse plans postoperative teaching for the patient, which information will be included? a. "You have an appointment with a physical therapist for tomorrow." b. "Leave the shoulder immobilizer on for the first few days to minimize pain." c. "The doctor will use the drop-arm test to determine the success of the procedure." d. "You should try to find a different position to play on the baseball team."

Correct Answer: A Rationale: Physical therapy after a rotator cuff repair begins on the first postoperative day to prevent "frozen shoulder." A shoulder immobilizer is used immediately after the surgery, but leaving the arm immobilized for several days would lead to loss of ROM. The drop-arm test is used to test for rotator cuff injury, but not after surgery. The patient may be able to return to pitching after rehabilitation.

9. A patient with a fractured radius asks when the cast can be removed. The nurse will instruct the patient that the cast can be removed only after the bone a. is strong enough to stand mild stress. b. union is complete on the x-ray. c. fragments are fully fused. d. healing has started.

Correct Answer: A Rationale: The cast may be removed when callus ossification has occurred. It is not necessary to wait until radiologic union or complete bone fusion occurs. Bone healing starts immediately after the injury, but the cast will need to be worn at least 3 weeks.

11. A patient in the emergency department is diagnosed with a patellar dislocation. The initial patient teaching by the nurse will focus on the need for a. conscious sedation. b. a knee immobilizer. c. gentle knee flexion. d. cast application.

Correct Answer: A Rationale: The first goal of collaborative management is realignment of the knee to its original anatomic position, which will require anesthesia or conscious sedation. Immobilization of the joint will be done after realignment. Later, gentle ROM exercises may be started if the joint is stable. Casting is not usually required for dislocations. Cognitive Level: Application Text Reference: p. 1632 Nursing Process: Implementation NCLEX: Physiological Integrity

10. A patient with a comminuted fracture of the right femur has Buck's traction in place while waiting for surgery. To assess for pressure areas on the patient's back and sacral area and to provide skin care, the nurse should a. have the patient lift the buttocks by bending and pushing with the left leg. b. turn the patient partially to each side with the assistance of another nurse. c. place a pillow between the patient's legs and turn gently to each side. d. loosen the traction and have the patient turn onto the unaffected side.

Correct Answer: A Rationale: The patient can lift the buttocks off the bed by using the left leg without changing the right-leg alignment. Turning the patient will tend to move the leg out of alignment. Disconnecting the traction will interrupt the weight needed to immobilize and align the fracture.

7. A 22-year-old patient started an exercise regimen 2 months ago that includes running 3 to 4 miles a day. The patient tells the nurse, "I enjoy my daily runs, but now I have shin splints." Which response by the nurse is appropriate? a. "You may be increasing your running time too quickly and need to cut back a little bit." b. "You need to have x-rays of your lower legs to be sure you do not have stress fractures." c. "You should expect some leg pain while running." d. "You should try speed-walking rather than running."

Correct Answer: A Rationale: The patient's information about running 3 to 4 miles daily after starting an exercise program only 2 months previously suggests that the shin splints are caused by overuse. Radiographs are not indicated for the type of injury described by the patient. Shin splints are not a normal or expected response to running. Because the patient expresses enjoyment of running, it would not be appropriate for the nurse to suggest a different sport. Cognitive Level: Application Text Reference: p. 1630 Nursing Process: Implementation NCLEX: Physiological Integrity

13. Following the application of a hip spica cast for a patient with a fracture of the proximal third of the left femur, an appropriate nursing intervention is to a. use the cast support bar to reposition the patient every 2 to 3 hours. b. ask the patient about any abdominal discomfort or nausea. c. discuss the reasons for remaining on bed rest for several weeks. d. promote drying of the cast by placing the patient in a prone position every 4 hours.

Correct Answer: B Rationale: Assessment of bowel tones, abdominal pain, and nausea and vomiting will detect the development of cast syndrome. To avoid breakage, the support bar should not be used for repositioning. After the cast dries, the patient can begin ambulating with the assistance of physical therapy personnel. The patient should not be placed in the prone position until the cast has dried to avoid breaking the cast. Cognitive Level: Application Text Reference: p. 1640 Nursing Process: Implementation NCLEX: Physiological Integrity

Following x-rays of an injured wrist, the patient is informed that it is badly sprained. In teaching the patient to care for the injury, the nurse tells the patient to a. apply a heating pad to reduce muscle spasms. b. wear an elastic compression bandage continuously. c. use pillows to keep the arm elevated above the heart. d. gently exercise the joint to prevent muscle shortening.

Correct Answer: C Rationale: Elevation of the arm will reduce the amount of swelling and pain. For the first 24 to 48 hours, cold packs are used to reduce swelling. Compression bandages are not left on continuously. The wrist should be rested and kept immobile to prevent further swelling or injury.

4. When working with a patient whose job involves many hours of word processing, the nurse will teach the patient about the need to a. do stretching and warm-up exercises before starting work. b. wrap the wrists with a compression bandage every morning. c. use acetaminophen (Tylenol) instead of NSAIDs for wrist pain. d. obtain a keyboard pad to support the wrist while word processing.

Correct Answer: D Rationale: Repetitive strain injuries caused by prolonged times working at a keyboard can be prevented by the use of a pad that will keep the wrists in a straight position. Stretching exercises during the day may be helpful, but these would not be needed before starting. Use of a compression bandage is not needed, although a splint may be used for carpal tunnel syndrome. NSAIDs are appropriate to use to decrease swelling. Cognitive Level: Application Text Reference: p. 1633 Nursing Process: Implementation NCLEX: Health Promotion and Maintenance

The nurse is reinforcing health teaching about osteoporosis with a 72-year-old patient admitted to the hospital. In reviewing this disorder, what should the nurse explain to the patient? A. With a family history of osteoporosis, there is no way to prevent or slow bone resorption. B. Continuous, low-dose corticosteroid treatment is effective in stopping the course of osteoporosis. C. Estrogen therapy must be maintained to prevent rapid progression of the osteoporosis. D. Even with a family history of osteoporosis, the calcium loss from bones can be slowed by increased calcium intake and exercise.

D. Even with a family history of osteoporosis, the calcium loss from bones can be slowed by increased calcium intake and exercise. The rate of progression of osteoporosis can be slowed if the patient takes calcium supplements and/or foods high in calcium and engages in regular weight-bearing exercise. Corticosteroids interfere with bone metabolism. Estrogen therapy is no longer used to prevent osteoporosis because of the associated increased risk of heart disease and breast and uterine cancer.

The nurse is reinforcing health teaching about osteoporosis with a 72-year-old patient admitted to the hospital. In reviewing this disorder, what should the nurse explain to the patient? A. With a family history of osteoporosis, there is no way to prevent or slow bone resorption. B. Continuous, low-dose corticosteroid treatment is effective in stopping the course of osteoporosis. C. Estrogen therapy must be maintained to prevent rapid progression of the osteoporosis. D. Even with a family history of osteoporosis, the calcium loss from bones can be slowed by increased calcium intake and exercise.

D. Even with a family history of osteoporosis, the calcium loss from bones can be slowed by increased calcium intake and exercise. The rate of progression of osteoporosis can be slowed if the patient takes calcium supplements and/or foods high in calcium and engages in regular weight-bearing exercise. Corticosteroids interfere with bone metabolism. Estrogen therapy is no longer used to prevent osteoporosis because of the associated increased risk of heart disease and breast and uterine cancer.

44. A patient with bone sarcoma had surgery to salvage an upper limb. The nurse has identified the patient has impaired physical mobility related to musculoskeletal impairment. Which intervention does the nurse perform in the early postoperative period? a. Encourage the patient to use the opposite hand to achieve forward flexion and abduction of the affected shoulder. b. Encourage the patient to emphasize strengthening the quadriceps muscles by using passive and active motion. c. Instruct the UAP to completely perform hygiene for the patient until the patient expresses readiness to do self-care. d. Evaluate the patient's and family's readiness to use the continuous passive motion machine in the home setting.

Encourage the patient to use the opposite hand to achieve forward flexion and abduction of the affected shoulder.

When caring for a patient with metastatic cancer, the nurse notes a hemoglobin level of 8.7 g/dL and hematocrit of 26%. What associated clinical manifestations does the nurse anticipate observing? Thirst Fatigue Headache Abdominal pain

Fatigue

What is the medical term for the bone disease caused by reduced blood flow to the bones in the joints?

Osteonecrosis

The nurse is planning care for a client with fibromyalgia. Which potential problems are priorities for the nurse to​ address? ​(Select all that​ apply.) Risk for injury Decreased cardiac output Activity intolerance Fatigue Pain

Pain Activity intolerance Fatigue Pain is a priority potential problem for the nurse to address because of the pain associated with fibromyalgia. Activity intolerance and fatigue are priority potential problems for the nurse to address because of the exacerbation of pain and fatigue associated with exercise or increased activity. Risk for injury and decreased cardiac output are not priority potential problems for a client with fibromyalgia.

32. Which patient is mostly likely to be a candidate for hyperbaric oxygen therapy? a. Patient with chronic, unremitting osteomyelitis b. Patient with an advanced case of Paget's disease c. Patient with osteomalacia related to poverty d. Patient with osteoporosis and recurrent fractures

Patient with chronic, unremitting osteomyelitis

A patient with cancer is having chemotherapy treatments and has now developed neutropenia. What care should the nurse expect to provide and teach the patient about (select all that apply.)? Strict hand washing Daily nasal swabs for culture Monitor temperature every hour. Daily skin care and oral hygiene Encourage eating all foods to increase nutrients. Private room with a high-efficiency particulate air (HEPA) filter

Strict hand washing Daily skin care and oral hygiene Private room with a high-efficiency particulate air (HEPA) filter

A patient is prescribed Alendronate (Fosamax) at 0800 for the treatment of osteoporosis. As the nurse you know you must administer this medication: A. on an empty stomach with a full glass of water and keep the patient upright for 30 minutes. B. right after breakfast and to lay the patient flat (as tolerated) for 30 minutes. C. with food but to avoid giving this medication with dairy products. D. on an empty stomach with a full glass of juice or milk.

The answer is A. Alendronate (Fosamax) is a bisphosphonate which is known for causing GI upset, especially inflammation of the esophagus. These medications should be taken with a full glass of water in morning on empty stomach with NO other medication. The patient should sit up for 30 minutes (60 minutes with Boniva) after taking the medication, and not eat anything for 1 hour after taking (helps the body absorb more of the medicine.)

A patient is prescribed Alendronate (Fosamax) at 0800 for the treatment of osteoporosis. As the nurse you know you must administer this medication:* A. on an empty stomach with a full glass of water and keep the patient upright for 30 minutes. B. right after breakfast and to lay the patient flat (as tolerated) for 30 minutes. C. with food but to avoid giving this medication with dairy products. D. on an empty stomach with a full glass of juice or milk.

The answer is A. Alendronate (Fosamax) is a bisphosphonate which is known for causing GI upset, especially inflammation of the esophagus. These medications should be taken with a full glass of water in morning on empty stomach with NO other medication. The patient should sit up for 30 minutes (60 minutes with Boniva) after taking the medication, and not eat anything for 1 hour after taking (helps the body absorb more of the medicine.)

A patient is ordered by the physician to take Allopurinol (Zyloprim) for treatment of gout. You've provided education to the patient about this medication. Which statement by the patient requires you to re-educate them about this medication? A. "This medication will help relieve the inflammation and pain during an acute attack." B. "It is important I have regular eye exams while taking this medication." C. "I will not take large doses of vitamin C supplements while taking this medication." D. "Allopurinol decreases the production of uric acid."

The answer is A. Allopurinol is used to PREVENT gout attacks....not treat an acute attack. NSAIDs and Colchicine are best for treating the inflammation and pain during a gout attack. Allopurinol can cause vision changes, therefore the patient should receive annual eye exams along with avoiding large doses of vitamin C due to the risk of renal calculi formation.

Your patient is scheduled for a DEXA scan this morning. The patient is having heartburn and requests a PRN medication to help with relief. Which medications can the patient NOT have at this time?* A. Calcium Carbonate B. Bismuth Salicylate C. Milk of Magnesia D. Famotidine

The answer is A. Before a DEXA scan, which is a bone density test, the patient should not take any type of calcium supplements (calcium carbonate (TUMs) or vitamins containing calcium.

A patient is recovering after having an appendectomy. The patient is 48 hours post-op from surgery and is tolerating full liquids. The physician orders for the patient to try solid foods. What types of foods should the patient incorporate in their diet? A. Foods high in fiber Foods low in fiber Foods high in carbohydrates Foods low in protein

The answer is A. It is best for the patient to follow a diet high in fiber to prevent straining during bowel movements.

You are providing a free clinic seminar to participants about gout. Which statement by a participant about the occurrence of gout is correct? A. "Gout attacks tend to awake the person out of their sleep in the middle of the night." B. "The pain felt with gout tends to be intense during the first 30 minutes." C. "It is best for a patient experiencing gout to tightly bandage the affected extremity." D. "Typically acute gout attacks are predictable and tend to occur once or twice a week."

The answer is A. This is the only correct statement about gout. Option B is wrong because the pain felt with gout tends to intensify within 4-24 hours (not 30 minutes). Option C is wrong because any type of pressure (even the pressure of bed linens) can majorly increase the pain felt with gout. Option D is wrong because gout attacks come on suddenly and may only occur once and tend to have several months or a year between attacks.

Parathyroid hormone plays an important role in bone health. When the parathyroid gland secretes PTH (parathyroid hormone) it causes: A. the body to increase the calcium levels by stimulating the osteoclast activity B. the body to decrease the calcium levels by inhibiting osteoclast activity C. the body to increase the calcium levels by stimulating osteoblast activity D. the body to decrease the calcium levels by inhibiting osteoblast activity.

The answer is A. When the calcium levels are low this stimulates the parathyroid gland to secrete PTH, which stimulates osteoCLAST activity. Remember osteoCLASTS break down the bone matrix within the spongy bone. This will cause calcium to enter the blood stream, hence increasing calcium levels.

During a head-to-toe assessment of a patient with osteoarthritis, you note bony outgrowths on the distal interphalangeal joints. You document these findings as:* A. Bouchard's Nodes B. Heberden's Nodes C. Neurofibromatosis D. Dermatofibromas

The answer is B. Bony outgrowths found on the DISTAL interphalangeal joint (closest to the fingernail and furthest away from the body) is called Heberden's Node. If the bony outgrowth was found on the PROXIMAL interphalangeal joint (middle joint of the finger...closest to the body) it is called Bouchard's Node.

You are providing education to a patient, who was recently diagnosed with rheumatoid arthritis, about physical exercise. Which statement made by the patient is correct? A. "It is best I try to incorporate a moderate level of high impact exercises weekly into my routine, such as running and aerobics." B. "I will be sure to rest joints that are experiencing a flare-up, but I will try to maintain a weekly regime of range of motion exercises along with walking and riding a stationary bike." C. "It is important I perform range of motion exercises during joint flare-ups and incorporate low-impact exercises into my daily routine." D. "Physical exercise should be limited to only range of motion exercises to prevent further joint damage."

The answer is B. During flare-ups of RA the patient should rest the joint. However, it is important the patient performs range of motion exercises along with LOW-IMPACT exercise weekly (such as stationary bike riding, walking, water aerobics etc.). This will help with increasing the patient's energy level along with muscle strength and maintain joint health.

Identify which patient below is at MOST risk for developing gout:* A. A 56 year old male who reports consuming foods low in purines. B. A 45 year old male with a BMI of 40 who reports taking hydrochlorothiazide and aspirin. C. A 39 year old female hospitalized with bulimia that has a BMI of 24. D. A 27 year old female with ulcerative colitis.

The answer is B. Gout is due to high levels of uric acid in the blood. This can either be due to the kidney's inability to excrete uric acid out of the body or the body is producing too much uric acid. Some causes that can lead to increased uric acid levels include: being overweight (BMI >25 is considered overweight), usage of aspirin or diuretics (thiazides (hydrochlorothiazide) or loop), HIGH consumption of purines, dehydration, renal problems. Option B is the only patient MOST at risk for developing gout.

A patient is post-op from surgery. The patient has a history of gout. While performing a head-to-toe assessment, you assess the patient for signs and symptoms of gout. As the nurse, you know that gout tends to start at what site? A. Elbow B. Big toe C. Thumb or index finger D. Knees.

The answer is B. Most patients tend to have an acute attack of gout that begins in the big toe. Remember that patients who have a history of gout or who are experiencing a hospitalization (due to the physical stress on the body) are at risk for an acute gout attack. Therefore, the nurse should assess the patient for this during the head-to-toe assessment

A patient is post-op from surgery. The patient has a history of gout. While performing a head-to-toe assessment, you assess the patient for signs and symptoms of gout. As the nurse, you know that gout tends to start at what site?* A. Elbow B. Big toe C. Thumb or index finger D. Knees

The answer is B. Most patients tend to have an acute attack of gout that begins in the big toe. Remember that patients who have a history of gout or who are experiencing a hospitalization (due to the physical stress on the body) are at risk for an acute gout attack. Therefore, the nurse should assess the patient for this during the head-to-toe assessment.

You're providing care to a patient with severe rheumatoid arthritis. While performing the head-to-toe nursing assessment, you note the patient's overall skin color to be pale and the patient looks exhausted. You ask the patient how she is feeling, and she says "I'm so tired. I can't even get out of this bed without getting short of breath." Which finding on the patient's morning lab work may confirm a complication that can be experienced with rheumatoid arthritis? A. Potassium 3.2 mEq/L B. Hemoglobin 7 g/dL C. Sodium 135 mEq/L D. WBC count 6,500

The answer is B. Patients with RA can experience anemia. A hemoglobin level can be helpful in diagnosing anemia (a normal level in females is 12 to 15.5 g/dL). The patient's signs and symptoms above are classic findings in anemia.

9. A patient is scheduled for appendectomy at noon. While performing your morning assessment, you note that the patient has a fever of 103.8 'F and rates abdominal pain 9 on 1-10. In addition, the abdomen is distended and the patient states, "I was feeling better last night but it seems the pain has become worst." The patient is having tachycardia and tachypnea. Based on the scenario, what do you suspect the patient is experiencing? Pulmonary embolism Colon Fistulae Peritonitis Hemorrhage

The answer is C. Based on the patient's presenting symptoms, the patient is most likely experiencing peritonitis because the appendix has ruptured. The key clues in this scenario are the classic signs and symptoms of peritonitis (tachycardia, tachypnea, high temperature, and abdominal pain/distension) along with the patient's statement that they were feeling better last night (hence probably the time the appendix ruptured) which periodically relieved the pain at the appendix but allowed for the contents of the appendix to leak into the peritoneal cavity....hence causing peritonitis.

Bones play an important role in the body. Which of the following in NOT a function performed by the bones? A. Provide protection and support for the organs. B. Give the body shape. C. Secrete the hormone calcitonin and store blood cells. D. Store calcium and phosphorus.

The answer is C. Bones (specifically bone marrow) are responsible for red blood cell, platelet, and white blood cell production. In addition, it stores blood cells and minerals, such as calcium and phosphorous. Calcitonin is secreted by the thyroid gland NOT the bones. However, calcitonin causes osteoclast activity to be inhibited, but is not secreted by the bone.

11. A patient is taking Calcitonin for osteoporosis. The patient should be monitored for? A. Hyperkalemia B. Hypokalemia C. Hypocalcemia D. Hypercalcemia

The answer is C. Calcitonin is made from salmon calcitonin and acts like the hormone calcitonin which is produced naturally by the thyroid gland. It decreases osteoclast activity, which can decrease calcium levels. The

A patient is taking Calcitonin for osteoporosis. The patient should be monitored for? A. Hyperkalemia B. Hypokalemia C. Hypocalcemia D. Hypercalcemia

The answer is C. Calcitonin is made from salmon calcitonin and acts like the hormone calcitonin which is produced naturally by the thyroid gland. It decreases osteoclast activity, which can decrease calcium levels. Therefore, the patient is at risk HYPOcalcemia. (TONIN IT DOWN)

A patient newly diagnosed with osteoarthritis asks about the medication treatments for their condition. Which medication is NOT typically prescribed for OA?* A. NSAIDs B. Topical Creams C. Oral corticosteroids D. Acetaminophen (Tylenol)

The answer is C. Intra-articular corticosteroids (an injection in the joint) are commonly prescribed rather than oral corticosteroids. Remember OA in within the joint...not systemic so oral corticosteroids are not as effective. All the other medications listed are prescribed in OA.

An 18 year old patient is admitted with appendicitis. Which statement by the patient requires immediate nursing intervention? "The pain hurts so much it is making me nauseous." "I have no appetite.". "The pain seems to be gone now." "If I position myself on my right side, it makes the pain less intense."

The answer is C. It is important that the nurse monitors the patient's pain level. If the patient reports that the pain has suddenly decreased or is gone, this is a warning sign that the appendix may have perforated (ruptured). If the appendix has ruptured, the sudden decrease in pain will be followed by more pain due to peritonitis (which is life- threatening). Therefore, the nurse should notify the doctor immediately.

8. During discharge teaching to a patient at risk for developing osteoporosis, you discuss the types of exercise the patient should perform. Which type of exercise is not the best to perform to prevent osteoporosis? A. Tennis B. Weight-lifting C. Walking D. Hiking

The answer is C. Low-impact exercises are not as beneficial in building bone mass as compared to weight-bearing exercises such as tennis, lifting weights, and hiking etc. The patient should perform exercises that put stress on the bones against gravity, which will help increase bone strength and build muscle.

During discharge teaching to a patient at risk for developing osteoporosis, you discuss the types of exercise the patient should perform. Which type of exercise is not the best to perform to prevent osteoporosis?* A. Tennis B. Weight-lifting C. Walking D. Hiking

The answer is C. Low-impact exercises are not as beneficial in building bone mass as compared to weight-bearing exercises such as tennis, lifting weights, and hiking etc. The patient should perform exercises that put stress on the bones against gravity, which will help increase bone strength and build muscle.

Identify the correct sequence in how rheumatoid arthritis develops: A. Development of pannus, synovitis, ankylosis B. Anklyosis, development of pannus, synovitis C. Synovitis, development of pannus, anklyosis D. Synovitis, anklyosis, development of pannus

The answer is C. The body attacks (specifically the WBCs) the synovium of the joint. The synovium becomes inflamed and this process is called synovitis. The inflammation of the synovium leads to thickening and the formation of a pannus, which is a layer of vascular fibrous tissue. The pannus will grow so large it will damage the bone and cartilage within the joint. The space in between the joints will disappear and anklyosis will develop, which is the fusion of the bone.

During the 1000 medication pass, your patient reports to you that he is having muscle pain and tingling in his fingers and toes. You note that the patient also has a grayish color to his lips. You immediately notify the doctor. In addition, you would hold which medication that is scheduled to be administered at 1000?* A. Ibuprofen B. Prednisone C. Colchicine D. Aspirin

The answer is C. The signs and symptoms presenting in this patient are classic signs of Colchicine toxicity. Therefore, the nurse should not administer Colchicine. This medication can also cause GI upset and neutropenia

It is important a patient with gout avoid medications that can increase uric acid levels. Which medication below is NOT known to increase uric acid levels?* A. Aspirin B. Niacin C. Cyclosporine D. Tylenol

The answer is D. Options A, B, and C increase uric acid. Option D does not.

3. ________ are found within the spongy bone and are responsible for building up the bone matrix. While ________, which are also found in the spongy bone, breakdown the bone matrix. A. Osteocytes, osteoclasts B. Osteoclasts, osteoblasts C. Osteocytes, osteoblasts D. Osteoblasts, osteoclasts

The answer is D. OsteoBLASTS are found within the spongy bone and are responsible for building up the bone matrix, while osteoCLASTS, which are also found in the spongy bone as well, breakdown the bone matrix.

A 75 year old male is admitted for chronic renal failure. You note that the patient has white/yellowish nodules on the helix of the ear and fingers. The patient reports they are not painful. As you document your nursing assessment findings, you will document this finding as? A. Nodosa B. Keloid C. Dermoid D. Tophi

The answer is D. Tophi are white/yellowish nodules that are urate crystals. They start to form together in large masses and can be found under the skin (helix ears, elbows, fingers, toes etc.), joints, bursae, bones, which can lead to bone deformity and joint damage. Patients with chronic renal failure are at risk for chronic gout due to the kidney's inability to remove uric acid remove the body.

A 75 year old male is admitted for chronic renal failure. You note that the patient has white/yellowish nodules on the helix of the ear and fingers. The patient reports they are not painful. As you document your nursing assessment findings, you will document this finding as?* A. Nodosa B. Keloid C. Dermoid D. Tophi

The answer is D. Tophi are white/yellowish nodules that are urate crystals. They start to form together in large masses and can be found under the skin (helix ears, elbows, fingers, toes etc.), joints, bursae, bones, which can lead to bone deformity and joint damage. Patients with chronic renal failure are at risk for chronic gout due to the kidney's inability to remove uric acid remove the body.

You're caring for a patient who has a health history of severe osteoporosis. On assessment you note the patient has severe kyphosis of the upper back. Which nursing diagnosis takes priority for this patient's care? A. Risk for skin breakdown B. Knowledge deficient regarding disease process C. Limited mobility D. Risk for falls

The answer is D. When assessing the options you want to select the option that is a priority for this patient and risk for falls is the priority. The patient is at risk for falls due to severe kyphosis, which is common in severe osteoporosis (also called Dowager's Hump). This deformity of the spine limits mobility and increases the chances of falls In addition, it is important the nurse takes precautions in preventing falls because the patient will most likely experience a fracture due to severe osteoporosis.

You're caring for a patient who has a health history of severe osteoporosis. On assessment you note the patient has severe kyphosis of the upper back. Which nursing diagnosis takes priority for this patient's care?* A. Risk for skin breakdown B. Knowledge deficient regarding disease process C. Limited mobility D. Risk for falls

The answer is D. When assessing the options you want to select the option that is a priority for this patient and risk for falls is the priority. The patient is at risk for falls due to severe kyphosis, which is common in severe osteoporosis (also called Dowager's Hump). This deformity of the spine limits mobility and increases the chances of falls In addition, it is important the nurse takes precautions in preventing falls because the patient will most likely experience a fracture due to severe osteoporosis.

True or False: Osteoarthritis develops due to the deterioration of the synovium within the joint that can lead to complete bone fusion.*

The answer is FALSE: Osteoarthritis is the most common type of arthritis that develops due to the deterioration of the HYALINE CARTILAGE (not synovium) of the bone. This can lead to bone break down, sclerosis of the bone, and osteophytes formation (bone spurs).

True or False: Osteoporosis is a disease process that results in the thinning of the matrix of pore-like structures within the compact bone.

The answer is FALSE: Osteoporosis is a disease process that results in the thinning of the matrix of pore-like structures within the SPONGY (not compact) bone. The compact bone is the outside part of the bone, and the spongy bone is found inside the compact bone. It contains a matrix of pore-like components such as protein and minerals...this starts to thin and becomes more porous in osteoporosis.

True or False: Osteoporosis is a disease process that results in the thinning of the matrix of pore-like structures within the compact bone.*

The answer is FALSE: Osteoporosis is a disease process that results in the thinning of the matrix of pore-like structures within the SPONGY (not compact) bone. The compact bone is the outside part of the bone, and the spongy bone is found inside the compact bone. It contains a matrix of pore-like components such as protein and minerals...this starts to thin and becomes more porous in osteoporosis.

Which patient below is NOT at risk for osteoporosis? A. A 50 year old female whose last menstrual period was 7 years ago. B. A 45 year old male patient who has been taking glucocorticoids for the last 6 months. C. A 30 year old male who drinks alcohol occasionally and has a BMI of 28. D. A 35 year old female who has a history of seizures and takes Dilantin regularly.

The answer is: C. All these patients are at risk for osteoporosis except the patient in option C. Remember the risk factors include: older age (45+), being a woman, Caucasian or Asian, post-menopause, glucocorticoids therapy, anticonvulsants (Dilantin), REGULAR alcohol usage, smoking, sedentary lifestyle, BMI <19, family history. Option C is not at risk.

Which patient below is NOT at risk for osteoporosis? A. A 50 year old female whose last menstrual period was 7 years ago. B. A 45 year old male patient who has been taking glucocorticoids for the last 6 months. C. A 30 year old male who drinks alcohol occasionally and has a BMI of 28. D. A 35 year old female who has a history of seizures and takes Dilantin regularly.

The answer is: C. All these patients are at risk for osteoporosis except the patient in option C. Remember the risk factors include: older age (45+), being a woman, Caucasian or Asian, post-menopause, glucocorticoids therapy, anticonvulsants (Dilantin), REGULAR alcohol usage, smoking, sedentary lifestyle, BMI <19, family history. Option C is not at risk.

Which patient below is NOT at risk for osteoporosis?* A. A 50 year old female whose last menstrual period was 7 years ago. B. A 45 year old male patient who has been taking glucocorticoids for the last 6 months. C. A 30 year old male who drinks alcohol occasionally and has a BMI of 28. D. A 35 year old female who has a history of seizures and takes Dilantin regularly.

The answer is: C. All these patients are at risk for osteoporosis except the patient in option C. Remember the risk factors include: older age (45+), being a woman, Caucasian or Asian, post-menopause, glucocorticoids therapy, anticonvulsants (Dilantin), REGULAR alcohol usage, smoking, sedentary lifestyle, BMI <19, family history. Option C is not at risk.

Which patients below are at risk for developing osteoarthritis? Select-all-that-apply:* A. A 65 year old male with a BMI of 35. B. A 59 year old female with a history of taking long term doses of corticosteroids. C. A 55 year old male with a history of repeated right knee injuries. D. A 60 year old female with high uric acid levels.

The answers are A and C. The risk factors for developing OA include: older age, being overweight (BMI >25), repeated injuries to the weight bearing joints, genetics. Option B is at risk for osteoporosis, and option D is at risk for gout.

A patient is ordered by the physician to take Allopurinol (Zyloprim) for treatment of gout. You've provided education to the patient about this medication. Which statement by the patient requires you to re-educate them about this medication?* A. "This medication will help relieve the inflammation and pain during an acute attack." B. "It is important I have regular eye exams while taking this medication." C. "I will not take large doses of vitamin C supplements while taking this medication." D. "Allopurinol decreases the production of uric acid."

The answers are A, C, E. These options are correct nursing intervention for this patient. Option B is wrong because the patient should consume food LOW in purines (remember purines increase uric acid levels). Option D is wrong because patients should AVOID aspirin. Aspirin (even low doses) increase uric acid levels.

You're developing a nursing care plan for a patient with gout present in the right foot. What specific nursing interventions will you include in this patient's plan of care? Select all that apply:* A. Encourage fluid intake of 2-3 liter per day. B. Provide patient with foods high in purine with each meal daily. C. Place patient's right foot in a foot board while patient is in bed. D. Administer PRN dose of Aspirin for a pain rating greater than 5 on 1-10 scale. E. Apply alternating cold and warm compresses to right foot as tolerated by the patient daily.

The answers are A, C, E. These options are correct nursing intervention for this patient. Option B is wrong because the patient should consume food LOW in purines (remember purines increase uric acid levels). Option D is wrong because patients should AVOID aspirin. Aspirin (even low doses) increase uric acid levels.

During a home health visit you are helping a patient with gout identify foods in their pantry they should avoid eating. Select all the foods below the patient should avoid: A. Sardines B. Whole wheat bread C. Chicken Livers D. Crackers E. Craft beer F. Bananas

The answers are A, C, and E. A patient with gout should avoid foods high in PURINES. These include most red meats, organ meats (liver, kidneys), and alcohol (especially beer).

During a home health visit you are helping a patient with gout identify foods in their pantry they should avoid eating. Select all the foods below the patient should avoid:* A. Sardines B. Whole wheat bread C. Sweetbreads D. Crackers E. Craft beer F. Bananas

The answers are A, C, and E. A patient with gout should avoid foods high in PURINES. These include most red meats, organ meats (liver, kidneys, sweetbreads), alcohol (especially beer).

A physician suspects a patient may have rheumatoid arthritis due to the patient's presenting symptoms. What diagnostic testing can be ordered to help a physician diagnose rheumatoid arthritis? Select all that apply: A. Rheumatoid factor B. Uric acid level C. Erythrocyte sedimentation D. Dexa-Scan E. X-ray imaging

The answers are A, C, and E. These are diagnostic tests to help diagnose RA. Option B is used in gout, and option D is used with osteoporosis.

During an outpatient visit you are assessing the patient's understanding about the signs and symptoms associated with osteoporosis. Select all of the signs and symptoms stated by the patient that are correct: A. Dowager's Hump B. Loss of 0.5 inches in height compared to young adult height C. Swelling and warmth at the bone site D. Some patients are asymptomatic E. Fractures most commonly in the hips, wrist, and spine

The answers are A, D, and E. Option B is wrong because there is normally a loss of 2-3 inches in height compared to the patient's height in young adulthood. Option C is wrong because the bone site will not present as warm or swollen (most patients are asymptomatic).

During an outpatient visit you are assessing the patient's understanding about the signs and symptoms associated with osteoporosis. Select all of the signs and symptoms stated by the patient that are correct:* A. Dowager's Hump B. Loss of 0.5 inches in height compared to young adult height C. Swelling and warmth at the bone site D. Some patients are asymptomatic E. Fractures most commonly in the hips, wrist, and spine

The answers are A, D, and E. Option B is wrong because there is normally a loss of 2-3 inches in height compared to the patient's height in young adulthood. Option C is wrong because the bone site will not present as warm or swollen (most patients are asymptomatic).

1. During a routine health check-up visit a patient states, "I've been experiencing severe pain and stiffness in my joints lately." As the nurse, you will ask the patient what questions to assess for other possible signs and symptoms of rheumatoid arthritis? Select-all-that-apply:* A. "Does the pain and stiffness tend to be the worst before bedtime?" B. "Are you experiencing fatigue and fever as well?" C. "Is your pain and stiffness symmetrical on the body?" D. "Is your pain and stiffness aggravated by extreme temperature changes?"

The answers are B and C. Patients with RA will experience pain and stiffness in the morning (for more than 30 minutes) not bedtime. It is common for patients to have a fever and be fatigued...remember RA affects the whole body not just the joints. It will also affect the same joints on the opposite side of the body. Therefore, if the right wrist is inflamed, painful, and stiff the left wrist will be as well. RA is NOT aggravated by extreme temperatures. This is found in osteoarthritis.

You receive your patient back from radiology. The patient had an x-ray of the hips and knees for the evaluation of possible osteoarthritis. What findings would appear on the x-ray if osteoarthritis was present? Select-all-that-apply: A. Increased joint space B. Osteophytes C. Sclerosis of the bone D. Abnormal sites of hyaline cartilage

The answers are B and C. The joint space would be DECREASED not increased in OA. In addition, an x-ray cannot show hyaline cartilage...therefore, the cartilage cannot be assessed on an x-ray. The radiologist would be looking for osteophytes (bone spurs), sclerosis of the bone (abnormal hardening of the bones), and decreased joint space.

A 58 year old female is experiencing a flare-up with rheumatoid arthritis. While assisting the patient with her morning routine, the patient verbalizes a pain rating of 7 on 1-10 scale in the right and left wrist along with severe stiffness. You note the wrist joints to be red, warm, and swollen. What nonpharmalogical nursing interventions can you provide to this patient to help alleviate pain and stiffness? Select-all-that-apply: A. Exercise the affected joints B. Assist the patient with a warm shower or bath C. Perform deep massage therapy to the wrist joints D. Assist the patient with applying wrist splints

The answers are B and D. During flare-ups of RA the joint should be rested (not exercised) and should not be deep massaged because this can further damage the joint (in addition cause the patient more pain). Heat therapy, like a warm shower or bath, will help alleviate the stiffness. Furthermore, cold therapy can be used to reduce the inflammation along with splinting the affected joints to protect and rest them.

A patient with osteoarthritis has finished their first physical therapy session. As the nurse you want to evaluate the patient's understanding of the type of exercises they should be performing regularly at home as self-management. Select all the appropriate types of exercise stated by the patient:* A. Jogging B. Water aerobics C. Weight Lifting D. Tennis E. Walking

The answers are B, C, E. The patient wants to perform exercises that are low impact like: walking, water aerobics, stationary bike riding along with strengthen training (lifting weights: helps strengthen muscles around the joint), ROM: improves the mobility of the joint and decreases stiffness. It is important patients with OA avoid high impact exercises that will increase stress on weight bearing joints such as running/jogging, jump rope, tennis, or any type of exercise with both feet off the ground.

A patient with osteoarthritis is describing their signs and symptoms. Which signs and symptoms below are NOT associated with osteoarthritis? Select-all-that-apply:* A. Morning stiffness greater than 30 minutes B. Experiencing grating during joint movement C. Fever and Anemia D. Symmetrical joint involvement E. Pain and stiffness tends to be worst at the end of the day

The answers are: A, C, D. These options are signs and symptoms found with rheumatoid arthritis NOT osteoarthritis. In OA: morning stiffness is LESS than 30 minutes, it is NOT systemic as RA (so fever and anemia will not be present), and it is asymmetrical (both joints are not involved). Pain and stiffness will actually be worst at the end of the day compared to the beginning due to overuse of the joints.

2. Select all the following options that are NOT causes of appendicitis: Fecalith Routine usage of NSAIDs Infection due to Helicobacter pylori Lymph node enlargement due to viral or bacterial infection Diet low in fiber

The answers are: B, C, and E. These options are NOT causes of appendicitis. Routine usage of NSAIDS and infection due H. pylori are causes of peptic ulcers. While a diet low in fiber is thought to be the cause of diverticulosis. Fecalith and lymph node enlargement due to viral or bacterial infection (such as mononucleosis etc.) can cause appendicitis.

Which of the following are the major hormonal regulators of calcium homeostasis? Select all that apply. Parathormone Calcitonin Thyroid Cortisol Growth hormone

The balance between bone resorption (removal or destruction) and formation is influenced by the following factors: physical activity; dietary intake of certain nutrients, especially calcium; and several hormones, including calcitriol (i.e., activated vitamin D), parathyroid hormone (PTH), calcitonin, thyroid hormone, cortisol, growth hormone, and the sex hormones estrogen and testosterone PTH AND CALCITONIN REGULATE CALCIUM HOMEOSTASIS!!!

A nurse is explaining a client's decreasing bone density in terms of the balance between bone resorption and formation. What dietary nutrients and hormones play a role in the resorption and formation of adult bones? Select all that apply. Thyroid hormone Growth hormone Estrogen Vitamin B12 Luteinizing hormone

The balance between bone resorption and formation is influenced by the following factors: physical activity; dietary intake of certain nutrients, especially calcium; and several hormones, including calcitriol (i.e., activated vitamin D), parathyroid hormone (PTH), calcitonin, thyroid hormone, cortisol, growth hormone, and the sex hormones estrogen and testosterone. Luteinizing hormone and vitamin B12 do not play a role in bone formation or resorption.

B Clinical manifestations of hip fracture include external rotation, muscle spasm, shortening of the affected extremity, and severe pain and tenderness in the region of the fracture site. Expected clinical manifestations of Parkinson's disease include a stooped posture, shuffling gait, and slow movements. An abrasion is a soft tissue injury. Mild pain and minimal swelling may occur with a sprain or strain.

The home care nurse visits a 74-yr-old man diagnosed with Parkinson's disease who fell while walking this morning. What observation is of most concern to the nurse? 2 × 6 cm right calf abrasion with sanguineous drainage Left leg externally rotated and shorter than the right leg Stooped posture with a shuffling gait and slow movements Mild pain and minimal swelling of the right ankle and foot

B The most common reason for knee arthroplasty is debilitating joint pain despite exercise, weight management, and drug therapy. Recent knee trauma, repeated knee infections, and onset of frozen knee joint are not primary indicators for a knee arthroplasty.

The nurse completes an admission history for a 73-yr-old man with osteoarthritis scheduled for total knee arthroplasty. Which response is expected when asking the patient the reason for admission? Recent knee trauma Debilitating joint pain Repeated knee infections Onset of frozen knee joint

A The patient is encouraged to engage in progressive leg exercises until 90-degree flexion is possible; continuous passive motion also may be used based on surgeon preference. Early ambulation is implemented, sometimes the day of surgery, but orders are likely to indicate weight bearing as tolerated rather than full weight bearing. Immobilization and bed rest are not indicated. The patient's knee is unlikely to dislocate.

The nurse is caring for a 76-yr-old man who has undergone left total knee arthroplasty to relieve the pain of severe osteoarthritis. What care would be expected postoperatively? Progressive leg exercises to obtain 90-degree flexion Early ambulation with full weight bearing on the left leg Bed rest for 3 days with the left leg immobilized in extension Immobilization of the left knee in 30-degree flexion to prevent dislocation

D The LPN/LVN can monitor pain intensity and administer analgesics. Assessment of skin integrity and determining correct alignment to enhance traction are within the RN scope of practice. Removing weights from the traction should not be delegated or done. Removal of weights can cause muscle spasms and bone misalignment, and should not be delegated or done.

The nurse is caring for a patient placed in Buck's traction before open reduction and internal fixation of a left hip fracture. Which care can be delegated to the LPN/LVN? Assess skin integrity around the traction boot. Determine correct body alignment to enhance traction. Remove weights from traction when turning the patient. Monitor pain intensity and administer prescribed analgesics.

C The patient should not force hip into adduction or internal rotation because these movements could dislocate the hip prosthesis. Sitting with feet flat on the floor (avoiding crossing the legs), using an elevated toilet seat, and notifying future caregivers about the prosthesis indicate understanding of discharge teaching.

The nurse is completing discharge teaching with an 80-yr-old male patient who is recovering from a right total hip arthroplasty by posterior approach. Which patient action indicates further instruction is needed? Uses an elevated toilet seat Sits with feet flat on the floor Maintains hip in adduction and internal rotation Verifies need to notify future caregivers about the prosthesis

A Arteriography determines viable tissue for salvage based on blood flow observed in real time and is considered the gold standard for evaluating arterial perfusion. Only arteriography determines where tissue perfusion stops and amputation needs to occur. Bilateral peripheral pulse assessment and areas of black, indurated, cold, and pale skin indicate ischemia.

The patient has frostbite on the distal toes of both feet. The patient is scheduled for amputation of damaged tissue. Which assessment finding or diagnostic study is most objective in determining tissue viability? Arteriogram showing blood vessels Peripheral pulse palpation bilaterally Patches of black, indurated, cold tissue Bilateral pale, cool skin below the ankles

A 25-year-old patient sustains a fractured hip and experiences hypoxemia, dyspnea, and tachypnea. What complication due to fracture should the nurse suspect from these findings?

The patient may have fat embolism syndrome (FES).

A nurse is providing education about fibromyalgia to a group of new nurses. When discussing the pathophysiology of this​ disorder, which systems of the body that are involved with fibromyalgia does the nurse need to​ address? ​(Select all that​ apply.) The endocrine system The cardiac system The renal system The somatic peripheral nervous system The autonomic nervous system

The somatic peripheral nervous system The autonomic nervous system The endocrine system The pathophysiology of fibromyalgia involves the autonomic nervous​ system, somatic peripheral nervous​ system, and endocrine system. The pathophysiology of fibromyalgia does not involve the renal or cardiac systems.

A nurse is performing an admission health history and physical assessment for a client who has severe rheumatoid arthritis. When assessing the client's hands, the nurse identifies that they are similar to the hand in the illustration. What should the nurse document in the medical record when describing this typical physiologic change associated with rheumatoid arthritis?

Ulnar drift

11. A patient with osteoporosis moves slowly and carefully with voluntary restriction of movement. The lower thoracic area is tender on palpation. How does the nurse interpret this assessment data? a. Vertebral compression fracture b. Kyphosis of the dorsal spine c. Osteopenia related to immobility d. Increased osteoblastic activity

Vertebral compression fracture

What occurrence usually precedes general symptom manifestations of​ fibromyalgia? Septicemia with Group A Streptococcus infection Bacterial infection of affected joint Viral illness Recent injury to joint or bone

Viral illness The precise etiology and precipitating factors for fibromyalgia are not​ known, however there is no correlation between Strep A​ infection, mechanical injury to bones or local bacterial infections and the onset of symptoms. Affected individuals often report​ viral-like illness prior to onset of symptoms.

A nurse is providing a class on osteoporosis at the local seniors' center. Which of the following statements related to osteoporosis is most accurate? Osteoporosis is categorized as a disease of the elderly. A nonmodifiable risk factor for osteoporosis is a person's level of activity. Secondary osteoporosis occurs in women after menopause. Slow discontinuation of corticosteroid therapy can halt the progression of the osteoporosis.

When corticosteroid therapy is discontinued, the progression of osteoporosis is halted, but restoration of lost bone mass does not occur. Osteoporosis is not a disease of the elderly because its onset occurs earlier in life, when bone mass peaks and then begins to decline. A person's level of physical activity is a modifiable factor that influences peak bone mass. Lack of activity increases the risk for the development of osteoporosis. Primary osteoporosis occurs in women after menopause.

A client has been prescribed alendronate for the prevention of osteoporosis. Which is the highest priority nursing intervention associated with the administration of the medication? Ensure adequate intake of vitamin D in the diet Assess for the use of corticosteroids Encourage the client to get yearly dental exams Have the client sit upright for at least 30 minutes following administration

While all interventions are appropriate, the highest priority is having the client sit upright for 60 minutes following administration of the medication. This will prevent irritation and potential ulceration of the esophagus. The client should have adequate intake of vitamin D and obtain yearly dental exams. The concurrent use of corticosteroids and alendronate is link to a complication of osteonecrosis.

What antibiotics can be used (trick question)

a variety, depending on the microorganism

On a visit to the family physician, a client is diagnosed with a bunion on the lateral side of the great toe, at the metatarsophalangeal joint. Which statement should the nurse include in the teaching session? a) "Some bunions are congenital; others are caused by wearing shoes that are too short or narrow." b) "Bunions are congenital and can't be prevented." c) "Bunions are caused by a metabolic condition called gout." d) "Bunions may result from wearing shoes that are too big, causing friction when the shoes slip back and forth."

a) "Some bunions are congenital; others are caused by wearing shoes that are too short or narrow." Explanation: Bunions may be congenital or may be acquired by wearing shoes that are too short or narrow, which increases pressure on the bursa at the metatarsophalangeal joint. Acquired bunions can be prevented. Wearing shoes that are too big may cause other types of foot trauma but not bunions. Gout doesn't cause bunions. Although a client with gout may have pain in the big toe, such pain doesn't result from a bunion. pg.1140

When performing a physical assessment of a client, the client reports numbness, tingling, and pain when the nurse percusses lightly over the median nerve. The nurse recognizes that this finding is consistent with: a) Carpal tunnel syndrome b) Dupuytren's contracture c) Impingement syndrome d) Morton's neuroma

a) Carpal tunnel syndrome Explanation: Tinel's sign (numbness, tingling, and pain in response to light percussion over the median nerve) is a positive finding for carpal tunnel syndrome. pg.1136

When an infection is blood borne the manifestations include which of the following symptoms? a) Chills b) Bradycardia c) Hypothermia d) Hyperactivity

a) Chills Explanation: Manifestations include chills, high fever, rapid pulse, and generalized malaise. pg.1148

A client seeks medical attention for a ganglion. Which statement about this musculoskeletal mass is true? a) Dorsiflexion exacerbates signs and symptoms of a ganglion. b) Surgical excision is the treatment of choice for a ganglion. c) A ganglion is a precursor to a primary bone tumor. d) A ganglion is the most common benign soft-tissue mass in the foot.

a) Dorsiflexion exacerbates signs and symptoms of a ganglion. Explanation: Dorsiflexion exacerbates signs and symptoms of a ganglion. A ganglion is the most common benign soft-tissue mass in the hand, not foot. It isn't a known precursor to a primary bone tumor. To treat a ganglion, the physician aspirates the ganglion, then injects a corticosteroid into the joint; the physician may also order nonsteroidal anti-inflammatory agents. Surgical excision is necessary only if signs and symptoms persist and the client's range of motion is impaired. pg.1137

Which is a flexion deformity caused by a slowly progressive contracture of the palmar fascia? a) Dupuytren's contracture b) Callus c) Hammertoe d) Hallux valgus

a) Dupuytren's contracture Explanation: Dupuytren's disease results in a slowly progressive contracture of the palmar fascia, called Dupuytren's contracture. A callus is a discretely thickened area of skin that has been exposed to persistent pressure or friction. A hammertoe is a flexion deformity of the interphalangeal joint, which may involve several toes. Hallux valgus is a deformity in which the great toe deviates laterally. pg.1137

Which of the following is a flexion deformity caused by a slowly progressive contracture of the palmar fascia? a) Dupuytren's contracture b) Callus c) Hallux valgus d) Hammertoe

a) Dupuytren's contracture Explanation: Dupuytren's disease results in a slowly progressive contracture of the palmar fascia, called Dupuytren's contracture. A callus is a discretely thickened area of skin that has been exposed to persistent pressure or friction. A hammertoe is a flexion deformity of the interphangeal joint, which may involve several toes. Hallux valgus is a deformity in which the great toe deviates laterally. pg.1137

Dupuytren's contracture causes flexion of which area(s)? a) Fourth and fifth fingers b) Ring finger c) Thumb d) Index and middle fingers

a) Fourth and fifth fingers Explanation: Dupuytren's contracture causes flexion of the fourth and fifth fingers, and frequently the middle finger. pg.1137

Which of the following was formerly called a bunion? a) Hallux valgus b) Ganglion c) Morton's neuroma d) Plantar fasciitis

a) Hallux valgus Explanation: Hallux valgus (commonly called a bunion) is a deformity in which the great toe deviates laterally. Plantar fasciitis, an inflammation of the foot-supporting fascia, present as an acute onset of heel pain experienced with the first steps in the morning. Morton's neuroma is a swelling of the third (lateral) branch of the median plantar nerve. A ganglion, a collection of gelatinous material near the tendon sheaths and joints, appears as a round, firm compressible cystic swelling, usually on the dorsum of the wrist. pg. 1140

The client presents to the emergency department with fever, chills, restlessness, and limited movement of a fractured jaw. The nurse interprets these findings as indicating which of the following complications? a) Osteomyelitis b) Avascular necrosis c) Fat embolism d) Compartment syndrome

a) Osteomyelitis Explanation: Clinical manifestations of osteomyelitis include signs and symptoms of sepsis and localized infection. pg.1148

Treatment of metastatic bone cancer includes which of the following? a) Palliation b) Radiation c) Combination chemotherapy and radiation d) Chemotherapy

a) Palliation Explanation: The treatment of metastatic bone cancer is palliative. The therapeutic goal is to relieve the patient's pain and discomfort while promoting quality of life. pg.1152

The nurse recognizes that goal of treatment for metastatic bone cancer is to: a) Promote pain relief and quality of life b) Cure the diseased bone and cartilage c) Diagnose the extent of bone damage d) Reconstruct the bone with a prosthesis

a) Promote pain relief and quality of life Explanation: Treatment of metastatic bone cancer is palliative. pg.1152

Ms. Malcolm has come to your clinic complaining of jaw pain. This is also associated with muscle spasm and tenderness of the masseter and temporalis muscles. The physician has diagnosed a temporomandibular disorder (TMD). What would the treatment course for this client include? Select all that apply. a) Referral to a dentist who has experience managing clients with TMD b) Analgesics c) Custom-fitted mouth guard during sleep d) Corticosteroids

a) Referral to a dentist who has experience managing clients with TMD b) Analgesics c) Custom-fitted mouth guard during sleep Explanation: Referral to a dentist who has experience managing clients with TMD, analgesics, and custom-fitted mouth guard during sleep are all part of the treatment course. Corticosteroids are not part of the treatment regimen. pg.1241

A client is diagnosed with osteomyelitis. This is most commonly caused by which of the following? a) Staphylococcus aureus b) Psuedomonas aeruginosa c) Escherichia coli d) Proteus vulgaris

a) Staphylococcus aureus Explanation: Staphylococcus aureus causes over 50% of bone infections. Other organisms include Proteus vulgaris and Pseudomonas aeruginosa, as well as E. coli. pg.1148

The nurse is educating a patient with lower back pain on proper lifting techniques. The nurse would document what behavior as evidence the education was effective? a) The patient placed the load close to the body. b) The patient reached over head with arms fully extended. c) The patient used a narrow base of support. d) The patient bent at the hips and tightened the abdominal muscles.

a) The patient placed the load close to the body. Explanation: Instructions for the patient with low back pain should include that when lifting, the patient should avoid overreaching. The patient should also keep the load close to the body, bend the knees and tighten the abdominal muscles, use a wide base of support, and use a back brace to protect the back. Bending at the hips increases the strain on the back muscles when lifting. pg.1134

A client with low back pain is being seen in the clinic. In planning care, which teaching point should the nurse include? a) Use the large muscles of the leg when lifting items. b) Avoid twisting and flexion activities. c) Sleep on the stomach to alleviate pressure on the back. d) A soft mattress is most supportive by conforming to the body.

a) Use the large muscles of the leg when lifting items. Explanation: The large muscles of the leg should be used when lifting. pg.1134

A nurse is planning discharge teaching regarding exercise for a client at risk for osteoporosis. Which of the following exercises would be appropriate? a) Walking b) Yoga c) Bicycling d) Swimming

a) Walking Explanation: Weight-bearing exercises should be incorporated into the client's lifestyle activities. pg.1141

A patient with osteomyelitis is treated with surgical debridement with implantation of antibiotic beads. When the patient asks why the beads are used, the nurse answers (select all that apply) a."The beads are used to directly deliver antibiotics to the site of the infection." b."There are no effective oral or IV antibiotics to treat most cases of bone infection." c."This is the safest method of delivering long-term antibiotic therapy for a bone infection." d."The beads are an adjunct to debridement and oral and IV antibiotics for deep infections." e."The ischemia and bone death that occur with osteomyelitis are impenetrable to IV antibiotics."

a."The beads are used to directly deliver antibiotics to the site of the infection." d."The beads are an adjunct to debridement and oral and IV antibiotics for deep infections." Treatment of chronic osteomyelitis includes surgical removal of the poorly vascularized tissue and dead bone and the extended use of IV and oral antibiotics. Antibiotic-impregnated polymethylmethacrylate bead chains may be implanted during surgery to aid in combating the infection.

Before discharge from the same-day surgery unit, instruct the patient who has had a surgical correction of bilateral hallux valgus to a.rest frequently with the feet elevated. b.soak the feet in warm water several times a day. c.expect the feet to be numb for the next few days. d.expect continued pain in the feet, since this is not uncommon.

a.rest frequently with the feet elevated. After surgical correction of bilateral hallux valgus, the feet should be elevated with the heel off the bed to help reduce discomfort and prevent edema.

Which of the following inhibits bone resorption and promotes bone formation? a) Estrogen b) Calcitonin c) Corticosteroids d) Parathyroid hormone

b) Calcitonin Explanation: Calcitonin, which inhibits bone resorption and promotes bone formation, is decreased in osteoporosis. Estrogen, which inhibits bone breakdown, decreases with aging. On the other hand, parathyroid hormone (PTH) increases with aging, increasing bone turnover and resorption. The consequence of these changes is net loss of bone mass over time. Corticosteroids place patients as risk for developing osteoporosis. pg.1143

The nurse is assessing the feet of a patient and observes an overgrowth of the horny layer of the epidermis. What does the nurse recognize this condition as? a) Hammer Toe b) Corn c) Clawfoot d) Bunion

b) Corn Explanation: A corn is an area of hyperkeratosis (overgrowth of a horny layer of epidermis) produced by internal pressure (the underlying bone is prominent because of a congenital or acquired abnormality, commonly arthritis) or external pressure (ill-fitting shoes). The fifth toe is most frequently involved, but any toe may be involved. pg.1139

The nurse notes that the patient's left great toe deviates laterally. This finding would be recognized as which of the following? a) Hammertoe b) Hallux valgus c) Pes cavus d) Flatfoot

b) Hallux valgus Explanation: Hallux valgus is commonly referred to as a bunion. Hammertoes are usually pulled upward. Pes cavus refers to a foot with an abnormally high arch and a fixed equinus deformity of the forefoot. In flatfoot, the patient demonstrates a diminished longitudinal arch of the foot. pg.1140

Which is a deformity in which the great toe deviates laterally? a) Hammertoe b) Hallux valgus c) Pes cavus d) Plantar fasciitis

b) Hallux valgus Explanation: Hallux valgus is a deformity in which the great toe deviates laterally. A hammertoe is a flexion deformity of the interphalangeal joint, which may involve several toes. Pes cavus refers to a foot with an abnormally high arch and a fixed equines deformity of the forefoot. Plantar fasciitis is an inflammation of the foot-supporting fascia. pg.1140

During a routine physical examination of a client, the nurse observes a flexion deformity of the promixal interphalangeal (PIP) joint of two toes on the right foot. The nurse documents this finding as which of the following? a) Mallet toe b) Hammer toe c) Bunion d) Hallux valgus

b) Hammer toe Explanation: Hammer toe is a flexion deformity of the proximal interphalangeal (PIP) joint and may involve several toes. Mallet toe is a flexion deformity of the distal interphalangeal joint (DIP), and also can affect several toes. Hallux valgus, also called a bunion, is a deformity of the great (large) toe at its metatarsophalangeal joint. pg.1139

Assessment of a client reveals signs and symptoms of Paget's disease. Which of the following would be most likely? a) Skull narrowing b) Long bone bowing c) Lordosis d) Waddling gait

b) Long bone bowing Explanation: Some clients with Paget's disease are asymptomatic with only some mild skeletal deformity. Other clients have marked skeletal deformities which may include enlargement of the skull, bowing of the long bones, and kyphosis. Waddling gait is associated with osteomalacia. pg.1146

Which of the following should be included in the teaching plan for a patient diagnosed with plantar fasciitis? a) The pain of plantar fasciitis diminishes with warm water soaks. b) Management of plantar fasciitis includes stretching exercises. c) Plantar fasciitis presents as an acute onset of pain localized to the ball of the foot that occurs when pressure is placed upon it and diminishes when pressure is released. d) Complications of plantar fasciitis include neuromuscular damage and decreased ankle range of motion.

b) Management of plantar fasciitis includes stretching exercises. Explanation: Management also includes wearing shoes with support and cushioning to relieve pain, orthotic devices (eg, heel cups, arch supports), and the use of nonsteroidal anti-inflammatory drugs (NSAIDs). Plantar fasciitis, an inflammation of the foot-supporting fascia, presents as an acute onset of heel pain experienced with taking the first steps in the morning. The pain is localized to the anterior medial aspect of the heel and diminishes with gentle stretching of the foot and Achilles tendon. Unresolved plantar fasciitis may progress to fascial tears at the heel and eventual development of heel spurs. pg.1139

A physician diagnoses primary osteoporosis in a client who has lost bone mass. In this metabolic disorder, the rate of bone resorption accelerates while bone formation slows. Primary osteoporosis is most common in: a) young menstruating women. b) elderly postmenopausal women. c) young children. d) elderly men.

b) elderly postmenopausal women. Explanation: Although the cause of primary osteoporosis is unknown, an important contributing factor may be faulty protein metabolism resulting from estrogen deficiency and a sedentary lifestyle. Typically, these conditions occur in elderly postmenopausal women. pg.1143

The nurse is reviewing the medication administration record of the client. Which of the following medications would lead the nurse to suspect that the client is at risk for osteoporosis? a) penicillamine (Cuprimine) b) raloxifene (Evista) c) plicamycin (Mithracin) d) methotrexate (Rheumatrex)

b) raloxifene (Evista) Explanation: Raloxifene (Evista) is used for the prevention and treatment of osteoporosis. pg.1144

5. During a follow-up visit to a patient with acute osteomyelitis treated with IV antibiotics, the home health nurse is told by the patient's wife that she can hardly get the patient to eat because his mouth is so sore. In assessing the patient's mouth, what should the nurse expect to find? a. A dry, cracked tongue with a central furrow b. White, curdlike membranous lesions of the mucosa c. Ulcers of the mouth and lips surrounded by a reddened base d. Single or clustered vesicles on the tongue and buccal mucosa

b. One of the most common adverse effects of prolonged and high-dose antibiotic therapy is overgrowth of Candida albicans in the oral cavity and genitourinary tract. These infections are manifested by whitish-yellow, curdlike lesions of the mucosa. A dry, cracked, furrowed tongue is characteristic of severe dehydration and vesicles are characteristic of herpes simplex infections. Mouth and lip ulcers are characteristic of aphthous somatitis, or canker sores.

In caring for a patient after a spinal fusion, the nurse would immediately report to the physician which patient symptom? a.The patient experiences a single episode of emesis. b.The patient is unable to move the lower extremities. c. The patient is nauseated and has not voided in 4 hours. d. The patient complains of pain at the bone graft donor site.

b.The patient is unable to move the lower extremities. After spinal fusion surgery, the nurse should frequently monitor peripheral neurologic signs. Movement of the arms and legs and assessment of sensation should be unchanged in comparison with the preoperative status. These assessments are usually repeated every 2 to 4 hours during the first 48 hours after surgery, and findings are compared with those of the preoperative assessment. Paresthesias, such as numbness and tingling sensation, may not be relieved immediately after surgery. The nurse should document any new muscle weakness or paresthesias and report this to the surgeon immediately.

A nurse is caring for an elderly female client with osteoporosis. When teaching the client, the nurse should include information about which major complication? a) Negative calcium balance b) Dowager's hump c) Bone fracture d) Loss of estrogen

c) Bone fracture Explanation: Bone fracture is a major complication of osteoporosis; it results when loss of calcium and phosphate increases the fragility of bones. Estrogen deficiencies result from menopause — not osteoporosis. Calcium and vitamin D supplements may be used to support normal bone metabolism, but a negative calcium balance isn't a complication of osteoporosis. Dowager's hump results from bone fractures. It develops when repeated vertebral fractures increase spinal curvature. pg.1144

Which of the following would the nurse use to determine that a client is exhibiting signs and symptoms of chronic osteomyelitis? a) Rapid pulse b) Tenderness over the affected area c) Persistent draining sinus d) High fever

c) Persistent draining sinus Explanation: Persistent draining sinus indicates a chronic infection in a client with osteomyelitis. High fever, rapid pulse, and tenderness or pain over the affected area is evidence of an acute infection. pg.1148

Which of the following presents with an onset of heel pain with the first steps of the morning? a) Morton's neuroma b) Hallux valgus c) Plantar fasciitis d) Ganglion

c) Plantar fasciitis Explanation: Plantar fasciitis, an inflammation of the foot-supporting fascia, present as an acute onset of heal pain experienced with the first steps in the morning. Hallux valgus (commonly called a bunion) is a deformity in which the great toe deviates laterally. Morton's neuroma is a swelling of the third (lateral) branch of the median plantar nerve. A ganglion, a collection of gelatinous material near the tendon sheaths and joints, appears as a round, firm compressible cystic swelling, usually on the dorsum of the wrist. pg.1139

Which of the following is the only selective estrogen receptor modulator approved for osteoporosis in post menopausal women? a) Denosumab b) Forteo c) Raloxifene d) Fosamax

c) Raloxifene Explanation: Raloxifene is the only selective estrogen receptor modulator (SERM) approved for osteoporosis in post menopausal women as it does not increase the risk of breast or uterine cancer, but it does come with an increased risk of thromboembolism. Fosamax is a bisphosphonate. Forteo is a subcutaneously administered medication that is given one daily for the treatment of osteoporosis. Denosumab has recently been approved for treatment of postmenopausal women with osteoporosis who are at risk for fractures. pg.1144

An infection lasting longer than one month or an infection that has failed to respond to the initial course of antibiotic therapy that has continuous and persistent exacerbation and remissions is classified as what?

chronic osteomyelitis

A client with osteoporosis is prescribed a selective estrogen receptor modifier (SERM) as treatment. The nurse would identify which drug as belonging to this class? a) Alendronate (Fosamax) b) Calcium gluconate c) Tamoxifen (Nolvadex) d) Raloxifene (Evista)

d) Raloxifene (Evista) Explanation: An example of a selective estrogen receptor modifier (SERM) is raloxifene (Evista). Alendronate is a biphosphonate; calcium gluconate is an oral calcium preparation; tamoxifen is an antiestrogen agent. pg.1144

In chronic osteomyelitis, antibiotics are adjunctive therapy in which of the following situations? a) Wound irrigation b) Vitamin supplements c) Wound packing d) Surgical debridement

d) Surgical debridement Explanation: In chronic osteomyelitis, antibiotics are adjunctive therapy to surgical debridement. pg.1148

Jennifer Carson is a​ 35-year-old woman who presents at the healthcare​ provider's office. She states that she​ "has been more tired than​ usual" for the past few months and that she has​ "a general​ achiness, especially in my back and​ hips." The healthcare provider is questioning a diagnosis of fibromyalgia and has ordered blood to be drawn for thyroid hormone levels. While preparing to perform the blood​ draw, Ms. Carson​ says, "If there are no tests to determine the diagnosis of​ fibromyalgia, why are they testing my​ thyroid"? What is your most informative​ response" ​"Sometimes the symptoms of fibromyalgia present similar to the symptoms of hypothyroidism. Knowing how your thyroid is functioning will allow for an accurate​ diagnosis." ​"Thyroid problems may precede the onset of fibromyalgia. If your thyroid is not functioning​ properly, it will be important to treat both​ conditions." ​"Checking thyroid function is the first endocrine gland to assess because fibromyalgia often presents with endocrine​ imbalances." ​"If your thyroid is not functioning​ properly, correcting thyroid hormone levels may prevent fibromyalgia from​ developing."

​"Sometimes the symptoms of fibromyalgia present similar to the symptoms of hypothyroidism. Knowing how your thyroid is functioning will allow for an accurate​ diagnosis." Explaining the process of exclusion for other diagnoses is the most informative response. The development and course of fibromyalgia are not dependent on thyroid function

The nursing instructor asks a nursing student to identify risk factors that are shared by clients who have osteoporosis or osteomalacia. Which statement by the student is correct? A. "High alcohol intake is a risk factor for both conditions." B. "A history of smoking is a risk factor for both conditions." C. "Inadequate exposure to sunlight is a risk factor for both conditions." D. "Being homeless is a risk factor for both conditions."

"High alcohol intake is a risk factor for both conditions."

13. A patient is scheduled to have a dual x-ray absorptiometry (DXA). What information does the nurse give to the patient about preparing for the test? a. "Leave metallic objects such as jewelry, coins, and belt buckles at home." b. "Have someone come with you to drive you home after the test." c. "You will be asked to give a urine specimen prior to the test." d. "Bring a comfortable loose nightgown without buttons or snaps."

"Leave metallic objects such as jewelry, coins, and belt buckles at home."

12. The home health nurse is visiting an older adult patient with osteoporosis and severe kyphosis. When the nurse asks about activities she has been doing, the patient replies, "I used to be very active and beautiful when I was younger." What is the nurse's best response? a. "You are still very beautiful." b. "Activity can help to prevent fractures and complications." c. "Tell me what you used to do." d. "Do you need information about ageappropriate exercises?"

"Tell me what you used to do."

What are some of the non-super obvious clinical manifestations of appendicitis?

1. Pain aggravated by walking or moving 2. Coated tongue and bad breath 3. Constipation OR Diarrhea 4. Fever 5. Rovsing's 6. Psoas 7. Obturator

What are the four major complications of appendicitis?

1. Peritonitis - ISOTONIC FLUIDS pushed to combat hypovolemia\ 2. Paralytic Ileus - absent or high pitched bowel sounds; distended abdomen 3. Pyelophlebitis - rare, but lethal inflammation of portal venous system 4. Abcess Formation - Collection of purulent tissue surrounded by inflamed tissues; tx = antibiotics, maybe a drain, then appendectomy

The physician has prescribed plicamycin (Mithracin) to control serum calcium levels in a client with Paget's disease. The dose prescribed is 25 micrograms per kg. The client weighs 132 lbs. How many milligrams will the nurse expect the client to receive?

1.5 Explanation: The client weighs 60 kg (132 lbs/2.2 lbs per kg). The client will receive 1500 micrograms (60 kg x 25 micrograms/kg). 1500 micrograms/1000 micrograms per mg = 1.5 mg. pg.1148

10. What should the nurse teach the patient recovering from an episode of acute low back pain? a. Perform daily exercise as a lifelong routine. b. Sit in a chair with the hips higher than the knees. c. Avoid occupations in which the use of the body is required. d. Sleep on the abdomen or on the back with the legs extended.

10. a. Proper daily exercise is an important part of the prevention of back injury, with the goal of maintaining mobility and strength in the back. Patients should sit with the knees higher than the hips and should sleep in a side-lying position, with knees and hips bent, or on the back, with a device to flex the hips and knees. Good body mechanics with proper transfer and turning techniques are necessary in all jobs and activities.

18. Identify methods to specifically prevent osteoporosis in postmenopausal women (select all that apply). a. Eating more beef b. Eating 8 ounces of yogurt daily c. Performing weight-bearing exercise d. Spending 15 minutes in the sun each day e. Taking postmenopausal estrogen replacement

18. b, c. To specifically prevent osteoporosis in postmenopausal women, increased calcium and vitamin D intake and weight-bearing exercises (i.e., walking) are the best methods. Beef is not high in calcium or vitamin D. Although 20 minutes in the sun each day provides vitamin D for most women, it is recommended that postmenopausal women take supplemental vitamin D doses of 800 to 1000 IU per day. Although estrogen replacement will protect the woman against bone loss and fractures, it is no longer given specifically to prevent osteoporosis because of increased risk of heart disease and breast or uterine cancer.

A football player is admitted to the emergency department with a sprained ankle. What emergency care is needed for the client with this type of injury? 1) Do not apply a splint below the injury. 2) Use elastic wrap for the first 24 to 48 hours. 3) Do not apply a splint above the injury. 4) Apply warm compresses for the first 24 hours.

2) Use elastic wrap for the first 24 to 48 hours. As part of emergency care for a client with sports-related injury, compression measures such as elastic wrap should be used for the first 24 to 48 hours. A splint may be applied above and below the injury to immobilize the joint. Ice is applied intermittently for the first 24 to 48 hours. Later, heat can be used.

Intervention to prevent osteoporosis in women should start at which age?

20-35 years

A client diagnosed with osteoporosis is being discharged home. Which priority education should the nurse should provide? Remove all small rugs from the home Participatie in weight-bearing exercises Classify medications Increase calcium and vitamin D in the diet

A client with osteoporosis is at risk for fractures related to falls. The home environment needs to be evaluated for safety issues, such as rugs and other objects that could cause a fall. All other education is important in educating the client, but the risk for injury from a fall and potential for a fracture makes safety in the home environment a priority.

A Exercises designed to restore strength and muscle tone will be done for months after surgery. The exercises include leg raises in supine and prone positions. Driving a car is not allowed for 4 to 6 weeks. In the posterior approach hip arthroplasties, extremes of internal rotation and 90-degree flexion of the hip must be avoided for 4 to 6 weeks postoperatively. The knees must be kept apart. The patient should never cross the legs or twist to reach behind. To prevent thromboembolism, enoxaparin is administered subcutaneously and can be given at home. Enoxaparin does not require monitoring of the patient's coagulation status.

A nurse performs discharge teaching for a 58-yr-old woman after a left hip arthroplasty using the posterior approach. Which statement by the patient indicates teaching is successful? "Leg-raising exercises are necessary for several months." "I should not try to drive a motor vehicle for 2 to 3 weeks." "I will not have any restrictions now on hip and leg movements." "Blood tests will be done weekly while taking enoxaparin (Lovenox)."

A patient is taking Calcitonin for osteoporosis. The patient should be monitored for?* A. Hyperkalemia B. Hypokalemia C. Hypocalcemia D. Hypercalcemia

A patient is taking Calcitonin for osteoporosis. The patient should be monitored for? A. Hyperkalemia B. Hypokalemia C. Hypocalcemia D. Hypercalcemia The answer is C. Calcitonin is made from salmon calcitonin and acts like the hormone calcitonin which is produced naturally by the thyroid gland. It decreases osteoclast activity, which can decrease calcium levels. Therefore, the patient is at risk HYPOcalcemia.

Which action will the nurse take first when a patient is seen in the outpatient clinic with neck pain? a. Provide information about therapeutic neck exercises. b. Ask about numbness or tingling of the hands and arms. c. Suggest that the patient alternate the use of heat and cold to the neck to treat the pain. d. Teach about the use of nonsteroidal antiinflammatory drugs such as ibuprofen (Advil).

ANS: B The nurse's initial action should be further assessment of the pain because cervical nerve root compression will require different treatment than musculoskeletal neck pain. The other actions may also be appropriate, depending on the assessment findings.

After laminectomy with a spinal fusion to treat a herniated disc, a patient reports numbness and tingling of the right lower leg. The first action the nurse should take is to a. report the patient's complaint to the surgeon. b. check the chart for preoperative assessment data. c. check the vital signs for indications of hemorrhage. d. turn the patient to the left to relieve pressure on the right leg.

ANS: B The postoperative movement and sensation of the extremities should be unchanged (or improved) from the preoperative assessment. If the numbness and tingling are new, this information should be immediately reported to the surgeon. Numbness and tingling are not symptoms associated with hemorrhage at the site. Turning the patient will not relieve the numbness. DIF: Cognitive Level: Analyze (analysis) REF: 1507 OBJ: Special Questions: Prioritization TOP: Nursing Process: Implementation MSC: NCLEX: Physiological Integrity

When administering alendronate (Fosamax) to a patient with osteoporosis, the nurse will a. ask about any leg cramps or hot flashes. b. assist the patient to sit up at the bedside. c. be sure that the patient has recently eaten. d. administer the ordered calcium carbonate.

ANS: B To avoid esophageal erosions, the patient taking bisphosphonates should be upright for at least 30 minutes after taking the medication. Fosamax should be taken on an empty stomach, not after taking other medications or eating. Leg cramps and hot flashes are not side effects of bisphosphonates

The nurse is caring for a patient who has had a surgical reduction of an open fracture of the left tibia. Which assessment finding is most important to report to the health care provider? a. Left leg muscle spasms b. Serous wound drainage c. Left leg pain with movement d. Temperature 101.4° F (38.6° C)

ANS: D An elevated temperature is suggestive of possible osteomyelitis. The other clinical manifestations are typical after a repair of an open fracture.

42. A patient with a bone tumor is grieving and anxious. The nurse includes which psychosocial interventions? (Select all that apply.) a. Allow the patient to verbalize feelings. b. Offer to call the patient's spiritual or religious adviser. c. Prepare the patient for death. d. Share stories of personal losses. e. Redirect the patient to more cheerful topics. f. Listen attentively while the patient talks.

Allow the patient to verbalize feelings. Offer to call the patient's spiritual or religious adviser. Listen attentively while the patient talks.

A Mirror therapy has been shown to reduce phantom limb pain in some patients. Opioid analgesics, rebandaging the residual limb, and showing the patient that the leg is gone may not decrease phantom limb pain.

An injured soldier underwent left leg amputation 2 weeks ago, but now reports shooting pain and heaviness in the left leg. What action by the nurse is supported by research findings? Use mirror therapy. Give opioid analgesics. Rebandage the residual limb. Show the patient the leg is gone.

The nurse is caring for a patient who is to receive a transfusion of two units of packed red blood cells. After obtaining the first unit from the blood bank, the nurse would ask which health team member in the nurses' station to assist in checking the unit before administration? Unit secretary A physician's assistant Another registered nurse An unlicensed assistive personnel

Another registered nurse

Which female patients are at risk for developing osteoporosis (Select all that apply) ? a-60 year old white aerobic instructor. b-55 year old Asian American cigarette smoker. c-62 year old African American on estrogen therapy. d-68 year old white who is underweight and inactive e-58 year old Native American who started menopause prematurely.

Answers are: b,d,e Risk factors for osteoporosis include age greater than 65, white or Asian ethnicity, cigarette smoking, inactive lifestyle, low weight and postmenopausal estrogen deficiency including premature menopause.

Colchicine (Colcrys) Indications Prophylaxis and treatment of acute attacks of gouty arthritis (oral solution is only for prophylaxis). Familial Mediterranean fever. Action Interferes with the functions of WBCs in initiating and perpetuating the inflammatory response to monosodium urate crystals. Contraindication/Precautions Contraindicated in: Hypersensitivity; Use of P-glycoprotein inhibitors or strong CYP3A4 inhibitors in patients with renal or hepatic impairment; Renal and hepatic impairment. Adverse Reactions/Side Effects Derm: alopecia GI: diarrhea, nausea, vomiting, abdominal pain Hemat: AGRANULOCYTOSIS, APLASTIC ANEMIA, leukopenia, thrombocytopenia Neuro: peripheral neuritis Route/Dosage Treatment of Acute Gout Attacks PO (Adults): 1.2 mg initially, then 0.6 mg 1 hr later (maximum dose of 1.8 mg in 1 hr); Concomitant use of strong CYP3A4 inhibitors in patients with normal renal and hepatic function (atazanavir, clarithromycin, darunavir/ritonavir, indinavir, itraconazole, ketoconazole, lopinavir/ritonavir, nefazodone, nelfinavir, ritonavir, saquinavir, tipranavir/ritonavir)- 0.6 mg × 1 dose, then 0.3 mg 1 hr later (do not repeat treatment course for ≥3 days); Concomitant use of moderate CYP3A4 inhibitors (aprepitant, diltiazem, erythromycin, fluconazole, fosamprenavir, grapefruit juice, verapamil)- 1.2 mg × 1 dose (do not repeat for ≥3 days); Concomitant use of P-glycoprotein inhibitors (cyclosporine, ranolazine) in patients with normal renal and hepatic function- 0.6 mg × 1 dose (do not repeat for ≥3 days).

Assessment Monitor intake and output ratios. Fluids should be encouraged to promote a urinary output of at least 2000 mL/day. Gout: Assess involved joints for pain, mobility, and edema throughout therapy. During initiation of therapy, monitor for drug response every 1-2 hr. Implementation Do not confuse colchicine with Cortrosyn. Intermittent therapy with 3 days between courses may be used to decrease risk of toxicity. Patient/Family Teaching Instruct patient to report muscle pain or weakness, tingling or numbness in fingers or toes; pale or gray color to lips, tongue, or palms of hands; severe diarrhea or vomiting; unusual bleeding, bruising, sore throat, fatigue, malaise, or weakness or tiredness promptly. Hold colchicine if symptoms of toxicity occur. Evaluation/Desired Outcomes Decrease in pain and swelling in affected joints within 12 hr.Relief of symptoms within 24-48 hr. Prevention of acute gout attacks. Reduced number of attacks of familial Mediterranean fever.

The nurse knows that a 60-year-old female client's susceptibility to osteoporosis is most likely related to: a-Lack of exercise. b-Hormonal disturbances. c-Lack of calcium. d-Genetic predisposition

B: After menopause, women lack hormones necessary to absorb and utilize calcium. A&C: Doing weight-bearing exercises and taking calcium supplements can help to prevent osteoporosis but are not causes, so answers A and C are incorrect. D: Body types that frequently experience osteoporosis are thin Caucasian females, but they are not most likely related to osteoporosis, so answer D is incorrect.

45. A patient with bone cancer has had the right lower leg surgically removed. The patient has been brave and uncomplaining, but the nurse recognizes that the patient is likely to experience grieving. What is the nurse's most important role? a. Act as a patient advocate to promote the physician-patient relationship. b. Encourage the patient to talk to the family and complete an advance directive. c. Be an active listener and encourage the patient and family to verbalize feelings. d. Help the patient and family cope with and resolve grief and loss issues.

Be an active listener and encourage the patient and family to verbalize feelings.

Which of the following is the first-line medication that would be used to treat and prevent osteoporosis? Bisphosphonates Calcitonin Selective estrogen receptor modulators Anabolic agents

Bisphosphonates, along with calcium and vitamin D supplements, are the first-line medications given to prevent/treat osteoporosis. The other medications are prescribed after these drugs are used.

Alendronate (Fosamax) is given to a client with osteoporosis. The nurse advises the client to? a-Take the medication in the morning with meals. b-Take the medication 2 hours before bedtime. c-Take the medication with a glass of water after rising in the morning. d-Take the medication during lunch.

C: Alendronate needs to be taken with a glass of water after rising in the morning in order to prevent gastrointestinal effects. A, B, D: Not taking Alendronate on the specified time of the day could cause GI upset.

21. A patient is prescribed calcitonin for treatment of Paget's disease. What does the nurse teach this patient regarding this drug? a. Avoid eating salmon because calcitonin is derived from salmon. b. Calcitonin is given by subcutaneous injection. c. Take a drug holiday after 1 year of therapy. d. Store the drug in a cool, dry, dark place.

Calcitonin is given by subcutaneous injection.

Before beginning a transfusion of packed red blood cells (PRBCs), which action by the nurse would be of highest priority to avoid an error during this procedure? Check the identifying information on the unit of blood against the patient's ID bracelet. Select new primary IV tubing primed with lactated Ringer's solution to use for the transfusion. Remain with the patient for 60 minutes after beginning the transfusion to watch for signs of a transfusion reaction. Add the blood transfusion as a secondary line to the existing IV and use the IV controller to maintain correct flow.

Check the identifying information on the unit of blood against the patient's ID bracelet.

47. Which type of benign tumor is commonly located in the hands and feet? a. Chondroma b. Giant cell tumor c. Osteochondroma d. Fibrogenic tumor

Chondroma

37. A patient is being seen in the clinic for dull pain and swelling of the proximal femur over 2-3 months. Which malignant bone tumor might this be? a. Ewing's sarcoma b. Chondrosarcoma c. Fibrosarcoma d. Osteosarcoma

Chondrosarcoma

A student nurse is asking questions about fibromyalgia. The nurse educates the student that this disorder closely resembles what other​ disorder? Osteoarthritis Chronic fatigue syndrome Muscular dystrophy ​Sjögren syndrome

Chronic fatigue syndrome Fibromyalgia closely resembles chronic fatigue​ syndrome, with the exception of the musculoskeletal pain typically associated with fibromyalgia. Fibromyalgia does not closely resemble​ Sjögren syndrome, muscular​ dystrophy, or osteoarthritis.

A client has been admitted to the hospital with a spontaneous vertebral fracture related to osteoporosis. Which of the following nursing diagnoses must be addressed in the plan of care?

Constipation is a problem related to immobility and medications used to treat vertebral fractures. The client's risks of aspiration, dysphagia, and decreased cardiac output are not necessarily heightened.

Postmenopausal women are more prone to suffer from osteoporosis due to?

Decreased estrogen level

18. Which are potential adverse reactions of alendronate (Fosamax)? (Select all that apply.) a. Difficulty swallowing b. Drowsiness c. Esophagitis d. Constipation e. Esophageal ulcers

Difficulty swallowing Esophagitis Esophageal ulcers

The patient with leukemia has acute disseminated intravascular coagulation (DIC) and is bleeding. What diagnostic findings should the nurse expect to find? Elevated D-dimers Elevated fibrinogen Reduced prothrombin time (PT) Reduced fibrin degradation products (FDPs)

Elevated D-dimers

Which areas of counseling are appropriate for the client affected by​ fibromyalgia? ​(Select all that​ apply.) Examples of mild to moderate exercise Nonpharmacologic methods of pain relief The importance of adhering to an​ around-the-clock schedule of narcotic analgesics Strategies for improving quality of sleep Identifying resources and support systems

Examples of mild to moderate exercise Nonpharmacologic methods of pain relief Strategies for improving quality of sleep Identifying resources and support systems Narcotics are not the treatment of choice for fibromyalgia pain as their side effects may lead to tolerance and dependence and worsen other fibromyalgia symptoms such as fatigue and activity intolerance. Nonpharmacologic methods of pain​ control, support​ systems, exercise, and improved sleep patterns are effective at improving symptoms and quality of life.

Which risk factor associated with fibromyalgia should the nurse inquire about while taking a client​'s health​ history? Age​ 60-75 years History of osteoarthritis Family history Male gender

Family history A family history of fibromyalgia is a risk factor for developing fibromyalgia.​ Female, not​ male, gender is a risk factor for developing​ fibromyalgia. Being diagnosed with another rheumatic disorder such as rheumatoid arthritis​ (not osteoarthritis) or systemic lupus erythematosus are risk factors associated with developing fibromyalgia. A risk factor for developing fibromyalgia is an age between 20 and 50 years​ old, not between 60 and 75 years old.

34. The nurse is caring for a patient with acute osteomyelitis. What assessment findings typically accompany this medical diagnosis? (Select all that apply.) a. Fever; temperature usually above 101° F b. Sinus tract formation c. Erythema of the affected area d. Swelling around the affected area e. Decreased peripheral pulses

Fever; temperature usually above 101° F Erythema of the affected area Swelling around the affected area

14. A patient is lactose intolerant and would like suggestions about food sources that supply adequate calcium and vitamin D. In addition to a generally well-balanced diet, what foods does the nurse suggest? a. Fresh apples and pears b. Whole-wheat bread c. Fortified soy or rice products d. Prune or cranberry juice

Fortified soy or rice products

20. The nurse is reviewing the prescriptions for a patient receiving drug therapy for the prevention of osteoporosis. The patient also has hypertension and heart disease. Which prescription order does the nurse question? a. Calcium supplements b. Hormone replacement therapy c. Alendronate (Fosamax) d. Raloxifene (Evista)

Hormone replacement therapy

The nurse is caring for a patient with a pelvic fracture. What nursing assessment for a pelvic fracture should be included? (Select all that apply.) Checking the urine for hematuria Palpating peripheral pulses in both lower extremities Testing the stool for occult blood Assessing level of consciousness Assessing pupillary response

In pelvic fracture, the nurse should palpate the peripheral pulses, especially the dorsalis pedis pulses of both lower extremities; absence of a pulse may indicate a tear in the iliac artery or one of its branches. To assess for urinary tract injury, the patient's urine is analyzed for blood.

Emergency medical technicians transport a client to the emergency department. They tell the nurse, "He fell from a two-story building. He has a large contusion on his left chest and a hematoma in the left parietal area. He has a compound fracture of his left femur and he's comatose. We intubated him and he's maintaining an arterial oxygen saturation of 92% by pulse oximeter with a manual-resuscitation bag." Which intervention by the nurse has the highest priority? Assessing the left leg Assessing the pupils Placing the client in Trendelenburg's position Assessing level of consciousness (LOC)

In this scenario, airway and breathing have been established, so the nurse's priority should be circulation. With a compound fracture of the femur, there is a high risk of profuse bleeding; therefore, the nurse should assess the left leg. Neurologic assessment of the pupils and LOC are secondary concerns to airway, breathing, and circulation. The nurse doesn't have enough data to warrant putting the client in Trendelenburg's position.

The nurse receives a physician's order to transfuse fresh frozen plasma to a patient with acute blood loss. Which procedure is most appropriate for infusing this blood product? Infuse the fresh frozen plasma as rapidly as the patient will tolerate. Hang the fresh frozen plasma as a piggyback to the primary IV solution. Infuse the fresh frozen plasma as a piggyback to a primary solution of normal saline. Hang the fresh frozen plasma as a piggyback to a new bag of primary IV solution without KCl.

Infuse the fresh frozen plasma as rapidly as the patient will tolerate.

39. Which radiographic findings are associated with benign bone tumor growth? (Select all that apply.) a. Intact cortices b. Cortical breakthrough c. Smooth uniform periosteal bone d. Sharp margins e. Irregular new periosteal bone

Intact cortices Smooth uniform periosteal bone Sharp margins

16. Calcitonin (Calcimar) has been prescribed to a patient. What instructions does the nurse provide about this medication? (Select all that apply.) a. It is a parathyroid hormone that will increase bone density. b. It will help prevent bone loss. c. It will stimulate the production of bone cells. d. Flushing, nausea, and skin rash may be side effects. e. Nasal mucosa irritation may occur if taken intranasally.

It will help prevent bone loss. Flushing, nausea, and skin rash may be side effects.

52. Which characteristics describe a ganglion hand disorder? (Select all that apply.) a. Joint discomfort after strain b. Progressive palmar flexion deformity c. Surgical release is required d. Round cyst-like lesion e. Painless on palpation f. Fourth and fifth digits affected

Joint discomfort after strain Round cyst-like lesion Painless on palpation

The nurse is instructing a patient on foods high in calcium. The nurse knows the teaching was effective when the patient chooses which foods for a meal? Select all that apply. 1. Chicken 2. Kale 3. Bananas 4. Salmon 5. Low-fat milk

Kale, Salmon and Milk are foods high in calcium, Chicken is high in protein Bananas are high in potassium

The nurse knows that hemolytic anemia can be caused by which extrinsic factors? Trauma or splenic sequestration crisis Abnormal hemoglobin or enzyme deficiency Macroangiopathic or microangiopathic factors Chronic diseases or medications and chemicals

Macroangiopathic or microangiopathic factors

43. A patient being evaluated for bone pain has a computed tomography (CT) report that includes a large tumor with a sclerotic center, periphery is soft, and extends through the bone cortex in a sunburst pattern. What do these findings indicate? a. Malignant bone tumor such as osteosarcoma b. Benign bone tumor such as osteochondroma c. Advanced Paget's disease d. Osteomalacia with osteoporosis

Malignant bone tumor such as osteosarcoma

46. Which definition best describes secondary tumors? a. Tumors arising from bones b. Malignant tumor metastasizing to bone c. Malignant tumor arising from underlying tissue d. Tumor arising from cartilage

Malignant tumor metastasizing to bone

A client with osteoporosis is prescribed raloxifene (Evista). What should the nurse monitor in the client? Check serum creatinine Monitor urinary calcium Monitor liver function tests Observe for anxiety and drowsiness

Monitor liver function tests Raloxifene increases the risk for hepatic disease. Therefore the liver function test is monitored in a client who is prescribed this drug. Serum creatinine is checked in a client who is prescribed zoledronic acid. Urinary calcium is monitored in a client who is prescribed calcium supplements. Anxiety and drowsiness is observed in a client who is prescribed risedronate.

The patient is admitted with hypercalcemia; polyuria; and pain in the pelvis, spine, and ribs with movement. Which hematologic problem is likely to display these manifestations in the patient? Multiple myeloma Thrombocytopenia Megaloblastic anemia Myelodysplastic syndrome

Multiple myeloma

Which procedure does the nurse anticipate for the patient with suspected muscular dystrophy?

Muscle biopsy Muscle biopsy is done for the diagnosis of atrophy or inflammation (such as muscular dystrophy or polymyositis).

A patient with an acute peptic ulcer and major blood loss requires an immediate transfusion with packed red blood cells. Which task is appropriate for the registered nurse (RN) to delegate to unlicensed assistive personnel (UAP)? Confirm the IV solution is 0.9% saline. Obtain the vital signs before the transfusion is initiated. Monitor the patient for shortness of breath and back pain. Double check the patient identity and verify the blood product.

Obtain the vital signs before the transfusion is initiated

9. Which patients are at risk for osteoporosis because of nutritional issues? (Select all that apply.) a. Older adult female patient who likes to drink a lot of coffee b. Patient who has had gastric bypass surgery for obesity c. Patient who is on the high-protein Atkins diet d. Patient who prefers to drink diluted powdered milk e. Patient who drinks two carbonated diet sodas per day f. Patient with chronic alcoholism

Older adult female patient who likes to drink a lot of coffee Patient who is on the high-protein Atkins diet Patient with chronic alcoholism

23. An x-ray shows the presence of radiolucent bands (Looser's lines or zones) in a patient. What is this diagnostic finding specific for? a. Osteoporosis b. Osteomalacia c. Paget's disease d. Osteomyelitis

Osteomalacia

33. A patient comes to the emergency department (ED) after accidentally puncturing his hand with an automatic nail gun. Which disorder is this patient primarily at risk for? a. Osteoporosis b. Osteomyelitis c. Osteomalacia d. Dupuytren's contracture

Osteomyelitis

Bone or soft tissue biopsy, blood and/or wound cultures, WBC count, Erythrocyte sedimentation rate (ESR), bone scans, and MRI/CTs are all diagnostic tests used to assess and diagnose what inflammatory disease affecting mobility?

Osteomyelitis

According to the World Health Organization, a patient with a bone mineral density T-score of less than -2.5 should be reported as having

Osteoporosis

40. What is the most common type of malignant bone tumor? a. Ewing's sarcoma b. Chondrosarcoma c. Fibrosarcoma d. Osteosarcoma

Osteosarcoma

A patient with a diagnosis of hemophilia had a fall down an escalator earlier in the day and is now experiencing bleeding in the left knee joint. What should be the emergency nurse's immediate action? Immediate transfusion of platelets Resting the patient's knee to prevent hemarthroses Assistance with intracapsular injection of corticosteroids Range-of-motion exercises to prevent thrombus formation

Resting the patient's knee to prevent hemarthroses

Which condition may be detected by using the drop arm test?

Rotator cuff injury

The x-ray report of a patient shows a fracture at the femur. The nurse anticipates that which type of traction will be prescribed to reduce muscle spasm and tissue damage?

Russell's traction

What are clinical manifestations of chronic osteomyelitis?

Systemic signs are dimished, local signs of infection are more common (pain, swelling, warmth)

It is important a patient with gout avoid medications that can increase uric acid levels. Which medication below is NOT known to increase uric acid levels? A. Aspirin B. Niacin C. Cyclosporine D. Tylenol

The answer is D. Options A, B, and C increase uric acid. Option D does not.

D The nurse immobilizes the long bone to reduce movement of the fractured bone ends and decrease the risk of a fat embolus development before surgical reduction. Enoxaparin is used to prevent blood clots not fat emboli. Range of motion and compression boots will not prevent a fat embolus in this patient.

The patient is brought to the emergency department after a car accident and is diagnosed with a femur fracture. What nursing intervention should the nurse implement at this time to decrease risk of a fat embolus? Administer enoxaparin (Lovenox). Provide range-of-motion exercises. Apply sequential compression boots. Immobilize the fracture preoperatively.

What is the purpose of pregabalin​ (Lyrica) in the treatment of​ fibromyalgia? To reduce symptoms of fatigue To treat neuropathic pain To increase serotonin To promote sleep

To treat neuropathic pain Pregabalin was developed for the treatment of neuropathic pain. Serotonin reuptake inhibitors such as fluoxetine increase​ serotonin, promote improved​ sleep, and may reduce the symptoms of fibromyalgia.

2. Which objective patient data are associated with osteomalacia? (Select all that apply.) a. Unsteady gait b. Vertebral fracture c. Long bone bowing d. Hip flexion contractures e. Discomfort on vertebral palpation f. Bone tenderness over ribcage

Unsteady gait Long bone bowing Bone tenderness over ribcage

What does the nurse teach a patient with osteomalacia (aka Ricketts) to include in the daily diet?

Vitamin D Osteomalacia causes softening of the bone due to the decrease in vitamin D in the body. The patient with osteomalacia should include vitamin D in the diet because it promotes the absorption of calcium and phosphorus from the small intestine. The bone accounts for 99% of the calcium and 90% of the phosphorus in the body.

31. The nurse is caring for a patient with osteomyelitis. Which laboratory results are of primary concern for this disorder? a. Bone-specific alkaline phosphatase and osteocalcin b. Serum calcium level and alkaline phosphatase c. White blood cell count and erythrocyte sedimentation rate d. Tyroid function tests and uric acid levels

White blood cell count and erythrocyte sedimentation rate

The nurse is taking the history of an adult female client. Which factor places the client at risk for osteoporosis? A. Consuming 12 ounces of carbonated beverages daily B. Working at a desk and playing the piano for a hobby C. Having a hysterectomy and taking estrogen replacement therapy D. Consuming one alcoholic drink per week

Working at a desk and playing the piano for a hobby

A patient has been diagnosed with osteomalacia. What common symptoms does the nurse recognize that correlate with the diagnosis? a) Bone pain and tenderness b) Bone fractures and kyphosis c) Muscle weakness and spasms d) Softened and compressed vertebrae

a) Bone pain and tenderness Explanation: Osteomalacia is a metabolic bone disease characterized by inadequate mineralization of bone. As a result, the skeleton softens and weakens, causing pain, tenderness to touch, bowing of the bones, and pathologic fractures. On physical examination, skeletal deformities (spinal kyphosis and bowed legs) give patients an unusual appearance and a waddling gait. pg.1146

A nurse is teaching a client who was recently diagnosed with carpal tunnel syndrome. Which statement should the nurse include? a) "This condition is associated with various sports." b) "Ergonomic changes can be incorporated into your workday to reduce stress on your wrist." c) "Surgery is the only sure way to manage this condition." d) "Using arm splints will prevent hyperflexion of the wrist."

b) "Ergonomic changes can be incorporated into your workday to reduce stress on your wrist." Explanation: Ergonomic changes, such as adjusting keyboard height, can help clients with carpal tunnel syndrome avoid hyperextension of the wrist. This condition is associated with repetitive tasks such as clerical work, not sports. The condition may be managed with medications, yoga, acupuncture, and wrist (not arm) splints. pg.1136

A client with diabetes punctured his foot with a broken acorn in the yard. Within a week, the client developed osteomyelitis of the foot. The client was admitted for IV antibiotic therapy. How long does the nurse anticipate the client will receive IV antibiotics? a) 7 to 10 days b) 3 to 6 weeks c) 6 months d) 3 months

b) 3 to 6 weeks Explanation: Identification of the causative organism to initiate appropriate and ongoing antibiotic therapy for infection control. IV antibiotic therapy is administered for 3 to 6 weeks. Oral antibiotics then follow for as long as 3 months. pg.1148

A patient is diagnosed with osteogenic sarcoma. What laboratory studies should the nurse monitor for the presence of elevation? a) Potassium level b) Magnesium level c) Alkaline phosphatase d) Troponin levels

c) Alkaline phosphatase Explanation: Serum alkaline phosphatase levels are frequently elevated with osteogenic sarcoma or bone metastasis. Hypercalcemia is also present with bone metastases from breast, lung, or kidney cancer. Symptoms of hypercalcemia include muscle weakness, fatigue, anorexia, nausea, vomiting, polyuria, cardiac dysrhythmias, seizures, and coma. Hypercalcemia must be identified and treated promptly. pg.1152

Which sign may be helpful in identifying carpal tunnel syndrome? a) Brudzinski's b) Babinski's c) Tinel's d) Kernig's

c) Tinel's Explanation: Tinel's sign may be used to help identify carpal tunnel syndrome. The presence of the Babinski's sign can identify disease of the brain and spinal cord in adults and also exists as a primitive reflex in infants. The Brudzinski's and Kernig's sign are indicative of meningeal irritation. pg.1136

The nurse has educated a patient with low back pain about techniques to relieve the back pain and prevent further complications. What statement by the patient shows understanding of the education the nurse provided? a) "I will bend at the waist when I am lifting objects from the floor." b) "Instead of turning around to grasp an object, I will twist at the waist." c) "I will lie prone with my legs slightly elevated." d) "I will avoid prolonged sitting or walking."

d) "I will avoid prolonged sitting or walking." Explanation: The nurse encourages the patient to alternate lying, sitting, and walking activities frequently, and advises the patient to avoid sitting, standing, or walking for long periods. pg.1133

Which area of the spinal column is subject to the greatest mechanical stress and degenerative changes? a) Cervical b) Thoracic c) Upper lumbar d) Lower lumbar

d) Lower lumbar Explanation: The lower lumbar disks, L4 to L5 and L5 to S1, are subject to the greatest mechanical stress and greatest degenerative changes. pg.1133

Morton's neuroma is exhibited by which of the following clinical manifestations? a) Inflammation of the foot-supporting fascia b) Longitudinal arch of the foot is diminished c) High arm and a fixed equinus deformity d) Swelling of the third (lateral) branch of the median plantar nerve

d) Swelling of the third (lateral) branch of the median plantar nerve Explanation: Morton's neuroma is swelling of the third branch of the median plantar nerve. Pes cavus refers to a foot with an abnormally high arch and a fixed equinus deformity of the forefoot. Flatfoot is a common disorder in which the longitudinal arch of the foot is diminished. Plantar fasciitis is an inflammation of the foot-supporting fascia. pg.1139

What are systemic manifestations of acute osteomyelitis?

fever, night sweats, chills, restless, nausea, malaise, and drainage (late)

The drop arm test is used to assess patients with suspected: cervical injury. rotator cuff injury. impingement syndrome. malingering.

rotator cuff injury Explanation: Rotator cuff injury is one of the most common causes of shoulder pain. The rotator cuff is composed of 4 muscles. The one most susceptible to injury is the supraspinatus. Consequently, this is the one most commonly torn. To assess whether the supraspinatus muscle is intact, the drop arm test is performed. The patient is asked to abduct his affected arm laterally to 90 degrees. He is then asked to slowly lower it to his side. A positive test is noted when the arm "drops". This usually indicates that the supraspinatus is injured.

Acute osteomyelitis is treated by what medications?

vigorous and prolonged IV antibiotics

A postmenopausal client is scheduled for a DEXA scan. To plan for the client's test, what should the nurse communicate to the client? 1. Request that the client remove all metal objects on the day of the scan. 2. Instruct the client to consume foods and beverages with a high content of calcium for 2 days before the test. 3. Inform the client that she will need to ingest 600 mg of calcium gluconate by mouth for 2 weeks before the test. 4. Tell the client that she should report any significant pain to her physician at least 2 days before the test.

1. Metal will interfere with the test. Metallic objects within the examination field, such as jewelry, earrings, and dental amalgams, may inhibit organ visualization and can produce unclear images. Ingesting foods and beverages days before the test will not affect bone mineral status. Short-term calcium gluconate intake will also not influence bone mineral status. The client may already have had chronic pain as a result of a bone fracture or from osteoporosis.

19. A patient is started on alendronate (Fosamax) once weekly for the treatment of osteoporosis. The nurse determines that further instruction about the drug is needed when what is said by the patient? a. "I should take the drug with a meal to prevent stomach irritation." b. "This drug will prevent further bone loss and increase my bone density." c. "I need to sit or stand upright for at least 30 minutes after taking the drug." d. "I will still need to take my calcium supplements while taking this new drug."

19. a. The bisphosphonates, such as alendronate, must be taken at least 30 minutes before food or other medications to promote their absorption. As well, because they are very irritating to the stomach and esophagus, the patient must remain upright for at least 30 minutes after taking the medication to prevent reflux into the esophagus. These drugs will prevent further bone loss and increase bone density but calcium and vitamin D supplementation is still needed for bone formation.

2. A patient with osteomyelitis has a nursing diagnosis of risk for injury. What is an appropriate nursing intervention for this patient? a. Use careful and appropriate disposal of soiled dressings. b. Gently handle the involved extremity during movement. c. Measure the circumference of the affected extremity daily. d. Provide range-of-motion (ROM) exercise q4hr to the involved extremity.

2. b. The patient with osteomyelitis is at risk for pathologic fractures at the site of the infection because of weakened, devitalized bone and careful handling of the extremity isnecessary. Careful handling of dressings is necessary to prevent the spread of infection to others but is not related to preventing injury to this patient. Splints may be used to immobilize the limb, range-of-motion (ROM) exercises will be limited because of the possibility of spreading infection, and edema is not a common finding in osteomyelitis.

On a visit to the clinic, a client reports the onset of early symptoms of rheumatoid arthritis. The nurse should conduct a focused assessment for: 1. Limited motion of joints. 2. Deformed joints of the hands. 3. Early morning stiffness. 4. Rheumatoid nodules.

3. Initially, most clients with early symptoms of rheumatoid arthritis complain of early morning stiffness or stiffness after sitting still for a while. Later symptoms of rheumatoid arthritis include limited joint range of motion; deformed joints, especially of the hand; and rheumatoid nodules.

At which of the following times should the nurse instruct the client to take ibuprofen (Motrin), prescribed for left hip pain secondary to osteoarthritis, to minimize gastric mucosal irritation? 1. At bedtime. 2. On arising. 3. Immediately after a meal. 4. On an empty stomach.

3. Drugs that cause gastric irritation, such as ibuprofen, are best taken after or with a meal, when stomach contents help minimize the local irritation. Taking the medication on an empty stomach at any time during the day will lead to gastric irritation. Taking the drug at bedtime with food may cause the client to gain weight, possibly aggravating the osteoarthritis. When the client arises, he is stiff from immobility and should use warmth and stretching until he gets food in his stomach.

3. A patient who experienced an open fracture of the humerus 2 weeks ago is having increased pain at the fracture site. To identify a possible causative agent of osteomyelitis at the site, what should the nurse expect testing to include? a. X-rays b. CT scan c. Bone biopsy d. WBC count and erythrocyte sedimentation rate (ESR)

3. c. Because large doses of appropriate antibiotics are necessary in the treatment of acute osteomyelitis, it is important to identify the causative microorganism. The definitive way to determine the causative agent is by bone biopsy or biopsy of the soft tissue surrounding the site. The other tests may help to establish the diagnosis but do not identify the causative agent.

Which of the following should the nurse assess when completing the history and physical examination of a client diagnosed with osteoarthritis? 1. Anemia. 2. Osteoporosis. 3. Weight loss. 4. Local joint pain.

4. Osteoarthritis is a degenerative joint disease with local manifestations such as local joint pain, unlike rheumatoid arthritis, which has systemic manifestation such as anemia and osteoporosis. Weight loss occurs in rheumatoid arthritis, whereas most clients with osteoarthritis are overweight.

A 70-year-old male patient who underwent open reduction internal fixation (ORIF) surgery experiences a seizure after 24 hours. Which drug can cause seizures related to its toxic metabolites? 1 Tramadol 2 Ketorolac 3 Morphine 4 Meperidine

4. Meperidine (Demerol) Meperidine should not be used for pain management. Its toxic metabolites cause seizures and other adverse drug events, especially among older adults. Tramadol, ketorolac, and morphine are all employed in the management of post-operative pain. These do not cause any adverse reactions if administered in safely tolerated amounts.

6. Which type of bone tumor is a benign overgrowth of bone and cartilage and may transform into a malignant form? a. Endochroma b. Osteoclastoma c. Ewing's sarcoma d. Osteochondroma

6. d. Osteochondroma is a benign overgrowth of bone and cartilage near the end of the bone at the growth plate, especially in long bones, pelvis, or scapula. It may transform to a malignant form. Osteoclastoma is a benign bone tumor with a high rate of recurrence, but does not become malignant. Endochroma is benign but is an intramedullary cartilage tumor found in a cavity of a single hand or foot bone. Ewing's sarcoma develops in the medullary cavity of long bones, especially the femur, humerus, pelvis, and tibia.

A 24-year-old patient with a 12-year history of Becker muscular dystrophy is hospitalized with heart failure. What is an appropriate nursing intervention for this patient? a. Feed and bathe the patient to avoid exhausting the muscle. b. Reposition frequently to avoid skin and respiratory complications. c. Provide hand weights for the patient to exercise the upper extremities. d. Use orthopedic braces to promote ambulation and prevent muscle wasting.

8. b. Promotion of muscle activity is important in any patient with muscular dystrophy but when the disease has progressed to cardiomyopathy or respiratory failure, activity must be balanced with oxygen supply. At this stage of the disease, care should be taken to prevent skin or respiratory complications. The patient should be encouraged to perform as much self-care and exercise as energy allows but this will be limited.

9. What does radicular pain that radiates down the buttock and below the knee, along the distribution of the sciatic nerve, generally indicate? a. Cervical disc herniation b. Acute lumbosacral strain c. Degenerative disc disease d. Herniated intervertebral disc

9. d. Intervertebral disc herniation is generally indicated by radicular pain radiating down the buttock, below the knee, and along the distribution of the sciatic nerve. Cervical disc disease has pain radiating into the arms and hands. Acute lumbosacral strain causes acute low back pain. Degenerative disc disease is a structural degeneration of discs that is a normal process of aging and results in intervertebral discs losing their elasticity, flexibility, and shock-absorbing capabilities.

When planning care for a client affected by​ fibromyalgia, the nurse addresses the potential problem of activity intolerance. What will the nurse recommend to the client in order to most effectively address this​ problem? Daily meditation and guided imagery NSAID medications taken on a regular schedule Referral to physical therapy for an assistive device A program of​ regular, mild to moderate exercise

A program of​ regular, mild to moderate exercise. Meditation and guided imagery can reduce anxiety. NSAIDs address the problem of pain. Assistive devices do not increase conditioning or activity tolerance in the absence of injury or neurologic deficits.​ Regular, mild to moderate exercise improves conditioning and activity tolerance.

The nurse is caring for a client with suspected fibromyalgia. Which diagnostic tool does the nurse anticipate will be used to properly diagnose this​ client? Blood tests for neurotransmitter levels Abnormalities on a thyroid panel Failure of a cardiac stress test A widespread pain index

A widespread pain index Fibromyalgia is a diagnosis of exclusion and based largely on client​ feedback, such as responding to questions on a screening tool like the widespread pain index. There is no laboratory or diagnostic study that establishes the diagnosis.

A client with chronic osteomyelitis is being discharged from the hospital. What information is important for the nurse to teach this client and family? (Select all that apply.) a. Adherence to the antibiotic regimen b. Correct intramuscular injection technique c. Eating high-protein and high-carbohydrate foods d. Keeping daily follow-up appointments e. Proper use of the intravenous equipment

A, C, E The client going home with chronic osteomyelitis will need long-term antibiotic therapy—first intravenous, then oral. The client needs education on how to properly administer IV antibiotics, care for the IV line, adhere to the regimen, and eat a healthy diet to encourage wound healing. The antibiotics are not given by IM injection. The client does not need daily follow-up.

A 54-year-old patient with acute osteomyelitis asks the nurse how this problem will be treated. Which response by the nurse is most appropriate? A. "IV antibiotics are usually required for several weeks." B. "Oral antibiotics are often required for several months." C. "Surgery is almost always necessary to remove the dead tissue that is likely to be present." D. "Drainage of the foot and instillation of antibiotics into the affected area is the usual therapy."

A. "IV antibiotics are usually required for several weeks." The standard treatment for acute osteomyelitis consists of several weeks of IV antibiotic therapy. This is because bone is denser and less vascular than other tissues, and it takes time for the antibiotic therapy to eradicate all of the microorganisms. Surgery may be used for chronic osteomyelitis, which may include debridement of the devitalized and infected tissue and irrigation of the affected bone with antibiotics.

The nurse is admitting a patient to the nursing unit with a history of a herniated lumbar disc and low back pain. In completing a more thorough pain assessment, the nurse should ask the patient if which action aggravates the pain? A. Bending or lifting B. Application of warm moist heat C. Sleeping in a side-lying position D. Sitting in a fully extended recliner

A. Bending or lifting Back pain that is related to a herniated lumbar disc often is aggravated by events and activities that increase the stress and strain on the spine, such as bending or lifting, coughing, sneezing, and lifting the leg with the knee straight (straight leg-raising test). Application of moist heat, sleeping position, and ability to sit in a fully extended recliner do not aggravate the pain of a herniated lumbar disc.

The nurse is caring for a patient hospitalized with exacerbation of chronic bronchitis and herniated lumbar disc. Which breakfast choice would be most appropriate for the nurse to encourage the patient to check on the breakfast menu? A. Bran muffin B. Scrambled eggs C. Puffed rice cereal D. Buttered white toast

A. Bran muffin Each meal should contain one or more sources of fiber, which will reduce the risk of constipation and straining with defecation, which increases back pain. Bran is typically a high-fiber food choice and is appropriate for selection from the menu. Scrambled eggs, puffed rice cereal, and buttered white toast do not have as much fiber.

Which nursing intervention is most appropriate when turning a patient following spinal surgery? A. Placing a pillow between the patient's legs and turning the body as a unit B. Having the patient turn to the side by grasping the side rails to help turn over C. Elevating the head of bed 30 degrees and having the patient extend the legs while turning D. Turning the patient's head and shoulders and then the hips, keeping the patient's body centered in the bed

A. Placing a pillow between the patient's legs and turning the body as a unit Placing a pillow between the legs and turning the patient as a unit (logrolling) helps to keep the spine in good alignment and reduces pain and discomfort following spinal surgery. Having the patient turn by grasping the side rail to help, elevating the head of the bed, and turning with extended legs or turning the patient's head and shoulders and then the hips will not maintain proper spine alignment and may cause damage.

In which order will the nurse implement these collaborative interventions prescribed for a patient being admitted who has acute osteomyelitis with a temperature of 101.2° F? (Put a comma and a space between each answer choice [A, B, C, D].) a. Obtain blood cultures from two sites. b. Send to radiology for computed tomography (CT) scan of right leg. c. Administer gentamicin (Garamycin) 60 mg IV. d. Administer acetaminophen (Tylenol) now and every 4 hours PRN for fever

ANS: A, C, D, B The highest priority for possible osteomyelitis is initiation of antibiotic therapy, but cultures should be obtained before administration of antibiotics. Addressing the discomfort of the fever is the next highest priority. Because the purpose of the CT scan is to determine the extent of the infection, it can be done last

A patient with an exacerbation of rheumatoid arthritis (RA) is taking prednisone (Deltasone) 40 mg daily. Which of these assessment data obtained by the nurse indicate that the patient is experiencing a side effect of the medication? a. The patient's blood glucose is 165 mg/dL. b. The patient has no improvement in symptoms. c. The patient has experienced a recent 5-pound weight loss. d. The patient's erythrocyte sedimentation rate (ESR) has increased.

ANS: A Corticosteroids have the potential to cause diabetes mellitus. The finding of an elevated blood glucose reflects this side effect of prednisone. Corticosteroids increase appetite and lead to weight gain. An elevated ESR and no improvement in symptoms would indicate that the prednisone was not effective but would not be side effects of the medication

Which assessment finding about a patient who has been using naproxen (Naprosyn) for 3 weeks to treat osteoarthritis is most important for the nurse to report to the health care provider? a. The patient has dark colored stools. b. The patient's pain has not improved. c. The patient is using capsaicin cream (Zostrix). d. The patient has gained 3 pounds over 3 weeks.

ANS: A Dark colored stools may indicate that the patient is experiencing gastrointestinal bleeding caused by the naproxen. The information about the patient's ongoing pain and weight gain also will be reported and may indicate a need for a different treatment and/or counseling about avoiding weight gain, but these are not as large a concern as the possibility of gastrointestinal bleeding. Use of capsaicin cream with oral medications is appropriate.

Which action will the nurse take when caring for a patient with osteomalacia? a. Teach about the use of vitamin D supplements. b. Educate about the need for weight-bearing exercise. c. Discuss the use of medications such as bisphosphonates. d. Emphasize the importance of sunscreen use when outside.

ANS: A Osteomalacia is caused by inadequate intake or absorption of vitamin D. Weight-bearing exercise and bisphosphonate administration may be used for osteoporosis but will not be beneficial for osteomalacia. Because ultraviolet light is needed for the body to synthesize vitamin D, the patient might be taught that 20 minutes a day of sun exposure is beneficial. DIF: Cognitive Level: Apply (application) REF: 1510 TOP: Nursing Process: Implementation MSC: NCLEX: Physiological Integrity

Which action will the nurse take when caring for a patient with osteomalacia? a. Teach about the use of vitamin D supplements. b. Educate about the need for weight-bearing exercise. c. Discuss the use of medications such as bisphosphonates. d. Emphasize the importance of sunscreen use when outside.

ANS: A Osteomalacia is caused by inadequate intake or absorption of vitamin D. Weight-bearing exercise and bisphosphonate administration may be used for osteoporosis but will not be beneficial for osteomalacia. Because ultraviolet light is needed for the body to synthesize vitamin D, the patient might be taught that 20 minutes/day of sun exposure is beneficial.

An assessment finding for a 55-year-old patient that alerts the nurse to the presence of osteoporosis is a. a measurable loss of height. b. the presence of bowed legs. c. the aversion to dairy products. d. a statement about frequent falls.

ANS: A Osteoporosis occurring in the vertebrae produces a gradual loss of height. Bowed legs are associated with osteomalacia. Low intake of dairy products is a risk factor for osteoporosis, but it does not indicate that osteoporosis is present. Frequent falls increase the risk for fractures but are not an indicator of osteoporosis

An assessment finding for a 55-year-old patient that alerts the nurse to the presence of osteoporosis is a. a measurable loss of height. b. the presence of bowed legs. c. the aversion to dairy products. d. a statement about frequent falls.

ANS: A Osteoporosis occurring in the vertebrae produces a gradual loss of height. Bowed legs are associated with osteomalacia. Low intake of dairy products is a risk factor for osteoporosis, but it does not indicate that osteoporosis is present. Frequent falls increase the risk for fractures but are not an indicator of osteoporosis.

A patient who had arthroscopic surgery of the left knee 5 days ago is admitted with a red, swollen, and hot knee. Which assessment finding by the nurse should be reported to the health care provider immediately? a. The blood pressure is 88/46 mm Hg. b. The white blood cell count is 14,200/µL. c. The patient is taking ibuprofen (Motrin). d. The patient says the knee is very painful.

ANS: A The low blood pressure suggests that the patient may be developing septicemia as a complication of septic arthritis. Immediate blood cultures and initiation of antibiotic therapy are indicated. The other information is typical of septic arthritis and also should be reported to the health care provider, but it does not indicate any immediately life-threatening problems.

A student nurse learns about changes that occur to the musculoskeletal system due to aging. Which changes does this include? (Select all that apply.) a. Bone changes lead to potential safety risks. b. Increased bone density leads to stiffness. c. Osteoarthritis occurs due to cartilage degeneration. d. Osteoporosis is a universal occurrence. e. Some muscle tissue atrophy occurs with aging.

ANS: A, C, E Many age-related changes occur in the musculoskeletal system, including decreased bone density, degeneration of cartilage, and some degree of muscle tissue atrophy. Osteoporosis, while common, is not universal. Bone density decreases with age, not increases.

Which assessment information will the nurse obtain to evaluate the effectiveness of the prescribed calcitonin (Cibacalcin) and ibandronate (Boniva) for a patient with Paget's disease? a. Pain level b. Oral intake c. Daily weight d. Grip strength

ANS: A Bone pain is one of the common early manifestations of Paget's disease, and the nurse should assess the pain level to determine whether the treatment is effective. The other information will also be collected by the nurse, but will not be used in evaluating the effectiveness of the therapy.

A patient whose work involves lifting has a history of chronic back pain. After the nurse has taught the patient about correct body mechanics, which patient statement indicates that the teaching has been effective? a. "I plan to start doing exercises to strengthen the muscles of my back." b. "I will try to sleep with my hips and knees extended to prevent back strain." c. "I can tell my boss that I need to change to a job where I can work at a desk." d. "I will keep my back straight when I need to lift anything higher than my waist."

ANS: A Exercises can help to strengthen the muscles that support the back. Flexion of the hips and knees places less strain on the back. Modifications in the way the patient lifts boxes are needed, but sitting for prolonged periods can aggravate back pain. The patient should not lift above the level of the elbows.

A patient is hospitalized for initiation of regional antibiotic irrigation for acute osteomyelitis of the right femur. Which intervention will be included in the plan of care? a. Immobilization of the right leg b. Frequent weight-bearing exercise c. Avoiding administration of nonsteroidal anti-inflammatory drugs (NSAIDs) d. Support of the right leg in a flexed position

ANS: A Immobilization of the affected leg helps decrease pain and reduce the risk for pathologic fractures. Weight-bearing exercise increases the risk for pathologic fractures. NSAIDs are frequently prescribed to treat pain. Flexion of the affected limb is avoided to prevent contractures.

Which actions will the nurse include in the plan of care for a patient with metastatic bone cancer of the left femur (select all that apply)? a. Monitor serum calcium. b. Teach about the need for strict bed rest. c. Discontinue use of sustained-release opioids. d. Support the left leg when repositioning the patient. e. Support family and patient as they discuss the prognosis.

ANS: A, D, E The nurse will monitor for hypercalcemia caused by bone decalcification. Support of the leg helps reduce the risk for pathologic fractures. Although the patient may be reluctant to exercise, activity is important to maintain function and avoid complications associated with immobility. Adequate pain medication, including sustained-release and rapid-acting opioids, is needed for the severe pain often associated with bone cancer. The prognosis for metastatic bone cancer is poor so the patient and family need to be supported as they deal with the reality of the situation. DIF: Cognitive Level: Apply (application) REF: 1501 TOP: Nursing Process: Planning MSC: NCLEX: Physiological Integrity

An assessment finding for a 55-yr-old patient that alerts the nurse to the presence of osteoporosis is a. bowed legs. b. a loss of height. c. the report of frequent falls. d. an aversion to dairy products.

ANS: B Osteoporosis occurring in the vertebrae produces a gradual loss of height. Bowed legs are associated with osteomalacia. Low intake of dairy products is a risk factor for osteoporosis, but it does not indicate osteoporosis is present. Frequent falls increase the risk for fractures but are not an indicator of osteoporosis. DIF: Cognitive Level: Understand (comprehension) REF: 1511 TOP: Nursing Process: Assessment MSC: NCLEX: Physiological Integrity

A client is undergoing computed tomography (CT) of a joint. What action by the nurse is most important before the test? a. Administer sedation as prescribed. b. Assess for seafood or iodine allergy. c. Ensure that the client has no metal on the body. d. Provide preprocedure pain medication.

ANS: B Because CT uses iodine-based contrast material, the nurse assesses the client for allergies to iodine or seafood (which often contains iodine). The other actions are not needed.

A patient with muscular dystrophy is hospitalized with pneumonia. Which nursing action will be included in the plan of care? a. Logroll the patient every 2 hours. b. Assist the patient with ambulation. c. Discuss the need for genetic testing with the patient. d. Teach the patient about the muscle biopsy procedure.

ANS: B Because the goal for the patient with muscular dystrophy is to keep the patient active for as long as possible, assisting the patient to ambulate will be part of the care plan. The patient will not require logrolling. Muscle biopsies are necessary to confirm the diagnosis but are not necessary for a patient who already has a diagnosis. There is no need for genetic testing because the patient already knows the diagnosis. DIF: Cognitive Level: Apply (application) REF: 1502 TOP: Nursing Process: Planning MSC: NCLEX: Physiological Integrity

A 23-year-old patient with a history of muscular dystrophy is hospitalized with pneumonia. Which nursing action will be included in the plan of care? a. Logroll the patient every 2 hours. b. Assist the patient with ambulation. c. Discuss the need for genetic testing with the patient. d. Teach the patient about the muscle biopsy procedure.

ANS: B Because the goal for the patient with muscular dystrophy is to keep the patient active for as long as possible, assisting the patient to ambulate will be part of the care plan. The patient will not require logrolling. Muscle biopsies are necessary to confirm the diagnosis but are not necessary for a patient who already has a diagnosis. There is no need for genetic testing because the patient already knows the diagnosis

Which information will the nurse include when teaching range-of-motion exercises to a patient with an exacerbation of rheumatoid arthritis? a. Affected joints should not be exercised when pain is present. b. Application of cold packs before exercise may decrease joint pain. c. Exercises should be performed passively by someone other than the patient. d. Walking may substitute for range-of-motion (ROM) exercises on some days.

ANS: B Cold application is helpful in reducing pain during periods of exacerbation of RA. Because the joint pain is chronic, patients are instructed to exercise even when joints are painful. ROM exercises are intended to strengthen joints as well as improve flexibility, so passive ROM alone is not sufficient. Recreational exercise is encouraged but is not a replacement for ROM exercises.

A patient with dermatomyositis is receiving long-term prednisone (Deltasone) therapy. Which assessment finding by the nurse is most important to report to the health care provider? a. The blood glucose is 112 mg/dL. b. The patient has painful hematuria. c. The patient has an increased appetite. d. Acne is noted on the back and face.

ANS: B Corticosteroid use is associated with increased risk for infection, so the nurse should report the urinary tract symptoms immediately to the health care provider. The increase in blood glucose, increased appetite, and acne also are adverse effects of corticosteroid use, but do not need diagnosis and treatment as rapidly as the probable urinary tract infection.

A 39-year-old patient whose work involves frequent lifting has a history of chronic back pain. After the nurse has taught the patient about correct body mechanics, which patient statement indicates that the teaching has been effective? a. "I will keep my back straight to lift anything higher than my waist." b. "I will begin doing exercises to strengthen the muscles of my back." c. "I can try to sleep with my hips and knees extended to prevent back strain." d. "I can tell my boss that I need to change to a job where I can work at a desk."

ANS: B Exercises can help strengthen the muscles that support the back. Flexion of the hips and knees places less strain on the back. Modifications in the way the patient lifts boxes are needed, but sitting for prolonged periods can aggravate back pain. The patient should not lift above the level of the elbows

A patient with acute osteomyelitis of the left femur is hospitalized for regional antibiotic irrigation. Which intervention will the nurse include in the initial plan of care? a. Quadriceps-setting exercises b. Immobilization of the left leg c. Positioning the left leg in flexion d. Assisted weight-bearing ambulation

ANS: B Immobilization of the affected leg helps to decrease pain and reduce the risk for pathologic fracture. Weight-bearing exercise increases the risk for pathologic fractures. Flexion of the affected limb is avoided to prevent contractures. DIF: Cognitive Level: Apply (application) REF: 1499 TOP: Nursing Process: Planning MSC: NCLEX: Physiological Integrity

A patient has systemic sclerosis manifested by CREST (calcinosis, Raynaud's phenomenon, esophageal dysfunction, sclerodactyly, telangiectasia) syndrome. Which action will the nurse include in the plan of care? a. Avoid use of capsaicin cream on hands. b. Keep patient's room warm and draft free. c. Obtain capillary blood glucose before meals. d. Assist to bathroom every 2 hours while awake.

ANS: B Keeping the room warm will decrease the incidence of Raynaud's phenomenon, one aspect of the CREST syndrome. Capsaicin cream may be used to improve circulation and decrease pain. There is no need to obtain blood glucose levels or to assist the patient to the bathroom every 2 hours.

A patient with gout tells the nurse that he takes losartan (Cozaar) for control of the condition. The nurse will plan to monitor a. blood glucose. b. blood pressure. c. erythrocyte count. d. lymphocyte count.

ANS: B Losartan, an angiotensin II receptor antagonist, will lower blood pressure. It does not affect blood glucose, red blood cell count (RBC), or lymphocytes.

After obtaining the health history from a 28-year-old woman who is taking methotrexate (Rheumatrex) to treat rheumatoid arthritis, which information about the patient is most important for the nurse to report to the health care provider? a. The patient had a history of infectious mononucleosis as a teenager. b. The patient is trying to have a baby before her disease becomes more severe. c. The patient has a family history of age-related macular degeneration of the retina. d. The patient has been using large doses of vitamins and health foods to treat the RA.

ANS: B Methotrexate is teratogenic, and the patient should be taking contraceptives during methotrexate therapy. The other information will not impact the choice of methotrexate as therapy.

A 22-year-old patient hospitalized with a fever and red, hot, and painful knees is suspected of having septic arthritis. Information obtained during the nursing history that indicates a risk factor for septic arthritis is that the patient a. has a parent who has reactive arthritis. b. is sexually active and has multiple partners. c. recently returned from a trip to South America. d. had several sports-related knee injuries as a teenager.

ANS: B Neisseria gonorrhoeae is the most common cause for septic arthritis in sexually active young adults. The other information does not point to any risk for septic arthritis.

The nurse is caring for a client with a fractured fibula. Which assessment prompts immediate action by the nurse? a. Reported pain of 4 on a scale of 0 to 10 b. Numbness and tingling in the extremity c. Swollen extremity where the injury occurred d. Reports of being cold in bed

ANS: B The client with numbness and tingling of the extremity may be displaying the first signs of acute compartment syndrome. This is an acute problem that requires immediate intervention because of possible decreased circulation. Moderate pain and swelling is an expected assessment after a fracture. These findings can be treated with comfort measures. Being cold can be treated with additional blankets or by increasing the temperature of the room.

When the nurse is reviewing laboratory results for a patient with systemic lupus erythematosus (SLE), which result is most important to communicate to the health care provider? a. Decreased C-reactive protein (CRP) b. Elevated blood urea nitrogen (BUN) c. Positive antinuclear antibodies (ANA) d. Positive lupus erythematosus cell prep

ANS: B The elevated BUN and creatinine levels indicate possible lupus nephritis and a need for a change in therapy to avoid further renal damage. The positive lupus erythematosus (LE) cell prep and ANA would be expected in a patient with SLE. A drop in CRP shows an improvement in the inflammatory process.

A 50-year-old patient is being discharged after a week of IV antibiotic therapy for acute osteomyelitis in the right leg. Which information will be included in the discharge teaching? a. How to apply warm packs to the leg to reduce pain b. How to monitor and care for the long-term IV catheter c. The need for daily aerobic exercise to help maintain muscle strength d. The reason for taking oral antibiotics for 7 to 10 days after discharge

ANS: B The patient will be on IV antibiotics for several months, and the patient will need to recognize signs of infection at the IV site and how to care for the catheter during daily activities such as bathing. IV antibiotics rather than oral antibiotics are used for acute osteomyelitis. Patients are instructed to avoid exercise and heat application because these will increase swelling and the risk for spreading infection

A patient hospitalized with polymyositis has joint pain, an erythematosus facial rash with eyelid edema, and a weak, hoarse voice. The priority nursing diagnosis for the patient is a. acute pain related to inflammation. b. risk for aspiration related to dysphagia. c. risk for impaired skin integrity related to scratching. d. disturbed visual perception related to eyelid swelling.

ANS: B The patient's vocal weakness and hoarseness indicate weakness of the pharyngeal muscles and a high risk for aspiration. The other nursing diagnoses also are appropriate but are not as high a priority as the maintenance of the patient's airway.

Following laminectomy with a spinal fusion to treat a herniated disc, a patient reports numbness and tingling of the right lower leg. The first action that the nurse should take is to a. report the patient's complaint to the surgeon. b. check the chart for preoperative assessment data. c. check the vital signs for indications of hemorrhage. d. turn the patient to the side to relieve pressure on the right leg.

ANS: B The postoperative movement and sensation of the extremities should be unchanged (or improved) from the preoperative assessment. If the numbness and tingling are new, this information should be immediately reported to the surgeon. Numbness and tingling are not symptoms associated with hemorrhage at the site. Turning the patient will not relieve the numbness

When administering alendronate (Fosamax) to a patient with osteoporosis, the nurse will a. ask about any leg cramps or hot flashes. b. assist the patient to sit up at the bedside. c. be sure that the patient has recently eaten. d. administer the ordered calcium carbonate.

ANS: B To avoid esophageal erosions, the patient taking bisphosphonates should be upright for at least 30 minutes after taking the medication. Fosamax should be taken on an empty stomach, not after taking other medications or eating. Leg cramps and hot flashes are not side effects of bisphosphonates. DIF: Cognitive Level: Apply (application) REF: 1513 TOP: Nursing Process: Implementation MSC: NCLEX: Physiological Integrity

When administering alendronate (Fosamax) to a patient with osteoporosis, the nurse will a. ask about any leg cramps or hot flashes. b. assist the patient to sit up at the bedside. c. be sure that the patient has recently eaten. d. administer the ordered calcium carbonate.

ANS: B To avoid esophageal erosions, the patient taking bisphosphonates should be upright for at least 30 minutes after taking the medication. Fosamax should be taken on an empty stomach, not after taking other medications or eating. Leg cramps and hot flashes are not side effects of bisphosphonates.

A patient has chronic osteomyelitis of the left femur, which is being managed at home with administration of IV antibiotics. The nurse chooses a nursing diagnosis of ineffective health maintenance when the nurse finds that the patient a. takes and records the oral temperature twice a day. b. is unable to plantar flex the foot on the affected side. c. uses crutches to avoid weight bearing on the affected leg. d. is irritable and frustrated with the length of treatment required.

ANS: B Foot drop is an indication that the foot is not being supported in a neutral position by a splint. Using crutches and monitoring the oral temperature are appropriate self-care activities. Frustration with the length of treatment is not an indicator of ineffective health maintenance of the osteomyelitis.

A patient is being discharged after 2 weeks of IV antibiotic therapy for acute osteomyelitis in the left leg. Which information will be included in the discharge teaching? a. How to apply warm packs safely to the leg to reduce pain b. How to monitor and care for the long-term IV catheter site c. The need for daily aerobic exercise to help maintain muscle strength d. The reason for taking oral antibiotics for 7 to 10 days after discharge

ANS: B The patient will be on IV antibiotics for several months, and the patient will need to recognize signs of infection at the IV site and how to care for the catheter during daily activities such as bathing. IV antibiotics rather than oral antibiotics are used for acute osteomyelitis. Patients are instructed to avoid exercise and heat application because these will increase swelling and the risk for spreading infection.

When caring for a patient with a new diagnosis of rheumatoid arthritis, which action will the nurse include in the plan of care? a. Instruct the patient to purchase a soft mattress. b. Teach patient to use lukewarm water when bathing. c. Suggest that the patient take a nap in the afternoon. d. Suggest exercise with light weights several times daily.

ANS: C Adequate rest helps decrease the fatigue and pain that are associated with rheumatoid arthritis. Patients are taught to avoid stressing joints, to use warm baths to relieve stiffness, and to use a firm mattress.

A patient is receiving IV antibiotics at home to treat chronic osteomyelitis of the left femur. The nurse identifies a need for additional teaching related to health maintenance when the nurse finds that the patient a. is frustrated with the length of treatment required. b. takes and records the oral temperature twice a day. c. is unable to plantar flex the foot on the affected side. d. uses crutches to avoid weight bearing on the affected leg.

ANS: C Foot drop is an indication that the foot is not being supported in a neutral position by a splint. Using crutches and monitoring the oral temperature are appropriate self-care activities. Frustration with the length of treatment is not an indicator of ineffective health maintenance of the osteomyelitis. DIF: Cognitive Level: Apply (application) REF: 1499 TOP: Nursing Process: Analysis MSC: NCLEX: Physiological Integrity

Prednisone (Deltasone) is prescribed for a patient with an acute exacerbation of rheumatoid arthritis. Which laboratory result will the nurse monitor to determine whether the medication has been effective? a. Blood glucose test b. Liver function tests c. C-reactive protein level d. Serum electrolyte levels

ANS: C C-reactive protein is a marker for inflammation, and a decrease would indicate that the corticosteroid therapy was effective. Blood glucose and serum electrolyte levels also will be monitored to check for side effects of prednisone. Liver function is not routinely monitored for patients receiving steroids.

A nurse who works on the orthopedic unit has just received the change-of-shift report. Which patient should the nurse assess first? a. Patient who reports foot pain after hammertoe surgery b. Patient with low back pain and a positive straight-leg-raise test c. Patient who has not voided 10 hours after having a laminectomy d. Patient with osteomyelitis who has a temperature of 100.5° F (38.1° C)

ANS: C Difficulty in voiding may indicate damage to the spinal nerves and should be assessed and reported to the surgeon immediately. The information about the other patients is consistent with their diagnoses. The nurse will need to assess them as quickly as possible, but the information about them does not indicate a need for immediate intervention

A patient with rheumatoid arthritis refuses to take the prescribed methotrexate (Rheumatrex), telling the nurse "That drug has too many side effects. My arthritis isn't that bad yet." The most appropriate response by the nurse is a. "You have the right to refuse to take the methotrexate." b. "Methotrexate is less expensive than some of the newer drugs." c. "It is important to start methotrexate early to decrease the extent of joint damage." d. "Methotrexate is effective and has fewer side effects than some of the other drugs."

ANS: C Disease-modifying antirheumatic drugs (DMARDs) are prescribed early to prevent the joint degeneration that occurs as soon as the first year with RA. The other statements are accurate, but the most important point for the patient to understand is that it is important to start DMARDs as quickly as possible.

A 67-year-old patient is receiving IV antibiotics at home to treat chronic osteomyelitis of the left femur. The nurse chooses a nursing diagnosis of ineffective health maintenance when the nurse finds that the patient a. is frustrated with the length of treatment required. b. takes and records the oral temperature twice a day. c. is unable to plantar flex the foot on the affected side. d. uses crutches to avoid weight bearing on the affected leg.

ANS: C Foot drop is an indication that the foot is not being supported in a neutral position by a splint. Using crutches and monitoring the oral temperature are appropriate self-care activities. Frustration with the length of treatment is not an indicator of ineffective health maintenance of the osteomyelitis

Which of these patients seen by the nurse in the outpatient clinic is most likely to require teaching about ways to reduce risk for osteoarthritis (OA)? a. A 56-year-old man who is a member of a construction crew b. A 24-year-old man who participates in a summer softball team c. A 49-year-old woman who works on an automotive assembly line d. A 36-year-old woman who is newly diagnosed with diabetes mellitus

ANS: C OA is more likely to occur in women as a result of estrogen reduction at menopause and in individuals whose work involves repetitive movements and lifting. Moderate exercise, such as softball, reduces risk for OA. Diabetes is not a risk factor for OA. Working on a construction crew would involve nonrepetitive work and thus would not be as risky

The nurse instructs a patient who has osteosarcoma of the tibia about a scheduled above-the-knee amputation. Which statement by a patient indicates additional patient teaching is needed? a. "I will need to participate in physical therapy after surgery." b. "I wish I did not need to have chemotherapy after this surgery." c. "I did not have this bone cancer until my leg broke a week ago." d. "I can use the patient-controlled analgesia (PCA) to manage postoperative pain."

ANS: C Osteogenic sarcoma may be diagnosed following a fracture, but it is not caused by the injury. The other statements indicate patient teaching has been effective. DIF: Cognitive Level: Apply (application) REF: 1501 TOP: Nursing Process: Evaluation MSC: NCLEX: Physiological Integrity

A 26-year-old patient with urethritis and knee pain is diagnosed with reactive arthritis. The nurse will plan to teach the patient about the need for several months of therapy with a. anakinra (Kineret). b. etanercept (Enbrel). c. doxycycline (Vibramycin). d. methotrexate (Rheumatrex).

ANS: C Reactive arthritis associated with urethritis is usually caused by infection with Chlamydia trachomatis and requires 3 months of treatment with doxycycline. The other medications are used for chronic inflammatory problems such as rheumatoid arthritis.

Which nursing action included in the care of a patient after laminectomy can the nurse delegate to experienced unlicensed assistive personnel (UAP)? a. Check ability to plantar and dorsiflex the foot. b. Determine the patient's readiness to ambulate. c. Log roll the patient from side to side every 2 hours. d. Ask about pain management with the patient-controlled analgesia (PCA).

ANS: C Repositioning a patient is included in the education and scope of practice of UAP, and experienced UAP will be familiar with how to maintain alignment in the postoperative patient. Evaluation of the effectiveness of pain medications, assessment of neurologic function, and evaluation of a patient's readiness to ambulate after surgery require higher level nursing education and scope of practice. DIF: Cognitive Level: Apply (application) REF: 1498 OBJ: Special Questions: Delegation TOP: Nursing Process: Planning MSC: NCLEX: Safe and Effective Care Environment

A nurse sees clients in an osteoporosis clinic. Which client should the nurse see first? a. Client taking calcium with vitamin D (Os-Cal) who reports flank pain 2 weeks ago b. Client taking ibandronate (Boniva) who cannot remember when the last dose was c. Client taking raloxifene (Evista) who reports unilateral calf swelling d. Client taking risedronate (Actonel) who reports occasional dyspepsia

ANS: C The client on raloxifene needs to be seen first because of the manifestations of deep vein thrombosis, which is an adverse effect of raloxifene.

A home health patient with rheumatoid arthritis (RA) complains to the nurse about having chronically dry eyes. Which action by the nurse is most appropriate? a. Reassure the patient that dry eyes are a common problem with RA. b. Teach the patient more about adverse affects of the RA medications. c. Suggest that the patient start using over-the-counter (OTC) artificial tears. d. Ask the health care provider about lowering the methotrexate (Rheumatrex) dose.

ANS: C The patient's dry eyes are consistent with Sjögren's syndrome, a common extraarticular manifestation of RA. Symptomatic therapy such as OTC eye drops is recommended. Dry eyes are not a side effect of methotrexate. Although dry eyes are common in RA, it is more helpful to offer a suggestion to relieve these symptoms than to offer reassurance. The dry eyes are not caused by RA treatment, but by the disease itself.

The nurse should reposition the patient who has just had a laminectomy and diskectomy by a. instructing the patient to move the legs before turning the rest of the body. b. having the patient turn by grasping the side rails and pulling the shoulders over. c. placing a pillow between the patient's legs and turning the entire body as a unit. d. turning the patient's head and shoulders first, followed by the hips, legs, and feet.

ANS: C The spine should be kept in correct alignment after laminectomy. The other positions will create misalignment of the spine. DIF: Cognitive Level: Apply (application) REF: 1507 TOP: Nursing Process: Implementation MSC: NCLEX: Physiological Integrity

Which information will the nurse include when teaching a patient with acute low back pain (select all that apply)? a. Sleep in a prone position with the legs extended. b. Keep the knees straight when leaning forward to pick something up. c. Expect symptoms of acute low back pain to improve in a few weeks. d. Avoid activities that require twisting of the back or prolonged sitting. e. Use ibuprofen (Motrin, Advil) or acetaminophen (Tylenol) to relieve

ANS: C, D, E Acute back pain usually starts to improve within 2 weeks. In the meantime, the patient should use medications such as nonsteroidal antiinflammatory drugs (NSAIDs) or acetaminophen to manage pain and avoid activities that stress the back. Sleeping in a prone position and keeping the knees straight when leaning forward will place stress on the back and should be avoided. DIF: Cognitive Level: Apply (application) REF: 1503 TOP: Nursing Process: Planning MSC: NCLEX: Physiological Integrity

Which information will the nurse include when teaching a patient with acute low back pain (select all that apply)? a. Sleep in a prone position with the legs extended. b. Keep the knees straight when leaning forward to pick something up. c. Avoid activities that require twisting of the back or prolonged sitting. d. Symptoms of acute low back pain frequently improve in a few weeks. e. Ibuprofen (Motrin, Advil) or acetaminophen (Tylenol) can be used to relieve pain.

ANS: C, D, E Acute back pain usually starts to improve within 2 weeks. In the meantime, the patient should use medications such as nonsteroidal antiinflammatory drugs (NSAIDs) or acetaminophen to manage pain and avoid activities that stress the back. Sleeping in a prone position and keeping the knees straight when leaning forward will place stress on the back, and should be avoided

The nurse is assessing four clients with musculoskeletal disorders. The nurse should assess the client with which laboratory result first? a. Serum alkaline phosphatase (ALP): 108 units/L b. Serum aspartate aminotransferase (AST): 26 units/L c. Serum calcium: 10.2 mg/dL d. Serum phosphorus: 2 mg/dL

ANS: D A normal serum phosphorus level is 3 to 4.5 mg/dL; a level of 2 mg/dL is low, and this client should be assessed first. The values for serum ALP, AST, and calcium are all within normal ranges. (phosphorus is close to potassium ranges)

Which assessment finding for a patient who has had a surgical reduction of an open fracture of the right radius is most important to report to the health care provider? a. Serous wound drainage b. Right arm muscle spasms c. Right arm pain with movement d. Temperature 101.4° F (38.6° C)

ANS: D An elevated temperature is suggestive of possible osteomyelitis. The other clinical manifestations are typical after a repair of an open fracture

Which assessment finding for a patient who has had surgical reduction of an open fracture of the right radius requires notification of the health care provider? a. Serous wound drainage c. Right arm pain with movement b. Right arm muscle spasms d. Temperature 101.4° F (38.6° C)

ANS: D An elevated temperature suggests possible osteomyelitis. The other clinical manifestations are typical after a repair of an open fracture. DIF: Cognitive Level: Apply (application) REF: 1499 OBJ: Special Questions: Prioritization TOP: Nursing Process: Assessment MSC: NCLEX: Physiological Integrity

A 54-yr-old woman who recently reached menopause and has a family history of osteoporosis is diagnosed with osteopenia following densitometry testing. In teaching the woman, the nurse explains that a. with a family history of osteoporosis, there is no way to prevent or slow bone resorption. b. estrogen replacement therapy must be started to prevent rapid progression to osteoporosis. c. continuous, low-dose corticosteroid treatment is effective in stopping the course of osteoporosis. d. calcium loss from bones can be slowed by increasing calcium intake and weight-bearing exercise.

ANS: D Progression of osteoporosis can be slowed by increasing calcium intake and weight-bearing exercise. Estrogen replacement therapy is no longer routinely given to prevent osteoporosis because of increased risk of heart disease as well as breast and uterine cancer. Corticosteroid therapy increases the risk for osteoporosis. DIF: Cognitive Level: Apply (application) REF: 1512 TOP: Nursing Process: Implementation MSC: NCLEX: Physiological Integrity

A 54-year-old woman who recently reached menopause and has a family history of osteoporosis is diagnosed with osteopenia following densitometry testing. In teaching the woman about her osteoporosis, the nurse explains that a. estrogen replacement therapy must be started to prevent rapid progression to osteoporosis. b. continuous, low-dose corticosteroid treatment is effective in stopping the course of osteoporosis. c. with a family history of osteoporosis, there is no way to prevent or slow gradual bone resorption. d. calcium loss from bones can be slowed by increasing calcium intake and weight-bearing exercise.

ANS: D Progression of osteoporosis can be slowed by increasing calcium intake and weight-bearing exercise. Estrogen replacement therapy does help prevent osteoporosis, but it is not the only treatment and is not appropriate for some patients. Corticosteroid therapy increases the risk for osteoporosis.

Which menu choice by a patient with osteoporosis indicates that the nurse's teaching about appropriate diet has been effective? a. Pancakes with syrup and bacon b. Whole wheat toast and fresh fruit c. Egg-white omelet and a half grapefruit d. Oatmeal with skim milk and fruit yogurt

ANS: D Skim milk and yogurt are high in calcium. The other choices do not contain any high-calcium foods.

The home health nurse is doing a follow-up visit to a patient with recently diagnosed rheumatoid arthritis (RA). Which assessment made by the nurse indicates that more patient teaching is needed? a. The patient requires a 2-hour midday nap. b. The patient has been taking 16 aspirins daily. c. The patient sits on a stool when preparing meals. d. The patient sleeps with two pillows under the head.

ANS: D The joints should be maintained in an extended position to avoid contractures, so patients should use a small, flat pillow for sleeping. The other information is appropriate for a patient with RA and indicates that teaching has been effective.

The nurse will determine that more teaching is needed if a patient with discomfort from a bunion says, "I will a. give away my high-heeled shoes." b. take ibuprofen (Motrin) if I need it." c. use the bunion pad to cushion the area." d. only wear sandals, no closed-toe shoes."

ANS: D The patient can wear shoes that have a wide forefoot. The other patient statements indicate that the teaching has been effective

A 58-year-old woman who has a family history of osteoporosis is diagnosed with osteopenia following densitometry testing. In teaching the woman about her osteoporosis, the nurse explains that a. estrogen replacement therapy must be started to prevent rapid progression to osteoporosis. b. continuous, low-dose corticosteroid treatment is effective in stopping the course of osteoporosis. c. with a family history of osteoporosis, there is no way to prevent or slow gradual bone resorption. d. calcium loss from bones can be slowed by increasing calcium intake and weight-bearing exercise.

ANS: D Progression of osteoporosis can be slowed by increasing calcium intake and weight-bearing exercise. Estrogen replacement therapy does help prevent osteoporosis, but it is not the only treatment and is not appropriate for some patients. Corticosteroid therapy increases the risk for osteoporosis.

Which menu choice by a patient with osteoporosis indicates that the nurse's teaching about appropriate diet has been effective? a. Pancakes with syrup and bacon b. Whole wheat toast and fruit jelly c. Two-egg omelet and a half grapefruit d. Oatmeal with skim milk and fruit yogurt

ANS: D Skim milk and yogurt are high in calcium. The other choices do not contain any high calcium foods.

The nurse is caring for a patient with a diagnosis of immune thrombocytopenic purpura (ITP). What is a priority nursing action in the care of this patient? Administration of packed red blood cells Administration of oral or IV corticosteroids Administration of clotting factors VIII and IX Maintenance of reverse isolation and application of standard precautions

Administration of oral or IV corticosteroids

A nurse practitioner provides health teaching to the family of a 75-year-old woman who has trouble walking independently. The nurse reviews age-related changes to the musculoskeletal system with the family. Which of the following statements would the nurse include in her teaching? Select all that apply. Tendons become more elastic. Intervertebral discs become thin. Muscles atrophy. Muscle fibrosis increases. Collagen increases

Age-related musculoskeletal changes could include: -Gradual, pregoressive loss of bone mass after age 30 - Vertebral Collapse -Increase in collagen and resultant fibrosis (<strength/flexibility) - Muscle atrophy - Tendons less elastic - Progressive deterioration/thinning of cartilage and intervertebral disks - Lax ligaments (weak)

Alendronate (Fosamax) is prescribed for a patient with osteoporosis. The nurse teaches the patient that a )the drug must be taken with food to prevent GI side effects. b )bisphosphonates prevent calcium from being taken from the bones. c )lying down after taking the drug prevents light-headedness and dizziness. d )taking the drug with milk enhances the absorption of calcium from the bowel.

Answer is: b

Which patient would be at greatest risk for developing osteoporosis? a )A 73-year-old man who has five alcoholic drinks per week and limits sun exposure to prevent recurrence of skin cancer. b )An 84-year-old man who has recently been diagnosed with hypothyroidism and is prescribed levothyroxine (Synthroid). c )A 69-year-old woman who had a renal transplant 5 years ago and has been taking prednisone to prevent organ rejection. d )A 55-year-old woman who recently had a hysterectomy with bilateral salpingo-oophorectomy and refuses estrogen therapy.

Answer is: c

1. A patient with an open fracture of the left tibia and soft tissue damage underwent a surgical reduction and fixation of the tibia with debridement of nonviable tissue and drain placement. When assessing the patient during the postoperative period, the nurse will be most concerned about a. fever with chills and night sweats. b. light yellow drainage from the wound. c. pain on movement of the affected limb. d. muscle spasms around the affected bone.

Answer: A Rationale: Fever, chills, and night sweats are suggestive of osteomyelitis. The other clinical manifestations are typical after a fracture repair. Cognitive Level: Application Text Reference: p. 1669 Nursing Process: Assessment NCLEX: Physiological Integrity

Calcitonin-Salmon (Miacalcin) Indications IM SC Treatment of Paget's disease of bone. Adjunctive therapy for hypercalcemia. IM SC Intranasal Management of postmenopausal osteoporosis. Action Inhibits osteoclastic bone resorption and promotes renal excretion of calcium. Contraindication/Precautions Contraindicated in: Hypersensitivity to calcitonin, salmon protein or gelatin diluent (in some products); OB: Lactation: Use not recommended. Adverse Reactions/Side Effects CNS: nasal only: headaches Derm: facial flushing, rash EENT: nasal only: rhinitis, epistaxis, nasal irritation GI: IM, subcut: nausea, vomiting GU: IM, subcut: urinary frequency Local: injection site reactions MS: nasal: arthralgia, back pain Route/Dosage Postmenopausal osteoporosis IM SC (Adults): 100 units every other day. Intranasal (Adults): 1 spray (200 units)/day in alternating nostrils. Paget's disease IM SC (Adults): 100 units/day initially, after titration, maintenance dose is usually 50 units/day or every other day. Hypercalcemia IM SC (Adults): 4 units/kg every 12 hr; if adequate response not achieved, may ↑ dose after 1-2 days to 8 units/kg every 12 hr, and if necessary after 2 more days may be ↑ to 8 units/kg every 6 hr.

Assessment Observe patient for signs of hypersensitivity (skin rash, fever, hives, anaphylaxis, serum sickness). Keep epinephrine, antihistamines, and oxygen nearby in the event of a reaction. Implementation In patients with suspected sensitivity to calcitonin, consider skin test before starting therapy. Prepare test dose in a dilution of 10 units/mL by withdrawing 0.05 mL in a tuberculin syringe and filling to 1 mL with 0.9% NaCl for injection. Mix well and discard 0.9 mL. Administer 0.1 mL intradermally on inner aspect on forearm and observe site for 15 min. More than mild erythema or wheal constitutes positive response. Patient/Family Teaching Advise patient to take calcitonin as directed. If dose is missed and medication is scheduled for twice a day, take only if possible within 2 hr of correct time. If scheduled for daily dose, take only if remembered that day. If scheduled for every other day, take when remembered and restart alternate day schedule. If taking 1 dose 3 times weekly (Mon, Wed, Fri), take missed dose the next day and set each injection back 1 day; resume regular schedule the following week. Do not double doses. Evaluation/Desired Outcomes Lowered serum calcium levels. Decreased bone pain. Slowed progression of postmenopausal osteoporosis. Significant increases in bone marrow density may be seen as early as 6 mo after initiation of therapy.

Naproxen Indications Mild to moderate pain. Dysmenorrhea. Fever. Inflammatory disorders, including:Rheumatoid arthritis (adults and children),Osteoarthritis. Action Inhibits prostaglandin synthesis. Contraindication/Precautions Contraindicated in: Hypersensitivity; Cross-sensitivity may occur with other NSAIDs, including aspirin; Active GI bleeding; Ulcer disease; Coronary artery bypass graft (CABG) surgery; Adverse Reactions/Side Effects CNS: dizziness, drowsiness, headache CV: HF, MYOCARDIAL INFARCTION, STROKE, edema, hypertension, palpitations, tachycardia Derm: STEVENS-JOHNSON SYNDROME, TOXIC EPIDERMAL NECROLYSIS, photosensitivity, rash, sweating, pseudoporphyria (12% incidence in children with juvenile rheumatoid arthritis-discontinue therapy if this occurs) EENT: tinnitus, visual disturbances Route/Dosage 275 mg naproxen sodium is equivalent to 250 mg naproxen Anti-Inflammatory/Analgesic/Antidysmenorrheal PO (Adults): Naproxen- 250-500 mg twice daily (up to 1.5 g/day). Delayed-release naproxen- 375-500 mg twice daily. Naproxen sodium- 275-550 mg twice daily (up to 1.65 g/day).

Assessment Patients who have asthma, aspirin-induced allergy, and nasal polyps are at increased risk for developing hypersensitivity reactions. Assess for rhinitis, asthma, and urticaria. Implementation Administration in higher than recommended doses does not provide increased effectiveness but may cause increased side effects. Use lowest effective dose for the shortest duration possible to minimize risk of cardiovascular thrombotic events. Patient/Family Teaching Caution patient to avoid the concurrent use of alcohol, aspirin, acetaminophen, or other OTC medications without consulting health care professional. Use of naproxen with 3 or more glasses of alcohol per day may increase risk of GI bleeding. Evaluation/Desired Outcomes Relief of pain. Improved joint mobility. Partial arthritic relief is usually seen within 2 wk, but maximum effectiveness may require 2-4 wk of continuous therapy. Patients who do not respond to one NSAID may respond to another. Reduction of fever.

The 24-year-old male patient who was successfully treated for Paget's disease has come to the clinic with a gradual onset of pain and swelling around the left knee. The patient is diagnosed with osteosarcoma without metastasis. The patient wants to know why he will be given chemotherapy before the surgery. What is the best rationale the nurse should tell the patient? A. The chemotherapy is being used to save your left leg. B. Chemotherapy is being used to decrease the tumor size. C. The chemotherapy will increase your 5-year survival rate. D. Chemotherapy will help decrease the pain before and after surgery.

B. Chemotherapy is being used to decrease the tumor size. Preoperative chemotherapy is used to decrease tumor size before surgery. The chemotherapy will not save his leg if the lesion is too big or there is neurovascular or muscle involvement. Adjunct chemotherapy after amputation or limb salvage has increased 5-year survival rate in people without metastasis. Chemotherapy is not used to decrease pain before or after surgery.

The nurse identifies a nursing diagnosis of pain related to muscle spasms for a 45-year-old patient who has low back pain from a herniated lumbar disc. What would be an appropriate nursing intervention to treat this problem? A. Provide gentle ROM to the lower extremities. B. Elevate the head of the bed 20 degrees and flex the knees. C. Place the bed in reverse Trendelenburg with the feet firmly against the footboard. D. Place a small pillow under the patient's upper back to gently flex the lumbar spine.

B. Elevate the head of the bed 20 degrees and flex the knees. The nurse should elevate the head of the bed 20 degrees and flex the knees to avoid extension of the spine and increasing the pain. The slight flexion provided by this position often is comfortable for a patient with a herniated lumbar disc. ROM to the lower extremities will be limited to prevent extremes of spinal movement. Reverse Trendelenburg and a pillow under the patient's upper back will more likely increase pain.

The nurse is caring for a patient admitted to the nursing unit with osteomyelitis of the tibia. Which symptom will the nurse most likely find on physical examination of the patient? A. Nausea and vomiting B. Localized pain and warmth C. Paresthesia in the affected extremity D. Generalized bone pain throughout the leg

B. Localized pain and warmth Osteomyelitis is an infection of bone and bone marrow that can occur with trauma, surgery, or spread from another part of the body. Because it is an infection, the patient will exhibit typical signs of inflammation and infection, including localized pain and warmth. Nausea and vomiting and paresthesia of the extremity are not expected to occur. Pain occurs, but it is localized, not generalized throughout the leg.

A nurse is conducting a health screening among females at the mall to assess those who are at risk for developing osteoporosis. Which of the following questions is most appropriate to be asked by the nurse in relation to development of osteoporosis? a) at what age did you have your menstruation? b) did you have any fracture? c) are you taking corticosteroids? d) are you on the diet high in vitamin D?

C - corticosteroids promote calcium loss. This increases the risk for osteoporosis.

The nurse is caring for patients in a primary care clinic. Which individual is most at risk to develop osteomyelitis caused by Staphylococcus aureus? A. 22-year-old female with gonorrhea who is an IV drug user B. 48-year-old male with muscular dystrophy and acute bronchitis C. 32-year-old male with type 1 diabetes mellitus and a stage IV pressure ulcer D. 68-year-old female with hypertension who had a knee arthroplasty 3 years ago

C. 32-year-old male with type 1 diabetes mellitus and a stage IV pressure ulcer Osteomyelitis caused by Staphylococcus aureus is usually associated with a pressure ulcer or vascular insufficiency related to diabetes mellitus. Osteomyelitis caused by Staphylococcus epidermidis is usually associated with indwelling prosthetic devices such as joint replacements. Osteomyelitis caused by Neisseria gonorrhoeae is usually associated with gonorrhea. Osteomyelitis caused by Pseudomonas is usually associated with IV drug use. Muscular dystrophy is not associated with osteomyelitis.

The nurse determines that dietary teaching for a 75-year-old patient with osteoporosis has been successful when the patient selects which highest-calcium meal? A. Chicken stir-fry with 1 cup each onions and green peas, and 1 cup of steamed rice B. Ham and Swiss cheese sandwich on whole wheat bread, steamed broccoli, and an apple C. A sardine (3 oz) sandwich on whole wheat bread, 1 cup of fruit yogurt, and 1 cup of skim milk D. A two-egg omelet with 2 oz of American cheese, one slice of whole wheat toast, and a half grapefruit

C. A sardine (3 oz) sandwich on whole wheat bread, 1 cup of fruit yogurt, and 1 cup of skim milk The highest calcium content is present in the lunch containing milk and milk products (yogurt) and small fish with bones (sardines). Chicken, onions, green peas, rice, ham, whole wheat bread, broccoli, apple, eggs, and grapefruit each have less than 75 mg of calcium per 100 g of food. Swiss cheese and American cheese have more calcium, but not as much as the sardines, yogurt, and milk.

The nurse should include which of the following client teachings for prevention of rapid progression of osteoporosis? a) avoid taking skim milk b) avoid taking protein-rich foods c) avoid calcium supplement d) avoid alcohol

D - avoiding alcohol and cigarette smoking will prevent rapid progression of osteoporosis. Skim milk is indicated among elderly because it is low in fats. Protein foods are necessary for calcium absorption. Calcium supplements help maintain integrity of the bones

The nurse cares for a 58-year-old woman with breast cancer who is admitted for severe back pain related to a compression fracture. The patient's laboratory values include serum potassium of 4.5 mEq/L, serum sodium of 144 mEq/L, and serum calcium of 14.3 mg/dL. Which signs and symptoms will the nurse expect the patient to exhibit? A. Anxiety, irregular pulse, and weakness B. Muscle stiffness, dysphagia, and dyspnea C. Hyperactive reflexes, tremors, and seizures D. Nausea, vomiting, and altered mental status

D. Nausea, vomiting, and altered mental status Breast cancer can metastasize to the bone. Vertebrae are a common site. Pathologic fractures at the site of metastasis are common because of a weakening of the involved bone. High serum calcium levels result as calcium is released from damaged bones. Normal serum calcium is between 8.6 to 10.2 mg/dL. Clinical manifestations of hypercalcemia include nausea, vomiting, and altered mental status (e.g., lethargy, decreased memory, confusion, personality changes, psychosis, stupor, coma). Other manifestations include weakness, depressed reflexes, anorexia, bone pain, fractures, polyuria, dehydration, and nephrolithiasis. Manifestations of hypomagnesemia include hyperactive reflexes, tremors, and seizures. Symptoms of hyperkalemia include anxiety, irregular pulse, and weakness. Symptoms of hypocalcemia include muscle stiffness, dysphagia, and dyspnea.

A 67-year-old patient hospitalized with osteomyelitis has an order for bed rest with bathroom privileges with the affected foot elevated on two pillows. The nurse would place highest priority on which intervention? A. Ambulate the patient to the bathroom every 2 hours. B. Ask the patient about preferred activities to relieve boredom. C. Allow the patient to dangle legs at the bedside every 2 to 4 hours. D. Perform frequent position changes and range-of-motion exercises.

D. Perform frequent position changes and range-of-motion exercises. The patient is at risk for atelectasis of the lungs and for contractures because of prescribed bed rest. For this reason, the nurse should place the priority on changing the patient's position frequently to promote lung expansion and performing range-of-motion (ROM) exercises to prevent contractures. Assisting the patient to the bathroom will keep the patient safe as the patient is in pain, but it may not be needed every 2 hours. Providing activities to relieve boredom will assist the patient to cope with the bed rest, and dangling the legs every 2 to 4 hours may be too painful. The priority is position changes and ROM exercises.

When the patient is diagnosed with muscular dystrophy, what information should the nurse include in the teaching about this disorder? A. Prolonged bed rest will be used to decrease fatigue. B. An orthotic jacket will limit mobility and may contribute to deformity. C. Continuous positive airway pressure will be used to facilitate sleeping. D. Remain active to prevent skin breakdown and respiratory complications.

D. Remain active to prevent skin breakdown and respiratory complications. With muscular dystrophy, it is important for the patient to remain active for as long as possible. Prolonged bed rest should be avoided because immobility leads to further muscle wasting. An orthotic jacket may be used to provide stability and prevent further deformity. Continuous positive airway pressure (CPAP) is used as respiratory function decreases, before mechanical ventilation is needed to sustain respiratory function.

Which patient should NOT be prescribed alendronate (Fosamax) for osteoporosis? a-A female patient being treated for high blood pressure with an ACE inhibitor. b-A patient who is allergic to iodine/shellfish. c-A patient on a calorie restricted diet. d-A patient on bed rest who must maintain a supine position

D: Alendronate can cause significant gastrointestinal side effects, such as esophageal irritation, so it should not be taken if a patient must stay in supine position. A&B: ACE inhibitors are not contraindicated with alendronate and there is no iodine allergy relationship. C: The patient should not eat or drink for 30 minutes after administration and should not lie down.

When the first class of drugs ((DMARDs) prescribed for rheumatoid arthritis FAILS, the nurse anticipates which category of drugs will be prescribed? A. Nonsteroidal anti-inflammatory B. Disease modifying antirheumatic C. Salicylates D. Biologic response modulators

D: Disease modifying antirheumatic drugs (DMARDs) are the first drugs used to try to reduce joint clinical manifestations in rheumatoid arthritis. Biologic response modulators (**examples adalimumab& infliximab**) have a 66% success rate AFTER failure with DMARDs.

Each bone is composed of cells, protein matrix, and mineral deposits. Which type of bone cell works to repair a bone fracture? osteocytes osteoblasts osteoclasts osteomytes

During times of rapid bone growth or bone injury, osteocytes function as osteoblasts to form new bone.

The blood bank notifies the nurse that the two units of blood ordered for a patient is ready for pick up. Which action should the nurse take to prevent an adverse effect during this procedure? Immediately pick up both units of blood from the blood bank. Infuse the blood slowly for the first 15 minutes of the transfusion. Regulate the flow rate so that each unit takes at least 4 hours to transfuse. Set up the Y-tubing of the blood set with dextrose in water as the flush solution.

Infuse the blood slowly for the first 15 minutes of the transfusion.

Alendronate is used in treatment of osteoporosis because it?

Inhibits osteoclastic activity

A nurse practitioner was asked to explain to a patient the age-related process of bone loss that results in osteoporosis. Which of the following is the best statement the nurse should use? Decreased estrogen inhibits bone breakdown. Increased calcitonin enhances bone resorption. Increased vitamin D use interferes with calcium use. Decreased parathyroid hormone increases bone resorption.

Osteoporosis is characterized by decreased estrogen, calcitonin, and vitamin D use; the parathyroid hormone increases with age, causing increased bone turnover and resorption.

Which element is included in the focused physical examination portion of the nursing assessment of the client with​ fibromyalgia? Pattern of fatigue Symptom severity scale Palpation of tenderness points Duration of pain

Palpation of tenderness points Palpation is an element of the physical examination portion of the nursing assessment. Duration of​ pain, fatigue​ pattern, and symptom severity scale are elements of the health history portion of the nursing assessment.

8. Which patient is at risk for regional osteoporosis? a. Patient who has been in a long leg cast for 10 weeks b. Patient on long-term corticosteroid therapy c. Patient with a history of hyperparathyroidism d. Menopausal patient

Patient who has been in a long leg cast for 10 weeks

A patient will receive a hematopoietic stem cell transplant (HSCT). What is the nurse's priority after the patient receives combination chemotherapy before the transplant? Prevent patient infection. Avoid abnormal bleeding. Give pneumococcal vaccine. Provide companionship while isolated.

Prevent patient infection.

A patient has been diagnosed with stage 1A Hodgkin's lymphoma. The nurse knows that which chemotherapy regimen is most likely to be prescribed for this patient? Brentuximab vedotin (Adcetris) Two to four cycles of ABVD: doxorubicin (Adriamycin), bleomycin, vinblastine, and dacarbazine Four to six cycles of ABVD: doxorubicin (Adriamycin), bleomycin, vinblastine, and dacarbazine BEACOPP: bleomycin, etoposide, doxorubicin (Adriamycin), cyclophosphamide, vincristine (Oncovin), procarbazine, and prednisone

Two to four cycles of ABVD: doxorubicin (Adriamycin), bleomycin, vinblastine, and dacarbazine

The nurse teaches the patient with a high risk for osteoporosis about risk-lowering strategies including which of the following actions? a) Walk or perform weight-bearing exercises b) Decrease the intake of vitamin A and D c) Reduce stress d) Increase fiber in the diet

a) Walk or perform weight-bearing exercises Explanation: Risk-lowering strategies for osteoporosis include walking or exercising outdoors, performing a regular weight-bearing exercise regimen, increasing dietary calcium and vitamin D intake, smoking cessation, and consuming alcohol and caffeine consumption in moderation. pg.1144

A patient is started on alendronate (Fosamax) once weekly for the treatment of osteoporosis. The nurse determines that further instruction about the drug is needed when what is said by the patient? a. "I should take the drug with a meal to prevent stomach irritation." b. "This drug will prevent further bone loss and increase my bone density." c. "I need to sit or stand upright for at least 30 minutes after taking the drug." d. "I will still need to take my calcium supplements while taking this new drug."

a. "I should take the drug with a meal to prevent stomach irritation." The bisphosphonates, such as alendronate, must be taken at least 30 minutes before food or other medications to promote their absorption. As well, because they are very irritating to the stomach and esophagus, the patient must remain upright for at least 30 minutes after taking the medication to prevent reflux into the esophagus. These drugs will prevent further bone loss and increase bone density but calcium and vitamin D supplementation is still needed for bone formation.

A patient stepped on an acorn while walking barefoot in the backyard and developed an infection progressing to osteomyelitis. What microorganism does the nurse understand is most often the cause of the development of osteomyelitis? a) Proteus b) Staphylococcus aureus c) Salmonella d) Pseudomonas

b) Staphylococcus aureus Explanation: More than 50% of bone infections are caused by Staphylococcus aureus and increasingly of the variety that is methicillin resistant (i.e., methicillin-resistant Staphylococcus aureus [MRSA]) (Miller & Kaplan, 2009). Other pathogens include the gram-positive organisms streptococci and enterococci, followed by gram-negative bacteria, including pseudomonas. pg.1148

A patient is having low back pain. What position can the nurse suggest to relieve this discomfort? a) Prone, with a pillow under the shoulders b) Supine, with the knees slightly flexed and the head of the bed elevated 30 degrees c) High-Fowler's to allow for maximum hip flexion d) Supine, with the bed flat and a firm mattress in place

b) Supine, with the knees slightly flexed and the head of the bed elevated 30 degrees Explanation: A medium to firm, nonsagging mattress (a bed board may be used) is recommended; there is no evidence to support the use of a firm mattress (National Guideline Clearinghouse, 2010). Lumbar flexion is increased by elevating the head and thorax 30 degrees by using pillows or a foam wedge and slightly flexing the knees supported on a pillow. Alternatively, the patient can assume a lateral position with knees and hips flexed (curled position) with a pillow between the knees and legs and a pillow supporting the head (Fig. 42-1). A prone position should be avoided because it accentuates lordosis. pg.1134

A nurse is teaching a female client about preventing osteoporosis. Which teaching point is correct? a) Obtaining the recommended daily allowance of calcium requires taking a calcium supplement. b) The recommended daily allowance of calcium may be found in a wide variety of foods. c) Obtaining an X-ray of the bones every 3 years is recommended to detect bone loss. d) To prevent fractures, the client should avoid strenuous exercise

b) The recommended daily allowance of calcium may be found in a wide variety of foods. Explanation: Premenopausal women require 1,000 mg of calcium per day. Postmenopausal women require 1,500 mg per day. Clients usually can get the recommended daily requirement of calcium by eating a varied diet. Osteoporosis doesn't show up on ordinary X-rays until 30% of bone has been lost. Bone densitometry, however, can detect bone loss of 3% or less. This test is sometimes recommended routinely for women older than 35 who are at risk for osteoporosis. Strenuous exercise won't cause fractures. Although supplements are available, they aren't always necessary. pg.1143

Identify methods to specifically prevent osteoporosis in postmenopausal women (select all that apply). a. Eating more beef b. Eating 8 ounces of yogurt daily c. Performing weight-bearing exercise d. Spending 15 minutes in the sun each day e. Taking postmenopausal estrogen replacement

b. Eating 8 ounces of yogurt daily. c. Performing weight-bearing exercise. To specifically prevent osteoporosis in postmenopausal women, increased calcium and vitamin D intake and weight-bearing exercises (i.e., walking) are the best methods. Beef is not high in calcium or vitamin D. Although 20 minutes in the sun each day provides vitamin D for most women, it is recommended that postmenopausal women take supplemental vitamin D doses of 800 to 1000 IU per day. Although estrogen replacement will protect the woman against bone loss and fractures, it is no longer given specifically to prevent osteoporosis because of increased risk of heart disease and breast or uterine cancer.

Which medication classification is prescribed when allergy is a factor causing the skin disorder? a) Antibiotics b) Local anesthetics c) Antihistamines d) Corticosteroids

c) Antihistamines Explanation: Antihistamines are frequently prescribed when an allergy is a factor in causing the skin disorder. They relieve itching and shorten the duration of allergic reaction. Corticosteroids are used to relieve inflammatory or allergic symptoms. Antibiotics are used to treat infectious disorders. Local anesthetics are used to relieve minor skin pain and itching. pg.1040

Which medication directly inhibits osteoclasts thereby reducing bone loss and increasing BMD? a) Teriparatide (Forteo) b) Vitamin D c) Calcitonin (Miacalcin) d) Raloxifene (Evista)

c) Calcitonin (Miacalcin) Explanation: Miacalcin directly inhibits osteoclasts, thereby reducing bone loss and increased BMD. Evista reduces the risk of osteoporosis by preserving BMD without estrogenic effects on the uterus. Forteo has been recently approved by the FDA for the treatment of osteoporosis. pg.1143

When describing malignant bone tumors to a group of students, which of the following would the instructor cite as the usual location? a) Wrist-hand junction b) Femur-hip area c) Distal femur around the knee d) Proximal humerus

c) Distal femur around the knee Explanation: Malignant bone tumors usually are located around the knee in the distal femur or proximal fibula; a few are found in the proximal humerus. The wrist-hand junction and femur-hip area are not common sites. pg.1151

The client diagnosed with osteosarcoma is scheduled for a surgical amputation. Which nursing diagnosis would be a priority for this client compared with other surgical clients? a) Impaired physical mobility b) Risk for infection c) Disturbed body image d) Inadequate nutrition

c) Disturbed body image Explanation: Amputation of a body part can result in disturbances in body image. pg.1153

The nurse teaches the patient with a high risk for osteoporosis about risk-lowering strategies including which of the following actions? a) Decrease the intake of vitamin A and D b) Reduce stress c) Walk or perform weight-bearing exercises outdoors d) Increase fiber in the diet

c) Walk or perform weight-bearing exercises outdoors Explanation: Risk-lowering strategies for osteoporosis include walking or exercising outdoors, performing a regular weight-bearing exercise regimen, increasing dietary calcium and vitamin D intake, smoking cessation, and consuming alcohol and caffeine consumption in moderation. pg.1145

A client with suspected osteomalacia has a fractured tibia and fibula. What test would give a definitive diagnosis of osteomalacia? a) Elevated levels of alkaline phosphatase b) Increased and decreased areas of bone metabolism c) Demineralization of the bone d) A bone biopsy

d) A bone biopsy Explanation: A definitive diagnosis is obtained by bone biopsy. Radiographic studies demonstrate demineralization of the bone. A bone scan detects increased and decreased areas of bone metabolism. Alkaline phosphatase levels are detected from a blood sample. pg.1146

The nurse is caring for patient with a hip fracture. The physician orders the patient to start on a bisphosphonate. Which medication would the nurse document as given? a) Raloxifene (Evista) b) Teriparatide (Forteo) c) Denosumab (Prolia) d) Alendronate (Fosamax)

d) Alendronate (Fosamax) Explanation: Alendronate (Fosamax) is a bisphosphonate medication. Raloxifene (Evista) is a selective estrogen receptor modulator. Terparatide (Forteo) is an anabolic agent, and denosumab (Prolia) is a monoclonal antibody agent. pg.1144

The nurse is preparing a client for a surgical procedure that will allow visualization of the extent of joint damage of the knee for a client with rheumatoid arthritis and also obtain a sample of synovial fluid. What procedure will the nurse prepare the client for? a) Arthroplasty b) Open reduction c) Needle aspiration d) Arthroscopy

d) Arthroscopy Explanation: Arthroscopic examination may be carried out to visualize the extent of joint damage as well as to obtain a sample of synovial fluid. An open reduction would be used for the treatment of a fracture. Needle aspiration will not allow visualization of the joint damage but will allow obtaining the sample of synovial fluid. Arthroplasty is the restructure of the joint surface after diagnosis is made. pg.1100

4. The nurse's responsibility for a patient with a suspected disc herniation who is experiencing acute pain and muscle spasms is a.encouraging total bed rest for several days. b.teaching the principles of back strengthening exercises. c.stressing the importance of straight-leg raises to decrease pain. d.promoting the use of cold and hot compresses and pain medication.

d.promoting the use of cold and hot compresses and pain medication. If the acute muscle spasms and accompanying pain are not severe and debilitating, the patient may be treated on an outpatient basis with nonsteroidal antiinflammatory drugs (NSAIDs; e.g., acetaminophen) and muscle relaxants (e.g., cyclobenzaprine [Flexeril]). Massage and back manipulation, acupuncture, and the application of cold and hot compresses may help some patients. Severe pain may necessitate a brief course of opioid analgesics. A brief period (1 to 2 days) of rest at home may be necessary for some people; most patients do better with a continuation of their regular activities. Prolonged bed rest should be avoided. All patients during this time should refrain from activities that aggravate the pain, including lifting, bending, twisting, and prolonged sitting.

Which of the following medical conditions or med- ications can lead to secondary osteoporosis? (A) Hypothyroidism (B) Zinc deficiency (C) Type 2 diabetes mellitus (D) Gonadotropin- releasing hormone (GnRH) agonists (E) Angiotensin- converting enzyme (ACE) inhibitors

(D) GnRH agonists. Osteoporosis can occur sec- ondary to any condition or medication that causes accelerated bone loss. 2 Conditions that may cause secondary osteoporosis include athletic amenor- rhea, anorexia nervosa, hyperthyroidism, type 1 dia- betes mellitus, and malabsorption syndromes. Med- ications that can cause secondary osteoporosis include excessive use of thyroid hormone, oral corti- costeroids (taken for > 3 months' duration), GnRH agonists, phenytoin, phenothiazines, long - term heparin, and phenobarbital. ACE inhibitors and zinc deficiency do not cause osteoporosis.

The nurse is assessing a client's left leg for neurovascular changes following a total left knee replacement. Which of the following are expected normal findings? Select all that apply. 1. Reduced edema of the left knee. 2. Skin warm to touch. 3. Capillary refill response. 4. Moves toes. 5. Pain absent. 6. Pulse on left leg weaker than right leg.

1, 2, 3, 4. Postoperatively, the knee in a total knee replacement is dressed with a compression bandage and ice may be applied to control edema and bleeding. Recurrent assessment by the nurse for neurovascular changes can prevent loss of limb. Normal neurovascular findings include: color normal, extremity warm, capillary refill less than 3 seconds, moderate edema, tissue not palpably tense, pain controllable, normal sensations, no paresthesia, normal motor abilities, no paresis or paralysis, and pulses strong and equal.

A patient with a compound fracture of the left femur is admitted to the emergency department after a motorcycle crash. Which action is most essential for the nurse to take first? 1 Check the dorsalis pedis pulses. 2 Immobilize the left leg with a splint. 3 Administer the prescribed analgesic.

1. Check the dorsalis pedis pulses. The first action should be to assess the circulatory status of the leg because the patient is at risk for acute compartment syndrome (ACS), which can begin as early as 6-8 hours after an injury. Severe tissue damage can also occur if neurovascular status is compromised. Immobilization will be needed, but the nurse must assess the patient's condition first. Administering an analgesic and placing a dressing on the affected area should both be done after the nurse has assessed the patient.

11. A laminectomy and spinal fusion are performed on a patient with a herniated lumbar intervertebral disc. During the postoperative period, which finding is of most concern to the nurse? a. Paralytic ileus b. Urinary incontinence c. Greater pain at the graft site than at the lumbar incision site d. Leg and arm movement and sensation unchanged from preoperative status

11. b. Urinary incontinence following spinal surgery may indicate nerve damage and should be reported to the health care provider. Paralytic ileus is common following surgery and is expected. Pain at the graft site, usually the iliac crest or the fibula, often is more severe than pain at the fused area. Although movement and sensation of the arms and legs should not be more impaired than before surgery, they often are not relieved immediately after surgery.

Which female patients are at risk for developing osteoporosis (select all that apply)? a. 60-year-old white aerobics instructor b. 55-year-old Asian American cigarette smoker c. 62-year-old African American on estrogen therapy d. 68-year-old white who is underweight and inactive e. 58-year-old Native American who started menopause prematurely

17. b, d, e. Risk factors for osteoporosis include age greater than 65, white or Asian ethnicity, cigarette smoking, inactive lifestyle, low body weight, and postmenopausal estrogen deficiency including premature menopause. Other factors include family history, excessive alcohol use, long-term use of medications such as corticosteroids, thyroid replacement, heparin, long-acting sedatives, or antiseizure drugs.

The nurse is assigned to care for several patients on a medical unit. Which patient should the nurse check on first? A 60-yr-old patient with a blood pressure of 92/64 mm Hg and hemoglobin of 9.8 g/dL A 50-yr-old patient with a respiratory rate of 26 breaths/minute and an elevated D-dimer A 40-yr-old patient with a temperature of 100.8F (38.2C) and a neutrophil count of 256/μL A 30-yr-old patient with a pulse of 112 beats/min and a white blood cell count of 14,000/μL

A 40-yr-old patient with a temperature of 100.8F (38.2C) and a neutrophil count of 256/μL

Which individuals would be at high risk for low back pain (select all that apply)? a.A 63-year-old man who is a long-distance truck driver b.A 36-year-old 6 ft, 2 in construction worker who weighs 260 lb c.A 28-year-old female yoga instructor who is 5 ft, 6 in and weighs 130 lb d.A 30-year-old male nurse who works on an orthopedic unit and smokes e.A 44-year-old female chef with prior compression fracture of the spine

A, B, D, E Risk factors associated with low back pain include a lack of muscle tone and excess body weight, stress, poor posture, cigarette smoking, pregnancy, prior compression fractures of the spine, spinal problems since birth, and a family history of back pain. Jobs that require repetitive heavy lifting, vibration (such as a jackhammer operator), and prolonged periods of sitting are also associated with low back pain. Low back pain is most often caused by a musculoskeletal problem. The causes of low back pain of musculoskeletal origin include (1) acute lumbosacral strain, (2) instability of the lumbosacral bony mechanism, (3) osteoarthritis of the lumbosacral vertebrae, (4) degenerative disc disease, and (5) herniation of an intervertebral disc. Health care personnel are at high risk for the development of low back pain. Lifting and moving patients, excessive time being stooped over or leaning forward, and frequent twisting can result in low back pain.

Which statement by a patient who is scheduled for an above-the-knee amputation for treatment of an osteosarcoma of the right tibia indicates that patient teaching is needed? a. "I did not have this bone cancer until my leg broke a week ago." b. "I wish that I did not have to have chemotherapy after this surgery." c. "I know that I will need to participate in physical therapy after surgery." d. "I will use the patient-controlled analgesia (PCA) to control postoperative pain."

ANS: A Osteogenic sarcoma may be diagnosed following a fracture, but it is not caused by the injury. The other patient statements indicate that patient teaching has been effective.

When caring for a patient with gout and a red and painful left great toe, which nursing action will be included in the plan of care? a. Gently palpate the toe to assess swelling. b. Use pillows to keep the left foot elevated. c. Use a footboard to hold bedding away from the toe. d. Teach patient to avoid use of acetaminophen (Tylenol).

ANS: C Since any touch on the area of inflammation may increase pain, bedding should be held away from the toe and touching the toe will be avoided. Elevation of the foot will not reduce the pain, which is caused by the urate crystals. Acetaminophen can be used for pain relief.

4. A patient has chronic osteomyelitis of the left femur, which is being managed at home with self-administration of IV antibiotics. On a home visit, the nurse identifies the nursing diagnosis of ineffective therapeutic regimen management when the nurse finds that the patient a. is unable to plantar-flex the foot on the affected side. b. uses crutches to avoid weight bearing on the affected leg. c. takes and records the oral temperature twice a day. d. is irritable and frustrated with the length of treatment required.

Answer: A Rationale: Footdrop is an indication that the foot is not being supported in a neutral position by a splint. Using crutches and monitoring the oral temperature are appropriate self-care activities. Frustration with the length of treatment is not an indicator of ineffective management of the osteomyelitis. Cognitive Level: Application Text Reference: p. 1672 Nursing Process: Diagnosis NCLEX: Health Promotion and Maintenance

The nurse prepares to perform a neurovascular assessment on a patient with closed multiple fractures of the right humerus. Which technique does the nurse use?

Assess sensation of the right upper extremity. Assessing sensation of the right upper extremity is part of a focused neurovascular assessment for the patient with multiple fractures of the right humerus. Inspecting the abdomen and auscultating lung sounds of the patient with multiple fractures are not part of a focused neurovascular assessment. Because the patient does not have a head injury, assessing the patient's level of consciousness and ability to follow commands is not part of a focused neurovascular assessment.

Which does a closed reduction involve?

Closed reduction involves applying a manual pull or traction to move the ends of a fractured bone to realign them. This is the most common nonsurgical method for managing a simple fracture.

Radiographic assessment of a patient with femoral fracture reveals spotty and diffuse osteoporosis. The patient gives a history of excessive sweating, intense burning pain, and edema. Which side effect of fractures does the nurse suspect?

Complex regional pain syndrome (CRPS) CRPS is actually a dysfunction of the central and peripheral nervous systems that leads to severe, chronic pain. Spotty and diffuse osteoporosis can be seen on x-ray examination, which is characteristic of CRPS. The symptoms may include intense burning pain, muscle spasms, and raised temperature. **the priority intervention in CRPS management is pain relief so that the patient can resume normal activities** CRPS might necessitate the use of an implanted device to block pain perception.

A patient arrives in the emergency department with ankle swelling and severe pain after twisting the ankle playing soccer. All of the following orders are written by the health care provider. Which one will the nurse act on first? a. Administer naproxen (Naprosyn) 500 mg PO. b. Wrap the ankle and apply an ice pack. c. Give acetaminophen with codeine (Tylenol #3). d. Take the patient to the radiology department for x-rays.

Correct Answer: B Rationale: Immediate care after a sprain or strain injury includes the application of cold and compression to the injury to minimize swelling. The other actions should be taken after the ankle is wrapped with a compression bandage and ice is applied. Cognitive Level: Application Text Reference: p. 1631 Nursing Process: Implementation NCLEX: Physiological Integrity

12. Following a motor-vehicle accident, a patient arrives in the emergency department with massive right lower-leg swelling. Which action will the nurse take first? a. Elevate the leg on pillows. b. Apply a compression bandage. c. Place ice packs on the lower leg. d. Check leg pulses and sensation.

Correct Answer: D Rationale: The initial action by the nurse will be to assess the circulation to the leg and to observe for any evidence of injury such as fractures or dislocations. After the initial assessment, the other actions may be appropriate based on what is observed during the assessment.

41. Which assessment finding in a patient who has undergone a bone graft for a tumor does the nurse report to the health care provider immediately? a. Extremity distal to the operative site is warm and pink. b. Cast over the operative site is cool to the touch. c. Delayed capillary refill presents in digits distal to the site. d. Pain is present in the operative extremity.

Delayed capillary refill presents in digits distal to the site.

38. The nurse is seeing a 49-year-old man in the clinic for left mid-tibia tenderness for the past 3 months. What type of malignant bone tumor might this patient have? a. Chondrosarcoma b. Ewing's sarcoma c. Fibrosarcoma d. Osteosarcoma

Fibrosarcoma

The nurse is preparing to teach a client who is diagnosed with fibromyalgia regarding nonpharmacologic therapies for treating the disorder. Which goals of therapy should the nurse include in the teaching​ session? ​(Select all that​ apply.) Enhancing​ self-efficacy and positive outlook Curing the disorder Improving activity tolerance Relieving pain Helping the client to understand they are imagining their symptoms

Improving activity tolerance Enhancing​ self-efficacy and positive outlook Relieving pain Clients with fibromyalgia experience real pain and fatigue that are disruptive to their daily lives. Suggesting they are imagining these things will increase frustration and is not therapeutic. There is no known cure. Clients gain great benefit form activities that increase conditioning and activity​ tolerance, enhance feelings of​ self-efficacy, and assist with pain control.

Which of the following assessment findings indicate to the nurse that a client may have peripheral neurovascular dysfunction? Absence of feeling Capillary refill of 4 to 5 seconds Cool skin Pain Redness of the skin Weakness in motion

Indicators of peripheral neurovascular dysfunction include pale, cyanotic or mottled skin with a cool temperature, capillary refill greater than 3 seconds, weakness or paralysis with motion, and paresthesia, unrelenting pain, pain on passive stretch, or absence of feeling.

A diabetic older adult patient who had arthroscopic surgery on the right knee the previous day has a red, swollen, and painful right knee. The nurse anticipates that the health care provider will request which medication?

Levofloxacin The patient's symptoms indicate a possible right knee infection, so the first action will be to start antibiotic therapy, especially because the patient is diabetic and is at greater risk for infection.

What are some risk factors for osteoporosis?

Lifestyle risk factors for osteoporosis include lack of exposure to sunshine, a diet low in calcium and vitamin D, cigarette smoking, consumption of alcohol and/or caffeine, and lack of weight-bearing exercise. Lack of weight-bearing exercise, not aerobic exercise, is a lifestyle risk factor for osteoporosis. A diet low in calcium and vitamin D, not a low-protein, high-fat diet, is a lifestyle risk factor for osteoporosis. An estrogen deficiency or menopause is an individual risk factor for osteoporosis. Other individual risk factors include female gender, non-Hispanic white or Asian race, increased age, low weight and body mass index, family history of osteoporosis, low initial bone mass, and contributing coexisting medical conditions and medications.

A patient has undergone a kyphoplasty (Use of a balloon in the vertebral body to contain bone cement). What discharge teaching does the nurse provide to the patient and family?

Monitor the puncture site for infection. Nursing intervention: Applying ice pack to the insertion site

A diabetic patient is admitted to the health care facility with a foot ulcer. The nurse teaches wound care to the patient and the caregiver to prevent the risk for which condition?

Osteomyelitis The diabetic patient with a foot ulcer is at high risk for osteomyelitis or bone infection. Diabetes also slows down the healing process. As a person ages, the cartilages at the synovial joints lose their elasticity and become compressible which leads to osteoarthritis. Joint dislocations and joint traumas also lead to osteoarthritis. Osteoporosis may occur due to age-related bone loss, or decreased intake of calcium and vitamin D. Osteomalacia or softening of the bone is caused by the deficiency of vitamin D in the body.

30. A 40-year-old patient is admitted for acute osteomyelitis of the left lower leg. What does the nurse expect to find documented in the patient's admitting assessment? a. Temperature greater than 101° F; swelling, tenderness, erythema, and warmth of area b. Ulceration resulting with sinus tract formation, localized pain, and drainage c. Pain is aching, poorly described, deep, and worsened by pressure and weight bearing d. Shortening of the extremity with pain during weight bearing or palpation

Temperature greater than 101° F; swelling, tenderness, erythema, and warmth of area

A 63 year old patient has severe osteoarthritis in the right knee. The patient is scheduled for a knee osteotomy. You are providing pre-op teaching about this procedure to the patient. Which statement made by the patient is correct about this procedure?* A. "This procedure will realign the knee and help decrease the amount of weight experienced on my right knee." B. "A knee osteotomy is also called a total knee replacement." C. "A knee osteotomy is commonly performed for patients who have osteoarthritis in both knees." D. "This procedure will realign the unaffected knee and help alleviate the amount of weight experienced on the right knee."

The answer is A. A knee osteotomy is NOT known as a total knee replacement. A knee osteotomy can be used as an alternative for a total knee replacement but is not the same thing. In addition, a knee osteotomy is performed when there is OA on only one side of the knee.

Your patient is scheduled for a DEXA scan this morning. The patient is having heartburn and requests a PRN medication to help with relief. Which medications can the patient NOT have at this time? A. Calcium Carbonate B. Bismuth Salicylate C. Milk of Magnesia d. Famotidine

The answer is A. Before a DEXA scan, which is a bone density test, the patient should not take any type of calcium supplements (calcium carbonate (TUMs) or vitamins containing calcium.

D Emphasis is placed on postoperative exercise of the affected leg, with isometric quadriceps setting beginning on the first day after surgery. Vitamin C and calcium do not improve muscle strength, but they will facilitate healing. The patient should be able to perform active range of motion to all joints. Keeping the leg in one position (extension and abduction) may contribute to contractures.

The nurse formulates a nursing diagnosis of Impaired physical mobility related to decreased muscle strength for an older adult patient recovering from left total knee arthroplasty. What nursing intervention is appropriate? Promote vitamin C and calcium intake in the diet. Provide passive range of motion to all of the joints q4hr. Keep the left leg in extension and abduction to prevent contractures. Encourage isometric quadriceps-setting exercises at least four times a day.

C The patient must be free of infection before total knee arthroplasty. An infection in the joint could lead to even greater pain and joint instability, requiring more extensive surgery. The nurse must assess the patient for signs of infection, such as redness, swelling, fever, and elevated white blood cell count. Pain, knee stiffness, or instability are typical of osteoarthritis.

The nurse is caring for a patient with osteoarthritis scheduled for total left knee arthroplasty. Preoperatively, the nurse assesses for which contraindication to surgery? Pain Left knee stiffness Left knee infection Left knee instability

A healthcare provider prescribes duloxetine​ (Cymbalta) for a client diagnosed with fibromyalgia. The client asks the nurse why this medication is being prescribed. The nurse will base the response on which​ rationale? To reduce neuropathic pain To increase levels of dopamine and serotonin To decrease joint pain and swelling To relax the client and promote sleep

To increase levels of dopamine and serotonin. Duloxetine​ (Cymbalta) is prescribed to a client with fibromyalgia to increase serotonin and norepinephrine levels. This medication is not prescribed to reduce neuropathic​ pain, decrease swelling to​ joints, or relax the client to promote sleep. Pregabalin​ (Lyrica) is prescribed to reduce neuropathic pain. Nonsteroidal​ anti-inflammatory drugs are prescribed to decrease swelling to joints. Fluoxetine​ (Prozac) and paroxetine​ (Paxil) are medications prescribed to promote sleep.

The nurse is planning discharge instructions for the client with osteomyelitis. What instructions should the nurse include in the discharge teaching? a) "You will receive IV antibiotics for 3 to 6 weeks." b) "Use your continuous passive motion machine (CPM) 2 hours each day." c) "You need to perform weight-bearing exercises twice a week." d) "You need to limit the amount of protein and calcium in your diet."

a) "You will receive IV antibiotics for 3 to 6 weeks." Explanation: Treatment of osteomyelitis requires IV antibiotics for 3 to 6 weeks. pg.1148

5. Which factors and/or patient data may be associated with osteomalacia? (Select all that apply.) a. Vitamin D deficiency b. Drinks 8 cups of coffee per day c. Muscle weakness in the pelvic girdle area d. Bone pain worsened by walking e. Muscle weakness in legs

Vitamin D deficiency Muscle weakness in the pelvic girdle area Muscle weakness in legs

The nurse is teaching a client newly diagnosed with osteoporosis about dietary and lifestyle interventions to decrease risk factors for osteoporosis. Which is the best way to decrease the risk for osteoporosis? A. Increase nutritional intake of calcium. B. Engage in high-impact exercise, such as running. C. Increase nutritional intake of phosphorus. D. Walk for 30 minutes three times a week.

Walk for 30 minutes three times a week. Walking for 30 minutes three to five times a week is the single most effective exercise for osteoporosis prevention. Walking is a safe way to promote weight-bearing and muscle strength.

CDE Appropriate care for a sprain is represented with the acronym RICE (rest, ice, compression, and elevation). Antiinflammatory medication should be used to decrease swelling if not contraindicated for the patient. After the injury, the ankle should be immobilized and rested. Prolonged immobilization is not required unless there is significant injury. Ice is indicated, but will cause tissue damage if applied directly to the skin. Apply ice to sprains as soon as possible and leave in place for 20 to 30 minutes at a time. Moist heat may be applied 24 to 48 hours after the injury.

When entering the grocery store, a patient trips on the curb and sprains the right ankle. Which initial care is appropriate (select all that apply.)? Apply ice directly to the skin. Apply heat to the ankle every 2 hours. Administer antiinflammatory medication. Compress ankle using an elastic bandage. Rest and elevate the ankle above the heart. Perform passive and active range of motion.

A patient with chronic osteomyelitis has undergone 6 weeks of antibiotic therapy. There is no improvement in the wound appearance. What action would the nurse anticipate to promote healing? a) Surgical debridement b) Vitamin supplements c) Wound irrigation d) Wound packing

a) Surgical debridement Explanation: In chronic osteomyelitis, surgical debridement is used when the wound fails to respond to antibiotic therapy. Wound packing, vitamin supplements, and wound irrigation are not the standard of care when treating chronic osteomyelitis. pg.1149

A nurse is performing discharge teaching for an elderly client with osteoporosis. Which instruction about taking a calcium supplement should the nurse include? a) Take the supplement with meals or with orange juice. b) Take the supplement on an empty stomach with a full glass of water. c) Remain in an upright position 30 minutes after taking the supplement. d) Take weekly on the same day and at the same time.

a) Take the supplement with meals or with orange juice. Explanation: Calcium supplements, such as Caltrate or Citracal, are over-the-counter medications. They should be taken with meals or with a beverage high in vitamin C. pg.1144

A client has been treated for migraine headaches for several months and comes to the clinic stating he is getting no better. The nurse is talking with the client and hears an audible click when the client is moving his jaw. What does the nurse suspect may be happening? a) Temporomandibular disorder b) Trigeminal neuralgia c) Loose teeth d) Dislocated jaw

a) Temporomandibular disorder Explanation: The disorder can be confused with trigeminal neuralgia and migraine headaches. The client experiences clicking of the jaw when moving the joint, or the jaw can lock, which interferes with opening the mouth. Loose teeth will not cause a clicking of the jaw. The client does not have a dislocated jaw. pg.1236

Instructions for the patient with low back pain include that when lifting the patient should a) use a narrow base of support. b) avoid overreaching. c) bend the knees and loosen the abdominal muscles. d) place the load away from the body.

b) avoid overreaching. Explanation: Instructions for the patient with low back pain should include that when lifting, the patient should avoid overreaching. The patient should also keep the load close to the body, bend the knees and tighten the abdominal muscles, use a wide base of support, and use a back brace to protect the back. When lifting, the patient with low back pain should keep the load close to the body. When lifting, the patient with low back pain should bend the knees and tighten the abdominal muscles. pg.1134

A patient comes to the clinic complaining of low back pain radiating down the left leg. After diagnostic studies rule out any pathology, the physician orders a serotonin-norepinephrine reuptake inhibitor (SNRI). Which medication does the nurse anticipate educating the patient about? a) Cyclobenzaprine (Flexeril) b) Amitriptyline (Elavil) c) Duloxetine (Cymbalta) d) Gabapentin (Neurontin)

c) Duloxetine (Cymbalta) Explanation: Nonprescription analgesics such as acetaminophen (Tylenol) and nonsteroidal anti-inflammatory drugs (NSAIDs) and short-term prescription muscle relaxants (e.g., cyclobenzaprine [Flexeril]) are effective in relieving acute low back pain. Tricyclic antidepressants (e.g., amitriptyline [Elavil) and the newer dual-action serotonin-norepinephrine reuptake inhibitors (e.g., duloxetine [Cymbalta]) (Karp et al., 2010) or atypical seizure medications (e.g., gabapentin [Neurontin], which is prescribed for pain from radiculopathy) are used effectively in chronic low back pain. pg.1133

Which assessment findings would the nurse expect in a client diagnosed with osteomyelitis? a) Pruritus and uremic frost b) Petechiae over the chest and abnormal ABGs c) Leukocytosis and localized bone pain d) Thrombocytopenia and ecchymosis

c) Leukocytosis and localized bone pain Explanation: Clinical manifestations of osteomyelitis include signs and symptoms of sepsis and localized infection. pg.1148

Which group is at the greatest risk for osteoporosis? a) Asian women b) Men c) African American women d) Caucasian women

d) Caucasian women Explanation: Small-framed, nonobese Caucasian women are at greatest risk for osteoporosis. Asian women of slight build are at risk for low peak BMD. African American women, who have a greater bone mass that Caucasian women, are less susceptible to osteoporosis. Men have a greater peak bone mass and do not experience sudden estrogen reduction. pg.1141

Ms. Simpson has come to the clinic with foot pain. The physician has described her problem as a flexion deformity of the proximal interphalangeal joint. What is the name of this disorder? a) Mallet toe b) Hallux valgus (bunion) c) Heberden's nodes d) Hammer toe

d) Hammer toe Explanation: Hammer toe is a flexion deformity of the proximal interphalangeal joint. Mallet toe is a flexion deformity of the distal interphalangeal joint. Bunion is a deformity of the great toe at its metatarsophalangeal joint. Heberden's nodes are bony enlargements of the distal interphalangeal joints. This is a finding in degenerative joint disease. pg.1139

What antibiotic do patients take for chronic osteomyelitis?

oral fluoroquinolone (cipro)

When taking a health​ history, which statement from the client would support a diagnosis of​ fibromyalgia? ​"I have experienced persistent pain in my​ knees, ankles, and​ feet." ​"I have had generalized pain throughout my body for at least 3 to 4​ months." ​"My joints are stiff when I get up in the​ mornings." ​"I have experienced heart palpitations and a fluttering sensation for the past few​ weeks."

​"I have had generalized pain throughout my body for at least 3 to 4​ months." Heart palpitations are not associated with fibromyalgia. Difficulty​ sleeping, morning stiffness and knee pain may result from fibromyalgia but the duration of symptoms for greater than 3 months is a diagnostic criterion.

Karen​ O'Connell is a young woman who recently graduated from college and is working at her first position as an architect. Up to this​ time, she has been in very good health and has led an active lifestyle. She enjoys​ hiking, cycling, and jogging. During your​ assessment, Ms.​ O'Connell states that she is frequently tired and has achy pain in her​ neck, back, and hips. You suspect fibromyalgia. When questioned​ further, how would you expect Karen to describe her fatigue and​ pain? ​"The tiredness and pain seem to be relieved after I do some light​ activity." ​"I am not able to determine a pattern with the pain and​ fatigue." ​"The pain and fatigue are greatest in the early​ evening." ​"The pain and tiredness are worse when I am very​ active."

​"The pain and tiredness are worse when I am very​ active." Fibromyalgia symptoms follow a specific pattern that includes morning stiffness and acute fatigue following increased activity. The other options do not adequately describe the pain associated with fibromyalgia.

The nurse instructs an African American man who has sickle cell disease about symptom management and prevention of sickle cell crisis. The nurse determines further teaching is necessary if the patient makes which statement? "When I take a vacation, I should not go to the mountains." "I should avoid contact with anyone who has a respiratory infection." "When my vision is blurred, I will close my eyes and rest for an hour." "I may experience severe pain during a crisis and need narcotic analgesics."

"When my vision is blurred, I will close my eyes and rest for an hour."

A 50 - year - old African American woman visits your office for an annual examination. She takes inhaled steroids and Beta- agonists for mild intermittent asthma. Her past medical history is negative for any fracture. She has no family history of osteoporosis. She exercises regularly. Her menses are regular. Based on her history, what are your recommendations for osteoporosis prevention and screening? (A) Take 800 mg of calcium and 800 IU vitamin D daily (B) Take 1200 mg of calcium and 400 IU vitamin D daily (C) Bone mineral density (BMD) screening (D) Begin calcitonin therapy to increase bone density (E) Begin alendronate therapy to increase bone density

(B) Take 1200 mg of calcium and 400 IU vitamin D daily. This patient has no risk factors for osteoporosis. Although oral steroids are a recognized cause of increased bone loss in adults, inhaled steroids are not. The National Osteoporosis Foundation (NOF)advises that all individuals consume at least 1200 mg of calcium and 400 to 800 IU vitamin D daily. The US Preventive Services Task Force recommends that physicians conduct routine BMD screening for all women older than 65 years. For women with risk factors for osteoporosis, screening is recommended beginning at age 60 years. Alendronate is a Food and Drug Administration (FDA)-approved medication for prevention and treatment of osteoporosis. Calcitonin is approved for treatment of osteoporosis only, not prevention.

56. Which statements about scoliosis are true? (Select all that apply.) a. Scoliosis screening is best done during middle-school years. b. Scoliosis is characterized by a C- or S shaped lateral curvature of the vertebral spine. c. Scoliosis screening is best done during preschool years. d. A curvature greater than 50 degrees results in an unstable spine. e. The forward-bend test is used to determine scoliosis.

- Scoliosis screening is best done during middle-school years. - Scoliosis is characterized by a C- or S shaped lateral curvature of the vertebral spine. - A curvature greater than 50 degrees results in an unstable spine. - The forward-bend test is used to determine scoliosis.

A The casted extremity should be elevated at or above heart level for 24 hours to reduce swelling or inflammation. The cast should be supported on pillows during the drying period to prevent denting and flattening of the cast. Ice (not heat) should be applied for the first 24 to 36 hours to reduce swelling or inflammation. Active movement of the thumb and fingers should be encouraged to reduce edema and increase venous return. A sling may be used to support and protect the extremity after the cast is completely dry, but the patient should perform active movements of the shoulder to prevent stiffness or contracture.

A 19-yr-old male patient has a plaster cast applied to the right arm for a Colles' fracture after a skateboarding accident. Which nursing action is most appropriate? Elevate the right arm on two pillows for 24 hours. Apply heating pad to reduce muscle spasms and pain. Limit movement of the thumb and fingers on the right hand. Place arm in a sling to prevent movement of the right shoulder.

C The patient does not understand the severity of ACL reconstructive surgery if planning to resume playing soccer soon; safe return will not occur for 6 to 8 months. Initial range of motion, immobilization, and progressive weight bearing will be overseen by a physical therapist.

A 21-yr-old soccer player has injured the anterior crucial ligament (ACL) and is having reconstructive surgery. Which patient statement indicates more teaching is required? "I probably won't be able to play soccer for 6 to 8 months." "They will have me do range of motion with my knee soon after surgery." "I can't wait to get this done now so I can play soccer for the next tournament." "I will need to wear an immobilizer and progressively bear weight on my knee."

The nurse provides instructions to a 30-year-old female office worker who has low back pain. Which statement by the patient requires an intervention by the nurse? A. "Acupuncture to the lower back would cause irreparable nerve damage." B. "Smoking may aggravate back pain by decreasing blood flow to the spine." C. "Sleeping on my side with knees and hips bent reduces stress on my back." D. "Switching between hot and cold packs provides relief of pain and stiffness."

A. "Acupuncture to the lower back would cause irreparable nerve damage." Acupuncture is a safe therapy when the practitioner has been appropriately trained. Very fine needles are inserted into the skin to stimulate specific anatomic points in the body for therapeutic purposes.

Which of these nursing actions included in the care of a patient after laminectomy can the nurse delegate to experienced nursing assistive personnel (NAP)? a. Ask about pain control with the patient-controlled analgesia (PCA). b. Determine the patient's readiness to ambulate. c. Check ability to plantar and dorsiflex the foot. d. Turn the patient from side to side every 2 hours.

ANS: D Repositioning a patient is included in the education and scope of practice of NAP, and experienced NAP will be familiar with how to maintain alignment in the postoperative patient. Evaluation of the effectiveness of pain medications, assessment of neurologic function, and evaluation of a patient's readiness to ambulate after surgery require higher level nursing education and scope of practice.

A client is diagnosed with osteoporosis. Which statements should the nurse include when teaching the client about the disease? SELECT ALL THAT APPLY a)"It's common in females after menopause." b) "It's a degenerative disease characterized by a decrease in bone density." c) "It's a congenital disease caused by poor dietary intake of milk products." d) "It can cause pain and injury." e) "Passive range-of-motion exercises can promote bone growth." f) "Weight-bearing exercise should be avoided."

Answer: a, b, d Osteoporosis is a degenerative metabolic bone disorder in which the rate of bone resorption accelerates and the rate of bone formation decelerates, thus decreasing bone density. Postmenopausal women are at increased risk for this disorder because of the loss of estrogen. The decrease in bone density can cause pain and injury. Osteoporosis isn't a congenital disorder; however, low calcium intake does contribute to the disorder. Passive range-of-motion exercises may be performed but they won't promote bone growth. The client should be encouraged to participate in weight-bearing exercise because it promotes bone growth.

Identify methods to specifically prevent osteoporosis in postmenopausal women (Select all that apply)? a-Eating more beef. b-Eating 8 oz. of yogurt daily. c-performing weight bearing exercises. d-Spending 15 minutes in the sun each day. e-Taking postmenopausal estrogen replacement.

Answers are: b,c To specifically prevent osteoporosis in postmenopausal women, increased calcium and vitamin D intake and weight bearing exercises (e.g. walking) are the best methods.

Which of the following type of fracture is associated with osteoporosis? Compression Stress Oblique Simple

Compression fractures are caused by compression of vertebrae and are associated frequently with osteoporosis. Stress fractures occur with repeated bone trauma from athletic activities, most frequently involving the tibia and metatarsals. An oblique fracture runs across the bone at a diagonal angle of 45 to 60 degrees. A simple fracture (closed fracture) is one that does not cause a break in the skin.

53. Which characteristics occur with the foot disorder hammertoe? (Select all that apply.) a. Great toe deviates laterally. b. Corns may develop on the dorsal side of the toe. c. First metatarsal head becomes enlarged. d. Insertion of wires or screws is required for fixation. e. Small tumor in a digital nerve of the foot.

Corns may develop on the dorsal side of the toe. Insertion of wires or screws is required for fixation.

The patient presents with contracture deformities of the hand and complains of severe pain. What musculoskeletal disorder does this patient manifest? 1. Rheumatoid arthritis 2. Osteomyelitis 3. Osteoporosis 4. Ankylosing spondylitis

Correct Answer: 1 Rationale The pattern of joint involvement in rheumatoid arthritis (RA) is typically polyarticular and symmetric. The proximal interphalangeal (PIP) and metacarpophalangeal (MCP) joints of the fingers, wrists, knees, ankles, and toes are most frequently involved, although RA can affect any joints.

The nurse is planning care for a patient who had an above-the-knee amputation 2 days ago. Which position should the nurse include in this patient's plan of care? 1. Sims' position as tolerated 2. Flat in bed 3. High Fowler's position with the stump elevated

Correct Answer: 2 Rationale 1: Sims' position is side-lying and would likely be uncomfortable for the patient. Rationale 2: Lying flat in bed keeps the hip extended, which helps to prevent contracture. Rationale 3: After 24 hours, the stump should not be elevated.

A patient is admitted to the emergency department with possible fractures of the bones of the left lower extremity. The initial action by the nurse should be to a. splint the lower leg. b. elevate the left leg. c. check the popliteal, dorsalis pedis, and posterior tibial pulses. d. obtain information about the patient's tetanus immunization status.

Correct Answer: C Rationale: The initial nursing action should be assessment of the neurovascular status of the injured leg. After assessment, the nurse may need to splint and elevate the leg, based on the assessment data. Information about tetanus immunizations should be done if there is an open wound. Cognitive Level: Application Text Reference: p. 1642 Nursing Process: Assessment NCLEX: Physiological Integrity

3. A checkout clerk in a grocery store has muscle and tendon tears that have become inflamed, causing pain and weakness in the left hand and elbow. The nurse identifies these symptoms as related to a. muscle spasms. b. meniscus injury. c. repetitive strain injury. d. carpal tunnel syndrome.

Correct Answer: C Rationale: The patient's occupation and the inflammation, pain, and weakness in the elbow and hand suggest a repetitive strain injury. Muscle spasms would be characterized by a palpable, firm muscle mass during the spasm. Meniscus injury would affect the knee. Carpal tunnel syndrome is characterized by weakness and numbness of the hand.

When counseling an older patient about ways to prevent fractures, which information will the nurse include? a. Tacking down scatter rugs in the home is recommended. b. Occasional weight-bearing exercise will improve muscle and bone strength. c. Most falls happen outside the home. d. Buying shoes that provide good support and are comfortable to wear is recommended.

Correct Answer: D Rationale: Comfortable shoes with good support will help to decrease the risk for falls. Scatter rugs should be eliminated, not just tacked down. Regular weight-bearing exercise will improve strength, but occasional exercise is not helpful in improving strength. Falls inside the home are responsible for many injuries. Cognitive Level: Application Text Reference: p. 1630 Nursing Process: Implementation NCLEX: Health Promotion and Maintenance

A patient comes to the clinic complaining of low back pain radiating down the left leg. After diagnostic studies rule out any pathology, the physician orders a serotonin-norepinephrine reuptake inhibitor (SNRI). Which medication does the nurse anticipate educating the patient about? Amitriptyline (Elavil) Duloxetine (Cymbalta) Gabapentin (Neurontin) Cyclobenzaprine (Flexeril)

Nonprescription analgesics such as acetaminophen (Tylenol) and nonsteroidal anti-inflammatory drugs (NSAIDs) and short-term prescription muscle relaxants (e.g., cyclobenzaprine [Flexeril]) are effective in relieving acute low back pain. Tricyclic antidepressants (e.g., amitriptyline [Elavil) and the newer dual-action serotonin-norepinephrine reuptake inhibitors (e.g., duloxetine [Cymbalta]) (Karp et al., 2010) or atypical seizure medications (e.g., gabapentin [Neurontin], which is prescribed for pain from radiculopathy) are used effectively in chronic low back pain.

The nurse is educating a client newly diagnosed with fibromyalgia about prescribed medications. Which medication will the nurse include in the teaching session that will assist in treating neuropathic pain and may reduce pain associated with​ fibromyalgia? Fluoxetine Tramadol Milnacipran Pregabalin

Pregabalin Pregabalin​ (Lyrica) is prescribed to a client with fibromyalgia to assist in treating neuropathic pain and to help in reducing pain associated with fibromyalgia. Tramadol​ (Ultram) is prescribed to a client with fibromyalgia to assist with pain​ relief, but it does not assist in treating neuropathic pain. Fluoxetine​ (Prozac) is prescribed to a client with​ fibromyalgia, but it does not assist in treating neuropathic pain or reduce pain associated with fibromyalgia. This medication promotes better sleep and helps in relieving other manifestations of fibromyalgia.Milnacipran​ (Savella) is prescribed to a client with​ fibromyalgia, but it does not assist in treating neuropathic pain or reduce pain associated with fibromyalgia. This medication mixes with reuptake inhibitors to increase serotonin and norepinephrine levels.

Which of the following should the nurse provide when explaining therapeutic measures to a client prescribed methotrexate (Rheumatrix) for rheumatoid arthritis (RA)? Fluids are restricted to prevent formation of edema in the joints. Drug doses will be adjusted for optimum effect at the lowest dose once relief has been established. Six months of therapy will be adequate to stop disease progression. Relief of symptoms will be assessed within 1 week of starting medication. Drug doses will be adjusted for optimum effect at the lowest dose once relief has been established.

Rationale: Methotrexate takes several weeks to effect relief. Once relief is obtained, the dose is adjusted to achieve maximum response at the lowest dose. If the drug is discontinued, the symptoms will reappear. Fluids are not restricted.

Parathyroid hormone plays an important role in bone health. When the parathyroid gland secretes PTH (parathyroid hormone) it causes:* A. the body to increase the calcium levels by stimulating the osteoclast activity. B. the body to decrease the calcium levels by inhibiting osteoclast activity. C. the body to increase the calcium levels by stimulating osteoblast activity. D. the body to decrease the calcium levels by inhibiting osteoblast activity.

The answer is A. When the calcium levels are low this stimulates the parathyroid gland to secrete PTH, which stimulates osteoCLAST activity. Remember osteoCLASTS break down the bone matrix within the spongy bone. This will cause calcium to enter the blood stream, hence increasing calcium levels.

A patient is prescribed Alendronate (Fosamax) at 0800 for the treatment of osteoporosis. As the nurse you know you must administer this medication: A. on an empty stomach with a full glass of water and keep the patient upright for 30 minutes. B. right after breakfast and to lay the patient flat (as tolerated) for 30 minutes. C. with food but to avoid giving this medication with dairy products. D. on an empty stomach with a full glass of juice or milk.

The answer is A. Alendronate (Fosamax) is a bisphosphonate which is known for causing GI upset, especially inflammation of the esophagus. These medications should be taken with a full glass of water in morning on empty stomach with NO other medication. The patient should sit up for 30 minutes (60 minutes with Boniva) after taking the medication, and not eat anything for 1 hour after taking (helps the body absorb more of the medicine.

4. Thinking back to the scenario in question 3, what other signs and symptoms are associated with appendicitis. SELECT-ALL-THAT-APPLY: Increased red blood Cells Patient has the desire to be positioned in the prone position to relieve pain Umbilical pain that extends in the right lower quadrant Abdominal rebound tenderness Abdominal Flaccidity

The answers are: C and D. These are classic signs and symptoms found in patients with appendicitis. Option A is wrong because the patient may have increased WHITE blood cells (not red). Option B is wrong because the patient may have the desire to be in the fetal position (side-lying with the knees bent) to relieve the pain. The prone position would increase the pain. Option E is wrong because the patient would have abdominal RIGIDITY (not flaccidity).

A client presents in the emergency department with complaints of cough, headache, and generalized aches and pains. Upon assessment, the nurse documents a temperature of 102.5°F (39.2°C) and redness on the arms, legs, and upper chest. She also notes that the client takes eight different medications each day. What nursing diagnosis is the priority for this client? Impaired physical mobility Impaired tissue integrity Impaired thermoregulation Ineffective therapeutic regimen management

The client is showing signs of Stevens-Johnson Syndrome (SJS), which is triggered by a reaction to medications. Signs and symptoms of SJS include conjunctival burning, fever, cough, sore throat, headache, aches and pains, and erythema and mucous membranes. As the disease progresses, large portions of the epidermis are shed, exposing the dermis and causing tender skin and a weeping surface. Keeping the tissue intact is the main priority for this client. Although Impaired physical mobility, Impaired thermoregulation, and Ineffective therapeutic regimen management apply to this client, these nursing diagnoses are lower priorities than Impaired tissue integrity.

3. A 23 year old patient is admitted with suspected appendicitis. The patient states he is having pain around the umbilicus that extends into the lower part of his abdomen. In addition, he says that the pain is worst on the right lower quadrant. The patient points to his abdomen at a location which is about a one-third distance between the anterior superior iliac spine and umbilicus. This area is known as what?

This is known as McBurney's Point and is a classic sign and symptom in patients with appendicitis.

D For the first 24 hours after a lower extremity cast is applied, the leg should be elevated on pillows above heart level to avoid excessive edema and compartment syndrome. A plaster cast will also be drying during this 24-hour period. RICE is used for soft tissue injuries, not with long leg casts.

This morning a 21-yr-old male patient had a long leg cast applied, and he asks to crutch walk before dinner. Which statement explains why the nurse will decline the patient's request? "No one is available to assist and accompany the patient." "The cast is not dry yet, and it may be damaged while using crutches." "Rest, ice, compression, and elevation are in process to decrease pain." "Excess edema and complications are prevented when the leg is elevated for 24 hours."

Which of the following terms refers to disease of a nerve root? a) Radiculopathy b) Contracture c) Involucrum d) Sequestrum

a) Radiculopathy Explanation: When the patient reports radiating pain down the leg, he or she is describing radiculopathy. Involucrum refers to new bone growth around the sequestrum. Sequestrum refers to dead bone in an abscess cavity. Contracture refers to abnormal shortening of muscle or fibrosis of joint structures. pg.1133

The nurse is educating the patient with low back pain about the proper way to lift objects. What muscle should the nurse encourage the patient to maximize? a) Gastrocnemius b) Quadriceps c) Rectus abdominis d) Latissimus dorsi

b) Quadriceps Explanation: The nurse instructs the patient in the safe and correct way to lift objects using the strong quadriceps muscles of the thighs, with minimal use of weak back muscles (Fig. 42-3). pg.1135

A client is admitted with acute osteomyelitis that developed after an open fracture of the right femur. When planning this client's care, the nurse should anticipate which measure? a) Instructing the client to ambulate twice daily b) Administering large doses of I.V. antibiotics as ordered c) Withholding all oral intake d) Administering large doses of oral antibiotics as ordered

b) Administering large doses of I.V. antibiotics as ordered Explanation: Treatment of acute osteomyelitis includes large doses of I.V. antibiotics (after blood cultures identify the infecting organism). Surgical drainage may be indicated, and the affected bone is immobilized. The client usually requires I.V. fluids to maintain hydration, but oral intake isn't necessarily prohibited. pg.1148

What food can the nurse suggest to the client at risk for osteoporosis? a) Carrots b) Broccoli c) Bananas d) Chicken

b) Broccoli Explanation: Calcium is important for the prevention of osteoporosis. Broccoli is high in calcium. pg.1141

A client with Paget's disease comes to the hospital and complains of difficulty urinating. The emergency department physician consults urology. What should the nurse suspect is the most likely cause of the client's urination problem? a) Urinary tract infection (UTI) b) Renal calculi c) Benign prostatic hyperplasia d) Dehydration

b) Renal calculi Explanation: Renal calculi commonly occur with Paget's disease, causing pain and difficulty when urinating. A UTI commonly causes fever, urgency, burning, and hesitation with urination. Benign prostatic hyperplasia is common in men older than age 50; however, because the client has Paget's disease, the nurse should suspect renal calculi, not benign prostatic hyperplasia. Dehydration causes a decrease in urine production, not a problem with urination. pg.1142

A nurse is performing discharge teaching for an elderly client diagnosed with osteoporosis. Which statement about home safety should the nurse include? a) "Most falls among the elderly occur outside the home. Clients should confine themselves to their homes as much as practical." b) "Because of the increase in home burglaries involving the elderly, these clients should have burglar bars on every window in the home." c) "Most falls among the elderly occur in the home. These clients should remove throw rugs and install bathroom grab bars." d) "Most accidental injuries among the elderly are automobile-related. Elderly clients should have vision testing every 6 months while they're still driving."

c) "Most falls among the elderly occur in the home. These clients should remove throw rugs and install bathroom grab bars." Explanation: Falls in the home cause most injuries among the elderly. Elderly clients should take measures to decrease the clutter that can contribute to falls, such as removing objects such as throw rugs from the floor. Elderly clients should also install grab bars in the shower and next to the toilet. The threat of fire makes burglar bars on every window impractical. pg.1144

Which is a risk-lowering strategy for osteoporosis? a) Diet low in calcium and vitamin D b) Increased age c) Smoking cessation d) Low initial bone mass

c) Smoking cessation Explanation: Risk-lowering strategies include increased dietary calcium and vitamin D intake, smoking cessation, alcohol and caffeine consumption in moderation, and outdoor activity. Individual risk factors include low initial bone mass and increased age. A lifestyle risk factor is a diet low in calcium and vitamin D. pg.1141

A nurse is caring for a client who's experiencing septic arthritis. This client has a history of immunosuppressive therapy and his immune system is currently depressed. Which assignment is the most appropriate for the nurse caring for this client? a) The nurse caring for this client is also caring for four clients receiving chemotherapy for cancer treatment on the oncology floor. b) The nurse caring for this client is also caring for two other immunosuppressed clients on the medical intensive care unit. c) The nurse is caring for this client on the intensive care unit. d) The nurse caring for this client is also caring for four other immunosuppressed clients on the medical floor.

c) The nurse is caring for this client on the intensive care unit. Explanation: This client is critically ill; his diagnosis and immunosuppression place him at a high risk for infection. The most appropriate place for this client is in an intensive care unit, where the nurse can focus exclusively on his health promotion. This client shouldn't be on the oncology floor. This client requires close monitoring. The nurse caring for this client shouldn't also be caring for other clients who may require frequent interventions. pg.1151

The nurse is caring for a client following foot surgery. Which nursing intervention is most important for the nurse to include in the nursing care plan? a) Monitor vital signs every 4 hours. b) Administer pain medication per client request. c) Examine surgical dressing every hour. d) Perform neuromuscular assessment every hour.

d) Perform neuromuscular assessment every hour. Explanation: The priority nursing intervention is to perform a neuromuscular assessment every hour. Early detection of neurological and perfusion problems is critical. pg.2061

A patient had hand surgery to correct a Dupuytren's contracture. What nursing intervention is a priority postoperatively? a) Applying a cock-up splint and immobilization b) Changing the dressing c) Having the patient exercise the fingers to avoid future contractures d) Performing hourly neurovascular assessments for the first 24 hours

d) Performing hourly neurovascular assessments for the first 24 hours Explanation: Hourly neurovascular assessment of the exposed fingers for the first 24 hours following surgery is essential for monitoring function of the nerves and perfusion. pg.1137

A 70-year-old client with a diagnosis of left-sided stroke is admitted to the facility. To prevent the development of disuse osteoporosis, which objective is most appropriate? a) Maintaining vitamin levels b) Maintaining protein levels c) Promoting range-of-motion (ROM) exercises d) Promoting weight-bearing exercises

d) Promoting weight-bearing exercises Explanation: When the mechanical stressors of weight bearing are absent, disuse osteoporosis can occur. Therefore, if the client does weight-bearing exercises, disuse complications can be prevented. Maintaining protein and vitamins levels is important, but neither will prevent osteoporosis. ROM exercises will help prevent muscle atrophy and contractures. pg.1141

A patient diagnosed with osteoporosis is being discharged home. Which of the following is the priority education the nurse should provide? a) Classifying medications b) Increasing calcium and vitamin D in the diet c) Participating in weight-bearing exercises d) Removing all small rugs from the home

d) Removing all small rugs from the home Explanation: A patient with osteoporosis is at risk for fractures related to falls. The home environment needs to be evaluated for safety issues, such as rugs and other objects that could cause a fall. All other education is important in educating the patient, but the risk for injury with a fall and potential for a fracture makes safety in the home environment a priority. pg.1143

Aisha Griffin is a​ 42-year-old mother of three who has returned to your practice for a​ follow-up visit regarding her fibromyalgia. You are interviewing her prior to her examination. Which statement from Mrs. Griffin would indicate the need for further teaching regarding​ self-care? ​"I've been pretty good about walking outside for at least 30 minutes every​ day." ​"I tried some pain medicine my friend had and it worked even though it made me​ sleepy." ​"Sometimes I get really frustrated about being tired so​ much." ​"I've been trying to go to bed at the same time every day and make sure I​ don't get on the computer for a couple of hours before​ bedtime."

​"I tried some pain medicine my friend had and it worked even though it made me​ sleepy." Mild to moderate exercise and consistent sleep habits have been proven effective at reducing the severity of fibromyalgia symptoms. Frustration is a​ normal, common response to symptoms. It is dangerous to take medicines prescribed to someone else​ and, based on the description of side​ effects, this medication may have been a​ narcotic, which can worsen activity intolerance.

The nurse is providing education about ongoing care for a client with fibromyalgia. Which information should the nurse​ include? ​(Select all that​ apply.) Use of prescription medications ​Follow-up care The use of cold therapy Strategies for stress reduction Information on contacting support organizations

​Follow-up care Use of prescription medications Strategies for stress reduction Information on contacting support organizations The nurse should educate the client about available support organizations such as the Fibromyalgia Network and the American College of Rheumatology.The nurse should educate the client about reducing stress to assist in managing symptoms of fibromyalgia.The nurse should educate the client about the importance of keeping​ follow-up appointments with healthcare providers.The nurse should educate the client about taking medications as prescribed. Cold therapy does not improve fibromyalgia symptoms.

Which patient is most likely to experience anemia related to an increased destruction of red blood cells? A 59-yr-old man whose alcoholism has precipitated folic acid deficiency A 23-yr-old African American man who has a diagnosis of sickle cell disease A 30-yr-old woman with a history of "heavy periods" accompanied by anemia A 3-yr-old child whose impaired growth and development is attributable to thalassemia

A 23-yr-old African American man who has a diagnosis of sickle cell disease

During assessment of the patient with fibromyalgia syndrome (FMS), the nurse would expect the patient to report (select all that apply)? a. sleep disturbances. b. multiple tender points. c. cardiac palpitations and dizziness. d. multijoint pain with inflammation and swelling. e. widespread bilateral, burning musculoskeletal pain.

ANS: A, B, C, E These symptoms are commonly described by patients with FMS. Cardiac involvement and joint inflammation are not typical of FMS.

A patient started Alendronate (Fosamax) once a week for the treatment of osteoporosis. The nurse determines that further instruction about the drug is needed when what is said by the patient? a-"I should take the drug with a meal to prevent stomach irritation". b-"This drug will prevent further bone loss and increase my bone density". c-"I need to sit or stand upright for at least 30 minutes after taking the drug". d-"I will still need to take my calcium supplements while taking this new drug ".

Answer: a The Bisphosphonate, such as Alendronate, must be taken at least 30 minutes before food or other medications to promote absorption

A patient has been diagnosed with acute myelogenous leukemia (AML). What should the nurse educate the patient that care will focus on? Leukapheresis Attaining remission One chemotherapy agent Waiting with active supportive care

Attaining remission

During a health screening event which assessment finding would alert the nurse to the possible presence of osteoporosis in a white 61-year-old female? A. The presence of bowed legs B. A measurable loss of height C. Poor appetite and aversion to dairy products D. Development of unstable, wide-gait ambulation

B. A measurable loss of height A gradual but measurable loss of height and the development of kyphosis or "dowager's hump" are indicative of the presence of osteoporosis in which the rate of bone resorption is greater than bone deposition. Bowed legs may be caused by abnormal bone development or rickets but is not indicative of osteoporosis. Lack of calcium and Vitamin D intake may cause osteoporosis but are not indicative it is present. A wide gait is used to support balance and does not indicate osteoporosis.

The nurse is admitting a patient who complains of a new onset of lower back pain. To differentiate between the pain of a lumbar herniated disc and lower back pain from other causes, what would be the best question for the nurse to ask the patient? A. "Is the pain worse in the morning or in the evening?" B. "Is the pain sharp or stabbing or burning or aching?" C. "Does the pain radiate down the buttock or into the leg?" D. "Is the pain totally relieved by analgesics, such as acetaminophen (Tylenol)?"

C. "Does the pain radiate down the buttock or into the leg?" Lower back pain associated with a herniated lumbar disc is accompanied by radiation along the sciatic nerve and can be commonly described as traveling through the buttock, to the posterior thigh, or down the leg. This is because the herniated disc causes compression on spinal nerves as they exit the spinal column. Time of occurrence, type of pain, and pain relief questions do not elicit differentiating data.

The nurse has reviewed proper body mechanics with a patient with a history of low back pain caused by a herniated lumbar disc. Which statement made by the patient indicates a need for further teaching? A. "I should sleep on my side or back with my hips and knees bent." B. "I should exercise at least 15 minutes every morning and evening." C. "I should pick up items by leaning forward without bending my knees." D. "I should try to keep one foot on a stool whenever I have to stand for a period of time."

C. "I should pick up items by leaning forward without bending my knees." The patient should avoid leaning forward without bending the knees. Bending the knees helps to prevent lower back strain and is part of proper body mechanics when lifting. Sleeping on the side or back with hips and knees bent and standing with a foot on a stool will decrease lower back strain. Back strengthening exercises are done twice a day once symptoms subside.

7. The nurse is reviewing T-scores for a 68-yearold woman. The patient has a T-score of 2.5. How does the nurse interpret this data? a. The patient has osteopenia. b. The patient has osteoporosis. c. This is a normal score for the patient's age. d. There is osteoblastic activity.

The patient has osteoporosis.

Of the following, which is not a risk factor for osteoporosis? a) Being male b) Small-framed, thin White or Asian women c) Being postmenopausal d) Family history

a) Being male Explanation: Being male is not considered a risk factor. The following are some of the risk factors for osteoporosis: being a small-framed, thin White or Asian women; being postmenopausal; family history; inactivity; chronic low calcium intake; and excessive caffeine or tobacco use. pg.1142

A client has Paget's disease. An appropriate nursing diagnosis for this client is: a) Fatigue b) Risk for falls c) Delayed wound healing d) Risk for infection

b) Risk for falls Explanation: The client with Paget's disease is at risk for falls secondary to pathological fractures and impaired gait/mobility. pg.1147

Which female patients are at risk for developing osteoporosis (select all that apply)? a. 60-year-old white aerobics instructor b. 55-year-old Asian American cigarette smoker c. 62-year-old African American on estrogen therapy d. 68-year-old white who is underweight and inactive e. 58-year-old Native American who started menopause prematurely

b. 55-year-old Asian American cigarette smoker. d. 68-year-old white who is underweight and inactive. e. 58-year-old Native American who started menopause prematurely. Risk factors for osteoporosis include age greater than 65, white or Asian ethnicity, cigarette smoking, inactive lifestyle, low body weight, and postmenopausal estrogen deficiency including premature menopause. Other factors include family history, excessive alcohol use, long-term use of medications such as corticosteroids, thyroid replacement, heparin, long-acting sedatives, or antiseizure drugs.

A patient with chronic osteomyelitis has been hospitalized for a surgical debridement procedure. What does the nurse explain to the patient as the rationale for the surgical treatment? a. Removal of the infection prevents the need for bone and skin grafting. b. Formation of scar tissue has led to a protected area of bacterial growth. c. The process of depositing new bone blocks the vascular supply to the bone. d. Antibiotics are not effective against microorganisms that cause chronic osteomyelitis.

b. Chronic infection of the bone leads to formation of scar tissue from the granulation tissue. This avascular scar tissue provides an ideal site for continued microorganism growth and is impenetrable to antibiotics. Surgical debridement is often necessary to remove the poorly vascularized tissue and dead bone and to instill antibiotics directly to the area. Involucrum is new bone laid down at the infection site, which seals off areas of sequestra. Antibiotics can be effective during acute osteomyelitis and prevention of chronic osteomyelitis requires early antibiotic treatment. Bone and skin grafting may be necessary following surgical removal of infection if destruction is extensive.


Set pelajaran terkait

AP Psychology - Social Psychology (Chapter 16)

View Set

RANDOM QUESTIONS 220 FLA - GENERAL LINES AGENT

View Set

Marketing Chapter 10 (Marketing research)

View Set

15 - German Possessive Pronouns - MEIN, DEIN, SEIN, IHR, UNSER

View Set

Adult Nursing - Chapter 43: Assessment of Digestive and Gastrointestinal Function - PrepU

View Set

Gender Issues (13-2 Title IX, 5 Media in Sport

View Set

(ORANGE BOOK)Therapeutic Equivalence Codes

View Set